CARE 2015 - Q&As

Download as pdf or txt
Download as pdf or txt
You are on page 1of 131

Case 1

A 45 year old woman with end-stage renal disease on hemodialysis with recent
infection presents with painful nodules on her abdomen, buttocks and thigh. She
has a past medical history of end-stage renal disease requiring dialysis for the
past 2 years. She had an arteriovenous graft infection 4 weeks ago treated with
intravenous nafcillin for 2 weeks, and she now uses a temporary hemodialysis
catheter. She has a history of hypertension, which had been poorly controlled for
many years before presentation with end-stage renal disease. She is being
treated with amlodipine 10 mg daily and lisinopril 20 mg daily to control her blood
pressure. She also takes 1,25-dihydroxyvitamin D3 and calcium
supplementation. She was in her usual state of health until about 3 weeks ago
when she developed intensely painful skin lesions on her anterior abdomen,
buttocks, and thigh. One of the skin lesions ulcerated as shown below (Figure).
The physical examination reveals an ill-appearing female in some distress due to
the painful skin lesions. Her blood pressure is 160/94 mm Hg, heart rate is 92
bpm, and temperature is 98.9°F (37.2°C). The skin examination reveals several
tender raised nodules with a violaceous discoloration located on the abdomen,
buttocks, and thigh. Neurologic examination reveals absent ankle reflexes
bilaterally with decreased sensation distally. No focal findings are detected. The
remainder of the physical examination is unremarkable. Laboratory studies are
listed below (Table), and a skin biopsy is shown in the figure.

Figure. Clinical and Histologic Images of Skin Lesions


Table. Results of Laboratory Tests
Laboratory Test Result Reference Range

Albumin, serum 3.6 g/dL 3.5–5.5 g/dL

Alkaline phosphatase, serum 98 U/L 30–120 U/L

Aminotransferase, serum alanine (ALT, SGPT) 35 U/L 10–40 U/L

Aminotransferase, serum aspartate (AST, SGOT) 32 U/L 10–40 U/L

Anti–double-stranded DNA antibodies 2.0 IU/mL 0–7 IU/mL

Antinuclear antibodies 1:40 Greater than 1:40 is abnormal

Antistreptolysin O titer 180 Todd units Less than 200 Todd units

Calcium, serum 8.2 mg/dL 8.6–10.2 mg/dL

Complements, serum
C3 94 mg/dL 100–233 mg/dL
C4 16 mg/dL 14–48 mg/dL

Creatinine, serum 8.2 mg/dL 0.7–1.5 mg/dL

Hemoglobin, blood 10.2 g/dL Female: 12–16 g/dL


Hepatitis B surface antigen Negative Negative

Hepatitis C antibody Negative Negative

Leukocyte count 8500/μL 4000–11,000/μL

Parathyroid hormone (intact), serum 135 pg/mL 10–65 pg/mL

Phosphorus, serum 7.5 mg/dL 3.0–4.5 mg/dL

Platelet count 180,000/μL 150,000–300,000/μL

Click here for references

Which of the following is the most likely diagnosis given the clinical history and
biopsy results?

A. Calciphylaxis

B. Cholesterol embolization

C. Cutaneous leukocytoclastic vasculitis

D. Septic emboli

Rationale

This patient with end-stage renal disease presents with painful nodules on her
abdomen, buttocks, and thigh. Several diagnoses should be considered for this
patient including a vasculitis and some imitators of vasculitis such as cholesterol
embolization and calciphylaxis. Calciphylaxis is an uncommon disorder
characterized by calcification of the arterioles leading to ischemia and
subcutaneous necrosis. It mainly occurs in patients with end-stage renal disease
on dialysis but can affect patients with earlier-stage renal disease. The laboratory
studies reveal elevated phosphorus and parathyroid hormone levels, which are
commonly found in this condition. The biopsy shows medial arteriolar
calcification, subintimal fibrosis, and arterial occlusion in the absence of vasculitic
change. Calcification of the subcutis may also be seen.
Cholesterol crystal embolization is another imitator of vasculitis. It is more likely
found in an older patient following some type of vascular procedure. The skin
lesions in this condition can include livedo reticularis and “blue toe” syndrome.
Biopsy is the definitive means to confirm the cholesterol embolization syndrome.
The histologic finding includes the presence of ghosts of cholesterol crystals,
since the crystals are dissolved during tissue fixation. Cholesterol clefts are
crescentic or elongated ovoid spaces present in the small or medium arteries or
arterioles.
Cutaneous small vessel vasculitis (Hypersensitivity vasculitis) often presents with
palpable purpura often in the dependent lower extremities. The biopsy reveals
inflammation in the small blood vessels, most prominent in the postcapillary
venules.
Septic emboli are unlikely to occur without deep seated infection such as
infective endocarditis. This patient is afebrile, and biopsy results are not
consistent with infection. The nodules due to calciphylaxis are very painful and
extremely firm. They may be located in more proximal locations compared with
septic emboli.
Educational Objective: Recognize calciphylaxis due to hyperparathyroidism as
an imitator of vasculitis.

Case 2
A 55-year-old man with seropositive rheumatoid arthritis (RA) for the past 3 years
comes in for regular follow-up. Clinical measures of disease activity show that
his RA is in remission.

Click here for references

What musculoskeletal ultrasound (MSUS) finding would be the most predictive of


progressive structural damage in his joints?

A. Anechoic intra-articular material that is displaceable and results in swirling


echoes on compression

B. Hyperechoic enhancement of the superficial margin of the articular


cartilage

C. Intra-articular discontinuity of the bone surface that is visible in orthogonal


planes

D. Noncompressible, hypoechoic intra-articular tissue, with positive power


Doppler signal

Rationale
It is well established that patients with RA who are in clinical remission may go
on to have structural progression of disease. Remission criteria represent
composite indices of clinical outcomes, including tender and swollen joint counts,
patient and physician global assessments, and inflammatory markers
(erythrocyte sedimentation rate or C-reactive protein). Current remission
composites may lack sensitivity to detect subclinical inflammation, which may
persist and result in progressive structural damage.
Newer imaging modalities (MSUS and magnetic resonance imaging) have shown
that patients who have achieved clinical remission according to multiple
composite indices may have persistent subclinical inflammation. Follow-up
studies of these patients have shown a significant increase in the risk of relapse
and structural progression.
The finding of synovitis on MSUS is defined by the presence of noncompressible,
hypoechoic intra-articular tissue. Power Doppler (PD) signal shows increased
vascularity of the synovium that is associated with active inflammation. A grading
scale for each of these components exists to help quantify the degree of synovitis
and PD signal. Gray-scale MSUS can show hypertrophy of the synovium, which
can become chronically thickened and less reversible in long-standing RA.
However, synovitis with positive PD signal has been shown to be the strongest
predictor of relapse and progressive structural damage in patients with RA,
despite achieving clinical remission by current criteria. Figure 1 shows the dorsal
metacarpophalangeal (MCP) joint in the long and transverse axis.

Figure 1. Dorsal MCP Joint With Synovitis and Positive PD Signal in Long
and Transverse Axis
Anechoic intra-articular material that is displaceable and results in swirling
echoes on compression defines a fluid collection in the joint space. Synovial fluid
may indicate synovial inflammation, but in the absence of synovitis with PD
positivity, it is not predictive of progression. By definition, synovial fluid is PD
negative, and the nature of the fluid cannot be determined by MSUS. Aspiration
would be required to determine if fluid is inflammatory. Figure 2 shows a small
MCP effusion.
Figure 2. Dorsal and Volar MCP Joint With Effusion in Longitudinal and
Transverse Axis
MCP Joint Longitudinal MCP Joint
Transverse
Top panels are volar; bottom panels are dorsal.
1. Phalanx; 2. Metacarpal; 3. Synovial effusion.

Figure 3. Dorsal MCP Joint With Double Contour Sign in Longitudinal and
Transverse Axis
Intra-articular discontinuity of the bone surface that is visible in orthogonal planes
is the definition of erosion seen by MSUS. Although erosion scores are
calculated to follow disease progression, erosions on MSUS have not been
shown to be predictive of progression in patients in clinical remission by
composite criteria. Figure 4 shows erosions in the dorsal proximal
interphalangeal (PIP) joint.
Figure 4. Dorsal PIP Joint With Erosions in Longitudinal and Transverse
Axis

1. Proximal phalanx; 2. Erosion.


All photos courtesy and copyright of the Ultrasound School of North American
Rheumatologists (USSONAR), pathology guide 2014. www.ussonar.org.

Case 3
A 25-year-old man presents with symptoms of severe pain in his lower back and
neck for the past 2 years. He is stiff for more than 4 hours in the morning and
after periods of inactivity. His symptoms were only mildly improved by treatment
with anti-inflammatory doses of nonsteroidal anti-inflammatory drugs. He
immigrated to the United States from South Africa 5 years ago and travels to
South Africa yearly for vacation.
His physical examination reveals normal vital signs and normal ophthalmologic,
cardiac, pulmonary, skin, and gastrointestinal systems. His joint examination is
normal with the exception of a reduced Schober expansion of 3 cm and reduced
neck rotation to 65 degrees bilaterally and lateral flexion to 25 degrees bilaterally.
He also has mild tenderness to palpation of the right Achilles tendon insertion.
His laboratory tests show a normal complete blood count and metabolic profile.
Erythrocyte sedimentation rate is 40 mm/hr (reference range [male]: 0–15
mm/hr). Radiographs of the spine reveal mild sclerosis of the anterior corners of
the lumbar vertebral bodies.
Previously, he was treated with multiple nonsteroidal anti-inflammatory drugs
(NSAIDs) without benefit and did not respond to adalimumab.
Click here for references

Blockade of which of the following pathways gives the best possibility of clinical and
radiographic improvement outcomes?

A. Interleukin-4/interleukin-10

B. Interleukin-6/receptor activator of nuclear factor kappa-B ligand

C. Phosphodiesterase type 5


D. Interleukin-17/interleukin-23

Rationale

This patient fits the profile of ankylosing spondylitis (AS) clinically, and this
diagnosis is supported by his radiographic findings and elevation in inflammatory
marker. He comes from an area endemic for tuberculosis. He has not received
benefit with NSAID therapy. Studies have shown that the interleukin 23 receptor
(IL23R) R381Q gene variant protects from the development of AS through
selective impairment of IL-17A production. Furthermore, IL-23 is over expressed
in AS spinal facet joints but not in osteoarthritis facet joints, whereas increase in
IL-17 has resulted in increased ankylosing enthesitis in mice. Blockade of IL-23
has also been shown to improve AS symptoms in a human open-labeled trial.
Blockade of IL-17 with secukinumab has also been shown to improve AS
symptoms in a human randomized, double blind, placebo-controlled trial and to
reduce spinal inflammation as measured by magnetic resonance imaging. A
potential additional benefit to targeting these pathways is that studies of IL-23
inhibition (ustekinumab) did not show increased risk of tuberculosis. As IL-17 is
primarily involved in protection against extracellular pathogens, it would not be
expected to increase risk of tuberculosis but would be expected to increase the
risk of fungal infections. Long term clinical trials will need to further address this
question.
The other answer choices are incorrect. The IL-4/IL-10 pathway interruption has
not been shown to be effective for the treatment of AS. The IL-6 blockade has
been studied in AS and was not found to be beneficial. Phosphodiesterase 5
inhibition has been shown to be effective in the treatment of psoriasis and
psoriatic arthritis but has not been studied in AS.
Educational Objective: Recognize novel treatment pathways for ankylosing
spondylitis

Case 4
A 52-year-old man has been followed up in your clinic for 10 years for
seropositive erosive rheumatoid arthritis. Previously he was treated with
combinations of methotrexate together with etanercept or adalimumab but has
had persistent disease activity despite the addition of prednisone 10 mg daily.
You are now considering discontinuation of his current biologic therapy and
initiating treatment with abatacept.
Click here for references

Which of the following pathways does abatacept inhibit?


A. Fc-receptor mediated signaling

B. Interaction of CD28 with antigen presenting cells

C. Interaction of interleukin 17 with its receptor

D. Janus kinase-signal transducer and activator of transcription mediated


signaling

E. Type I interferon responses

Rationale

Co-stimulation, or engagement of cognate ligands and receptors on antigen


presenting cells and T cells, is essential for adaptive immune responses. Major
histocompatibility complex class II presentation of a given peptide to the
appropriate T cell receptor, together with a second signal provided by co-
stimulation, is usually required for full immune activation. However, not only
activating signals are provided by co-stimulation; the second signal received can
be inhibitory (co-inhibition). The correct balance between co-stimulation and co-
inhibition is necessary to keep immune responses in check and in maintaining
peripheral tolerance. Multiple ligand/receptor co-stimulatory/co-inhibitory pairs
have been recognized and characterized, and more are being discovered.
One important activating signal is mediated by the T cell ligand CD28, which
binds both B7.1 (CD80) and B7.2 (CD86) on antigen presenting cells. Cytotoxic
T-lymphocyte associated antigen 4 (CTLA-4) on T cells, which is structurally
related to CD28, binds to these same receptors but has an opposite and
inhibitory effect. Temporally, the activating CD28 signal comes up first, promoting
immune activation, while CTLA-4 expression makes a delayed appearance, to
turn off inflammation. Abatacept, a biologic approved for the treatment of
rheumatoid arthritis, is this CTLA-4 molecule engineered with an Fc tail to
increase its in vivo half-life. Since CTLA-4 has a much higher avidity to B7 than
CD28, administration of CTLA-4 will prevent binding by CD28 to the antigen
presenting cell and T cell activation, with decreased T cell proliferation and
cytokine production.
Although earlier studies focused mostly on the regulation of T cell activation by
CTLA-4 via its ectodomain as described above, more recent studies have
described contributions of CTLA-4 to the regulation of T cells via other
mechanisms as well. These include interaction of the CTLA-4 cytoplasmic
domain with intracellular tyrosine phosphatases to modify the T cell activation
threshold, regulation of regulatory T cells (and thus indirectly influencing
conventional T cells), and modulating the development of T cells in the thymus
(central tolerance).
The success of biologic treatment in patients with rheumatoid arthritis,
spondyloarthropathies, and systemic lupus erythematosus, and improvement in
our understanding of important immunologic pathways, have encouraged the
development of multiple novel biologics and small molecules for the treatment of
rheumatic diseases. In the arena of small molecules (that can be given orally),
tofacitnib, an inhibitor of Janus kinase (JAK) signaling, is approved for the
treatment of rheumatoid arthritis, whereas additional inhibitors targeting spleen
tyrosine kinase (SyK), Bruton’s tyrosine kinase (Btk), and other signaling
pathways are in development. Inhibition of interleukin 17 by monoclonal
antibodies is showing promise for psoriasis and psoriatic arthritis, whereas
antibodies targeting type I interferon are being studied in lupus.
Educational Objective: Understand how CTLA4 regulates T cell function and
the mechanism of action of abatacept.

Case 5
A 44-year-old man presents with a new diagnosis of eosinophilic granulomatosis
with polyangiitis.
The patient originally presented with fevers, dyspnea, and migratory
polyarthralgias.
On physical examination, the patient is noted to have tachycardia with a
pericardial rub and decreased breath sounds at the bases, but otherwise clear to
auscultation. Mild synovitis of bilateral wrists and knees is noted.
Laboratory results reveal eosinophilia, elevated serum creatinine level,
proteinuria, elevated erythrocyte sedimentation rate and C-reactive protein level,
and positive titer of perinuclear antineutrophil cytoplasmic (antimyeloperoxidase)
antibodies (p-ANCA). Chest imaging shows bilateral peripheral opacities with
multiple nodules and small pleural effusion. Renal biopsy shows crescentic
glomerulonephritis.
Click here for references
Which of the following treatment options has the best evidence as an efficacious
treatment for this patient in addition to glucocorticoids?

A. Azathioprine

B. Cyclophosphamide

C. Methotrexate

D. Rituximab

Rationale

The mainstay of therapy for eosinophilic granulomatosis with polyangiitis (EGPA)


is systemic glucocorticoids. In patients with poor prognostic features, as in this
case, additional initial treatment with cyclophosphamide has been associated
with improved mortality.
Poor prognostic features in the systemic necrotizing vasculitides (polyarteritis
nodosa, microscopic polyangiitis, granulomatosis with polyangiitis, and EGPA),
include age > 65 years, cardiac symptoms, gastrointestinal involvement, and
renal insufficiency. Absence of ear, nose, and throat symptoms is also
associated with poor prognosis in patients with granulomatosis with polyangiitis
and EGPA. In this case, the patient has a new onset of renal insufficiency in the
setting of crescentic glomerulonephritis, pneumonitis, and pericarditis, making
this a severe presentation of the disease.
Azathioprine is typically used as a maintenance medication after remission is
induced by the cyclophosphamide, and has been used for relapsing diseaseWe
do not have data to support azathioprine as the best option for initial induction
therapy in a patient with EGPA with poor prognostic features.
Similarly, we do not have data to support methotrexate as the best option for
initial induction therapy in a patient with EGPA with poor prognostic features.
Methotrexate can be used as a maintenance medication, but use of methotrexate
can be challenging and risky in the context of pulmonary andrenal disease.
Presently only a few case reports and case series support use of rituximab in
patients with eosinophilic granulomatosis with polyangiitis; we are lacking any
controlled data on this agent in EGPA. This is in contrast to other ANCA-
associated vasculitides, granulomatosis with polyangiitis and microscopic
polyangiitis, where rituximab has been shown to be equally efficacious as
cyclophosphamide for initial therapy and better for treatment of relapsing
disease.
Educational Objective: To recognize that in patients with severe eosinophilic
granulomatosis with polyangiitis manifestations, cyclophosphamide is an
effective therapy with best available evidence.

Case 6
A 33-year-old woman presents to your clinic for evaluation of right shoulder
pain. She played college volleyball and was known for her excellent serve. She
recalls no specific injury to the right shoulder. Her primary recreational activity is
playing in a competitive volleyball league, and she is frustrated by a recent
decline in her performance. She reports pain in the right shoulder and down into
the right scapula. She has discomfort with lifting her right arm overhead and pain
in her back when leaning back against a chair. The physical examination reveals
no effusion or warmth in the right shoulder, no evidence of muscle wasting, and
well-healed surgical scars. A push-off test against the wall reveals medial
winging of the right scapula. Results of a right shoulder radiograph are normal.
Click here for references

Which of the following best describes the injury and is the most appropriate
treatment for the above scenario at this juncture?

A. Long thoracic nerve injury with nonoperative management with bracing


and observation

B. Long thoracic nerve injury with operative pectoralis transfer

C. Spinal accessory nerve injury with referral to physical therapy for trapezius
strengthening

D. Spinal accessory nerve injury with operative exploration of the nerve

Rationale

The scapular muscles contribute to keeping the scapula aligned against the
thoracic wall. Paralysis of one of the nerves innervating the muscles of the
scapula will result in scapular winging. Medial winging, such as that described in
the case scenario, is the most common form of scapular winging and results from
injury to the long thoracic nerve and subsequent deficit of the serratus anterior
muscle. The superior medial scapula will elevate and migrate medially with this
physical finding exacerbated during a push-off test from the wall. Injury to the
long thoracic nerve can occur with repetitive stretching of the neck in activities
such as volleyball, weight lifting, or football. Patients will complain of shoulder
and scapula pain upon presentation. Conservative therapy with bracing and
observation and strengthening of the serratus anterior with physical therapy are
the first course of action in this case.
Pectoralis transfer can be done if there is no response to a sufficient trial of
conservative therapy. As the patient has not completed a trial of conservative
therapy, surgery would not be indicated. Damage to the spinal accessory nerve
results in a weakness of the trapezius muscle, with the medial aspect of the
scapula dropping down and moving laterally. Injury to the spinal accessory nerve
typically results from surgical exploration of the neck. Treatment can involve
observation and trapezius strengthening or additional surgery in the form of a
nerve exploration.
Educational Objective: Identify different patterns and causes of scapular
winging.

Case 7
A 25-year-old woman with a 5-year history of inflammatory lower back pain
presents for further management of her symptoms. She also notes alternating
buttock pain and left heel pain. Her calculated Bath Ankylosing Spondylitis
Disease Activity Index (BASDAI) score is 6.20, corresponding to high disease
activity.
She has no history of psoriasis, inflammatory bowel disease, conjunctivitis, or
uveitis. She also denies history of peripheral arthritis and enthesitis. She has a
positive family history of ankylosing spondylitis (AS) in her father but denies
family history or personal history of psoriasis or inflammatory bowel disease. On
examination, her vital signs are stable. No psoriatic skin or nail changes are
noted. She has no synovitis. She does have tenderness in her left heel at the
origin of her plantar fasciia. Patrick’s maneuvers are positive bilaterally.
Sacroiliac radiographs with Ferguson’s views are in the normal range. Her
baseline laboratory tests reveal a C-reactive protein (CRP) level of 7.4 mg/dL
(reference range: 0.8 mg/dL or less) and a human leukocyte antigen (HLA)-B27
test that is positive.
Click here for references

Based on the clinical data presented, which feature of her condition is associated
with structural damage related to her inflammatory spondylitis?

A. Elevated CRP level

B. Family history of AS

C. High BASDAI score

D. Positive test for HLA-B27

Rationale

The Assessment of SpondyloArthritis international Society (ASAS) classification


criteria for axial spondyloarthritis (SpA) require a history of chronic back pain for
3 or more months and an age at onset of less than 45 years. In addition, one of
the following needs to be present: 1) sacroiliitis on radiographs or magnetic
resonance imaging and 1 or more typical SpA features or 2) the presence of
HLA–B27 and 2 or more typical SpA features. Patients who fulfil these criteria but
have NOT developed radiographic changes in the sacroiliac joints are classified
as nonradiographic axial SpA (nr-axSpA). Typical SpA features include
peripheral arthritis, peripheral enthesitis, dactylitis, alternating buttock pain, and
anterior uveitis. This patient has appropriate back pain and age of onset but
does not have appropriate imaging findings. As she does have a positive HLA-
B27 test, alternating buttock pain, and enthesitis, she meets the criteria for nr-
axSpA.
In a cross-sectional analysis of 462 patients who were participants in the German
Spondyloarthritis Inception Cohort, 226 patients with nr-axSpA (symptom
duration ≤ 5 years) and 236 patients with AS (symptom duration ≤ 10 years) were
compared to identify parameters associated with structural damage. In both
groups, HLA–B27 positivity determined the age at disease onset but was not
necessarily a predictor of disease progression. Male sex and an elevated C-
reactive protein level were associated with radiographic sacroiliitis and the
presence of syndesmophytes. In the German cohort, positive family history was
not a factor predictive of radiographic progression. For this patient, the family
history and HLA status affect her risk of developing the disease but not her risk of
progression to structural damage. The elevated CRP level, however, is
associated with structural damage, as seen in the above study.
No statistically significant difference was found between the nr-axSpA and AS
cohorts in disease activity scores and functional assays including the BASDAI,
Bath Ankylosing Spondylitis Functional Index (BASFI), Short Form 36 health
survey (SF-36), mental component summary (MCS), physical component
summary (PCS), and Ankylosing Spondylitis Quality of Life questionnaire
(ASQoL).
Educational Objective: Recognize the spectrum of axial spondyloarthropathy
and association of CRP elevation with radiographic disease.

Case 8
A 42-year-old man presents for follow-up of his seropositive rheumatoid arthritis
(RA). He has been compliant with taking methotrexate 20 mg once weekly and
folic acid 1 mg once daily. He has had no side effects or abnormal laboratory
findings. The physical examination reveals 8 tender joints and 3 swollen joints.
The Physician Global Assessment score is 40 mm, and the Patient Global
Assessment score is 50 mm. Hand radiographs show joint space narrowing and
erosive disease. The patient asks you about starting a biologic therapy but is
reluctant due to the cost of therapy.

Click here for references

Which combination therapy below has been shown to be noninferior to the


combination of methotrexate and etanercept in established RA?

A. Methotrexate and leflunomide

B. Methotrexate, hydroxychloroquine, and minocycline


C. Methotrexate, sulfasalazine, and hydroxychloroquine

D. Methotrexate, sulfasalazine, and prednisone

Rationale

This patient with established RA and a poor prognosis has moderate disease
activity despite therapy with methotrexate at an adequate dose. According to the
2012 American College of Rheumatology (ACR) recommendations for the use of
disease-modifying antirheumatic drugs (DMARD) and biologic agents in the
treatment of RA, acceptable choices for change in therapy would include: adding
or switching to another DMARD or adding a biologic agent such as a tumor
necrosis factor inhibitor, abatacept, or rituximab.
A recently published randomized controlled trial evaluated a population of
patients with established RA and an incomplete response to methotrexate. In the
Rheumatoid Arthritis Comparison of Active Therapies (RACAT) trial, patients
were randomized to receive either etanercept (biologic combination) or
sulfasalazine and hydroxychloroquine (triple combination) in addition to baseline
methotrexate. The primary endpoint was the change in the Disease Activity
Score for 28 joints (DAS28) at week 48. The results showed that triple
combination therapy was noninferior to biologic combination therapy (P = .002).
Importantly, patients in both groups had significant improvement in the first 24
weeks, but the protocol allowed for patients to switch groups at this time if they
did not meet a predefined improvement in the DAS28. The switchover rate did
not differ between the study groups (27% in each). Patients who switched at
week 24 continued to have significant improvement by the end of the study. This
outcome has shown a more cost-effective strategy to be effective in patients with
RA, nonresponsive to methotrexate.
The other treatment strategies are acceptable choices, according to the 2012
ACR recommendations, but none has been shown to be noninferior to a biologic
therapy.
Educational Objective: Triple disease-modifying antirhuematic drug therapy
(methotrexate, hydroxychloroquine, and sulfasalazine) is not inferior to
methotrexate plus etanercept in the treatment of adult rheumatoid arthritis.

CASE 9
The presence of which of the following has the highest positive predictive value for
pregnancy loss after 12 weeks gestation in women with antiphospholipid
antibodies?

A. Anti-beta-2 glycoprotein I antibody

B. Anticardiolipin antibody

C. Lupus anticoagulant

D. Prior pregnancy loss

Rationale

The presence of lupus anticoagulant was found to the primary predictor of


adverse pregnancy outcome after 12 weeks gestation in a multicenter,
prospective, observational study of women with antiphospholipid antibodies.
Between 2003 and 2011, 144 pregnant patients enrolled; 28 had adverse
pregnancy outcomes. Women enrolled for this prospective study were patients
with antiphospholipid antibodies including those with lupus anticoagulant,
antibodies to cardiolipin or beta-2-glycoprotein l and/or a diagnosis of systemic
lupus erythematosus. Among the women with lupus anticoagulant, 39% had
adverse pregnancy outcome, compared with only 3% of those without lupus
anticoagulant (P < .0001).
The presence of anticardiolipin and anti-beta-2 glycoprotein I antibodies, in the
absence of lupus anticoagulant, did not predict adverse pregnancy outcome.
Specifically, the presence of immunoglobulin (Ig) M anticardiolipin, IgG anti–beta-
2 glycoprotein I, and IgM anti–beta-2 glylcoprotein I antibodies alone did not
predict adverse pregnancy outcomes. Additionally, the findings demonstrated
that simultaneous presence of antibodies to cardiolipin and beta-2 glycoprotein I,
if lupus anticoagulant was not also present, did not predict adverse pregnancy
outcomes. Although an IgG anticardiolipin antibody level 40 units/mL or higher
was associated with adverse pregnancy outcomes, the presence of lupus
anticoagulant was a stronger predictor of adverse pregnancy outcomes, similar
to other clinical clotting events in general in antiphospholipid syndrome, unrelated
to pregnancy.
In bivariate analysis, adverse pregnancy outcomes occurred in 52% of patients
with and 13% without prior thrombosis (P = .00005) and in 23% with and 17%
without SLE (P = .52, not significant). In contrast, prior pregnancy loss did not
predict adverse pregnancy outcome (P = .96) nor did maternal race (white vs
non-white, P = .11).
Educational Objective: Recognize lupus anticoagulant positivity as the
antiphospholipid antibody test with the highest positive predictive value for
pregnancy loss in patients with antiphospholipid antibodies.

Case 10
A 15-year-old boy presents to the outpatient clinic 3 days after he presented to
the emergency department with fevers, rashes, and a swollen knee. The patient
has been having high-spiking fevers every night for the past 3 weeks
accompanied by “hives” over his extremities and abdomen. He has had
polyarticular arthralgia upon waking for the past 2 weeks but otherwise feels well
during the day. Two days before presentation to the emergency department, his
right knee became swollen and painful. The patient underwent arthrocentesis in
the emergency department and was sent home with a prescription for
indomethacin 3 times daily, which seems to have decreased the intensity of his
fevers and knee pain. He tells you that he is feeling well overall and is able to
attend school.
On physical examination, the patient has a macular rash on his arms and chest,
which demonstrates dermatographism and fades throughout the course of the
office visit. The heart and lung sounds are clear. He has mild cervical and
inguinal lymphadenopathy. His liver edge is palpable 1 cm below the right costal
margin. His right knee is mildly swollen and warm with a ballottable patella and
mild pain upon flexion. The rest of his joint examination is unremarkable. He is
able to ambulate without difficulty.
In the emergency department, the patient’s chest radiograph showed clear lungs
and normal heart size. Blood, urine, and synovial fluid cultures drawn at that time
were negative after 48 hours. Notable laboratory results are listed below (Table).

Table. Results of Laboratory Tests


Laboratory Test Result Reference Range

C-reactive protein 8.2 mg/dL 0.8 mg/dL or less

Erythrocyte sedimentation rate (Westergren) 57 mm/hr Male: 0-15 mm/hr

Ferritin, serum 390 ng/mL Male: 20-235 ng/mL


Hemoglobin, blood 12 g/dL Male: 14-18 g/dL

Leukocyte count, blood 16,300/µL 4000-11,000/µL

Leukocyte count, synovial fluid 10,200/µL Less than 200/µL

Platelet count 552,000/µL 150,000-300,000/µL

Click here for references

Which of the following is the best treatment for this patient?

A. Anakinra

B. Etanercept

C. Methotrexate

D. Methylprednisolone

Rationale

This patient has systemic onset juvenile idiopathic arthritis (sJIA). He has
arthritis of 1 joint and although systemic symptoms of fever and rash are present,
he is not severely ill. Interleukin (IL)-1 and IL-6 inhibitors are increasingly being
used early in the course of disease to spare patients the adverse effects of
glucocorticoids. According to the 2011 American College of Rheumatology
(ACR) treatment recommendations, patients with systemic JIA and fevers should
receive systemic glucocorticoids for initial treatment. However, the 2013 revision
of these guidelines recommends anakinra and nonsteroidal anti-inflammatory
drugs (NSAIDs) for initial treatment when the active joint count is less than 4 and
the physician global assessment score is less than 5. Given the patient’s current
state of health, anakinra would be the best choice at this time.
Etanercept would not be a correct choice for this patient at this time. Tumor
necrosis factor inhibitors have no role in the initial treatment of sJIA, although
they can be used in these patients for follow-up therapy after failing an IL-1
inhibitor and/or tocilizumab, an IL-6 inhibitor.
Methotrexate and leflunomide also have no role in the initial treatment of sJIA.
These medications can be used as adjunct therapies when patients continue to
have active arthritis after 1 month of anakinra therapy, but even then, switching
to another biologic is favored over starting methotrexate. In the 2013 ACR
guidelines, methotrexate is one of the valid options after 1 month of NSAIDs,
steroid monotherapy, or anakinra. However, as an initial therapy approach, it
does not work very quickly.
This child is ambulatory and attending school, and his joint manifestations are
minor. The 2013 ACR guidelines suggest that for certain patients, initiating
treatment with glucocorticoids may not be necessary. Using glucocorticoids in a
patient with sJIA is a common initial approach to treatment in children who are
severely affected. Intravenous glucocorticoid monotherapy is recommended for
initial treatment in patients who present with more severe disease: those with a
physician global assessment score of 5 or greater, irrespective of active joint
count, and those with a physician global assessment score of less than 5 with 4
or more active joints. Since physician global assessment scores are subjective,
some clinicians could argue that this patient should merit a physician global
assessment score of 5 or higher. However, given the patient’s current state of
health in which he feels well overall and has only minor findings on examination,
it is not necessary to start him on intravenous steroids, especially when a trial of
anakinra may be successful and therefore allow you to avoid steroids altogether.
Educational Objective: Understand the role of biologics in the treatment of
systemic juvenile idiopathic arthritis (sJIA).

Case 11
A 19-year-old woman presents to clinic with fatigue and persistent fevers for the
last 2 weeks. She has a known history of systemic juvenile idiopathic arthritis
(sJIA), diagnosed 4 years ago. At that time, she presented with symptoms of
quotidian fevers, sore throat, and arthritis. Laboratory studies at the time included
erythrocyte sedimentation rate (Westergren) of 100 mm/hr (reference range
[female]: 0-20 mm/hr), C-reactive protein level of 4.0 mg/dL (reference range: 0.8
mg/dL or less), and serum ferritin level of 1000 ng/mL (reference range [female]:
11-211 ng/mL).
She has been treated with methotrexate and prednisone since diagnosis.
Anakinra had also been added to her regimen, but this was stopped a year ago.
Before her recent presentation, she traveled to India 4 weeks previously and was
taking malaria prophylaxis. She has noticed some swollen lymph nodes in her
neck and a fullness in the right side of the abdomen along with easy bruising.
She mentions that she has missed 1 dose of methotrexate recently.
On examination today, her temperature is 101.5°F (38.6°C), heart rate is 120
bpm, blood pressure is 140/85 mm Hg, respiratory rate is 24 breaths per minute,
and oxygen saturation is 98% on room air. Palpable cervical lymphadenopathy is
present. The lungs are clear without rales. The cardiovascular examination
reveals tachycardia with regular rate and rhythm. The abdomen is soft, but
hepatomegaly is revealed by scratch testing. Scattered ecchymoses are present
on the arms and legs. Laboratory studies performed at this time show the
following results (Table).

Table. Results of Laboratory Tests


Laboratory Test Result Reference Range

Activated partial thromboplastin time 38 seconds 25–35 seconds

Aminotransferase, serum alanine (ALT, SGPT) 150 10–40 U/L

Aminotransferase, serum aspartate (AST, SGOT) 120 10–40 U/L

C-reactive protein 20.4 mg/dL 0.8 mg/dL or less

Creatinine, serum 1.1 mg/dL 0.7–1.5 mg/dL

Cultures, blood/urine Pending Negative

Erythrocyte sedimentation rate (Westergren) 8 mm/hr Female: 0–20 mm/hr

Ferritin, serum 4000 ng/mL Female: 11–211 ng/mL

Hemoglobin, blood 9.0 g/dL Female: 12–16 g/dL

Leukocyte count 7000/μL 4000–11,000/μL

Platelet count 150,000/μL 150,000–300,000/μL

Prothrombin time, plasma 15 seconds 11–13 seconds

Triglycerides, serum 350 mg/dL Less than 150 mg/dL

Urinalysis No protein, trace blood Negative

Click here for references

Which of the following conditions has most likely developed?

A. Exacerbation of sJIA

B. Leishmaniasis infection

C. Macrophage activation syndrome

D. Malaria

E. Methotrexate induced hepatitis

Rationale

Macrophage activation syndrome (MAS) is a secondary form of hemophagocytic


lymphohistiocytosis (HLH) triggered by infectious, oncologic or rheumatologic
conditions, including but not limited to sJIA, adult-onset Still’s disease, and
systemic lupus erythematosus. MAS is characterized by an exuberant immune
response characterized by T cell and macrophage activation with generation of
proinflammatory cytokines. It is estimated that the prevalence of MAS in patients
with sJIA is about 10%. No single clinical/laboratory feature is diagnostic of MAS.
. In this scenario, the absence of leukocytosis and thrombocytosis, the reduction
in ESR with increase in CRP (as can be seen with hypofibrinogenemia), the
coagulopathy, the elevated triglycerides and the degree of hyperferritinemia all
support a diagnosis of MAS rather than a flare of sJIA. Ferritin levels of 2000
mcg/L or greater have a sensitivity of 70% and a specificity of 68% for MAS. .
With a flare of sJIA, white cell counts and platelets typically increase rather than
decrease, and ESR and CRP tend to rise together.
In this patient, her travel history is concerning for a possible infectious process
(malaria or leishmaniasis) that may be associated with MAS. However, she was
taking malaria prophylaxis, and with infections in the absence of MAS, an
increase in both ESR and CRP level would be expected.
Hepatotoxicity related to methotrexate toxicity is usually an indolent process
characterized by mild transaminase elevations and is not associated with the
systemic symptoms described in this case.
Educational Objective: To recognize macrophage activation syndrome (MAS)
in a patient with systemic juvenile idiopathic arthritis (sJIA).
Case 12
A 70-year-old man with long-standing seropositive rheumatoid arthritis (RA)
presents for routine 3-month evaluation. His disease has been well controlled on
a stable dose of methotrexate 20 mg weekly and hydroxychloroquine 400 mg
daily, and he reports feeling well overall except for an unintentional 15-lb (6.8-kg)
weight loss and several nonhealing ulcers. He denies significant morning
stiffness, joint swelling, and pain.
The physical examination is significant for 2 skin ulcers with necrosis and sharply
demarcated erythematous borders in the preauricular area. Inguinal and left
axillary lymphadenopathy is also noted. He has 5 swollen and 7 tender joints on
examination. No nodules are noted. Plain radiographs of his hands and feet
show stable marginal erosions. An excisional lymph node biopsy shows a
polymorphic lymphoproliferative disorder.
Click here for references

Which of the following is the best next step in management?

A. Addition of infliximab

B. Addition of rituximab

C. Withdrawal of hydroxychloroquine

D. Withdrawal of methotrexate

Rationale

Lymphoproliferative disorders (LPDs) occur with increased frequency in patients


with RA. Lymphoma incidence increases as active RA persists and correlates
with the severity of disease activity. Rarely, LPD may develop in association with
long-term immunosuppressive therapy; they are usually of B-cell origin,
sometimes associated with latent Epstein Barr virus (EBV) infection, and include
polymorphic lymphoproliferative disorder, Hodgkin lymphoma, small B cell
lymphoma, and diffuse large B cell lymphoma. In fact, methotrexate (MTX)-
associated LPDs (MTX-LPD) are well described, and EBV is believed to play an
important pathogenic role. Under normal circumstances, EBV-specific cytotoxic
T lymphocytes act to suppress EBV-infected B cells. However, if the function of
EBV-specific cytotoxic T lymphocytes is impaired by immunosuppressants such
as methotrexate, EBV-infected B cells are reactivated to induce B cell
proliferation, leading to the development of lymphoproliferative disorder. LPD
with EBV-positive mucocutaneous ulcer (EBVMCU) has been reported as a
distinct disease entity with a self-limiting and indolent clinical course but can also
be seen in association with MTX-LPD or with age-related immunosenesce. The
pathogenesis of the ulceration is unclear, but some lesions have shown a
lymphocytic vasculitis. Like MTX-LPD alone, it has been shown to regress
spontaneously after MTX is withdrawn without initiation of chemotherapy.
Since MTX is associated with EBV-positive LPD with or without EBVMCU in
patients with RA, and withdrawal of MTX may be the only treatment needed, this
would be the safest choice listed until the evaluation is complete.
Withdrawal of hydroxychloroquine has not been associated with regression of
lymphoproliferative disease. Rituxmab can be used to treat lymphoma, but it is
administered as part of a combination chemotherapy regimen in most patients.
Infliximab would not be a good choice for a patient with a suspected hematologic
malignancy.
Educational Objective: Identify the treatment of Epstein Barr virus associated
lymphoma in a patient with rheumatoid arthritis.

Case 13
A 28-year-old man presents to the rheumatology clinic with a 4-week history of
pain and swelling in his knees and ankles. About 2 months ago, he developed
painless bilateral inguinal lymphadenopathy, along with low grade fever, sore
throat, and fatigue. He also recalls an unusual pink rash on his trunk, which did
not worsen with fever. When this rash began to improve, a new rash developed
on his hands and feet, affecting primarily the palms and soles. He saw a
dermatologist for this rash and underwent a skin biopsy, which revealed
perivascular mononuclear infiltrate with endarteritis. He was given a topical
steroid cream without improvement in the lesions. About 2 weeks later, he
noticed swelling in both knees and around his ankles, along with mild pain and
stiffness. He denies any change in his vision, difficulty breathing, urethral
discharge, and low back pain. His past medical history is remarkable for a history
of chlamydia treated 5 years ago. He denies any family history of rheumatic
disease. He drinks about 5 cans of beer a week and does not smoke. He denies
illicit drug use.
On physical examination, his temperature is 100.3°F (37.9°C), heart rate is 90
bpm, blood pressure is 130/85 mm Hg, and respiratory rate is 12 breaths per
minute. A few whitish-grey ulcers with surrounding erythema are present on the
tongue. No cervical lymphadenopathy is noted. His lungs are clear. The
cardiovascular examination reveals a regular rate and rhythm, with no murmurs
or rubs. Numerous 8-mm papular lesions are noted on the palms and soles,
without central clearing or bull’s eye lesions. A palpable, nontender inguinal
lymphadenopathy is noted. Both knees are without warmth, but moderate-sized
effusions are present. He has mild pain with passive range of motion of the joint.
Both ankles have small cool effusions and are mildly tender to manipulation of
the tibiotalar joint. Notable laboratory results are listed below (Table).

Table. Results of Laboratory Tests


Laboratory Test Result Reference Range

Antistreptolysin O titer 90 Todd units Less than 200 Todd units

C-reactive protein 3.5 mg/dL 0.8 mg/dL or less

Creatinine, serum 1.3 mg/dL 0.7–1.5 mg/dL

Erythrocyte sedimentation rate (Westergren) 90 mm/hr Male: 0–15 mm/hr

Ferritin, serum 350 ng/mL Male: 20–235 ng/mL

Hemoglobin, blood 10.9 g/dL Male: 14–18 g/dL

HIV RNA viral load, plasma Negative Less than 20 copies/mL

Human leukocyte antigen B27 Absent Negative

Rheumatoid factor (nephelometry) 11 IU/mL Less than 24 IU/mL

Urinalysis 1+ blood, 1+ protein Negative

Click here for references

Which of the following is the most likely diagnosis in this patient?

A. Adult-onset Still's disease

B. Reactive arthritis

C. Rheumatic fever

D. Secondary syphilis

Rationale

Syphilis is the great imitator. Although more published literature is available on


the arthritic manifestations of congenital (Clutton’s joints) and tertiary (tabetic
arthropathy) syphilis, the rheumatic manifestations of secondary syphilis,
although uncommon, should not be ignored. It is important to consider this
diagnosis in the young adult patient with fever, sore throat, rash, and arthritis. In
this patient, the palmoplantar lesions, oral lesions (known as mucous patches,
which are highly infectious), and symmetric arthritis limited to the lower
extremities should raise suspicion for syphilis. The palmoplantar rash in particular
can mimic keratoderma blennhorragica. Although a symmetric synovitis of the
knees and ankles is more common, shoulder and metacarpophalangeal and
proximal interphalangeal involvement may also occur and be confused with
rheumatoid arthritis. The histopathology of cutaneous secondary syphilis
characteristically shows a dense perivascular infiltrate with abundant plasma
cells and endothelial swelling. The most common renal manifestation of syphilis
is proteinuria secondary to membranous glomerulonephritis; however,
proliferative and crescentic glomerulonephritis have also been reported.
Still’s disease usually manifests with quotidian, high-spiking fevers. At the time of
a febrile episode, patients often describe the appearance of a salmon-colored
rash on the trunk and extremities. Although the rash of Still’s disease can rarely
affect the palms and soles, it would be expected to appear and disappear with
fever. Up to 70% of patients have marked elevations in serum ferritin level,
characteristically greater than 3000 ng/mL. This patient’s elevated ferritin level is
consistent with an acute phase reaction.
Rheumatic fever is certainly a consideration in any patient with fever, rash, and
arthritis but is generally uncommon in young adults and in developed countries.
Usually, patients develop symptoms about 3 weeks after sore throat symptoms.
Patients can develop a characteristic rash (erythema marginatum) and evidence
of valvular heart disease, which is not seen in the described patient. Oral lesions
are uncommon. Urinalysis can show pyuria, but frank blood and protein is
uncommon.
Reactive arthritis should always be in the differential diagnosis of a young patient
with rash and arthritis. Although both reactive arthritis and syphilis can cause a
palmoplantar rash, the oral lesions and lymphadenopathy in this case point
toward syphilis.
Most patients with reactive arthritis develop an asymmetric oligoarthritis
before developing skin lesions. The circinate balanitis and psoriasiform rashes
seen in reactive arthritis are distinct from this patient’s rash both clinically and
histologically. However, palmoplantar lesions mimicking keratoderma
blennhorragica have been reported in secondary syphilis. At least 50% of
patients with reactive arthritis carry the allele for human leukocyte antigen B27
(absent in this patient). Joint examination in reactive arthritis usually reveals
ballotable effusions, which are warm and quite painful with manipulation, unlike
the cool tender joints in this patient.
Educational Objective: Recognize that secondary syphilis can cause a
polyarthritis.

Case 14
A 17-year-old girl with a known history of juvenile arthritis, which is antinuclear
antibody positive, rheumatoid factor negative, and cyclic citrullinated peptide
antibody negative presents to clinic. She initially presented to medical care at 3
years of age with a swollen knee. Three months after her diagnosis at her first
ophthalmology examination, she was found to have right anterior uveitis. She
reports that her arthritis has been well controlled over the past many years but
that her uveitis remains an ongoing problem. She has been taking methotrexate
25 mg subcutaneously once weekly for the past 5 years and has been using
steroid drops in her right eye 3 times daily for the past year.
On examination, she has no signs of active arthritis in any joint. Her right pupil is
irregular and is poorly reactive to light. Fundoscopic examination of the right eye
reveals a large cataract and normal-appearing fundus and vessels. Complete
blood cell count, erythrocyte sedimentation rate, C-reactive protein level, and
comprehensive metabolic panel are normal. Her ophthalmologist reports that her
uveitis is still active.

Click here for references

What medication has the best evidence to support treatment of her arthritis and
uveitis?

A. Adalimumab


B. Etanercept

C. Leflunomide

D. Prednisone

Rationale

Uveitis is a common complication of juvenile idiopathic arthritis (JIA), especially


in patients with the oligoarticular subtype. Uveitis is very uncommon in patients
with systemic JIA and rheumatoid factor-positive polyarthritis. Risk factors for the
development of uveitis include young age, female gender, recent diagnosis, and
antinuclear antibody positivity. This patient has active uveitis not controlled on an
adequate dose of methotrexate and topical steroids. Given the amount of time
this patient has been on these medications, discussing tumor necrosis factor
(TNF) inhibition as a long-term treatment option is reasonable. Although no
standardized treatment regimens are available for treating JIA-associated uveitis,
TNF inhibitors are most commonly used after methotrexate failure. Infliximab and
adalimumab have been shown in a meta-analysis to be superior to etanercept in
the treatment of pediatric uveitis. Therefore, adalimumab is the correct answer,
and etanercept is incorrect.
Disease-modifying antirheumatic drugs (DMARDs) other than methotrexate, such
as cyclosporine, azathioprine, mycophenolate mofetil, and leflunomide, have
been reported as being effective in the treatment of JIA-associated uveitis.
However, methotrexate is used as the standard of care DMARD. Published
evidence-based treatment guidelines recommend escalating therapy to a TNF
inhibitor after methotrexate failure rather than using another DMARD such as
leflunomide.
The patient has cataracts in her right eye, likely an adverse effect from long-term
topical steroid use. Although oral prednisone may help to control her
inflammation initially, long-term use of steroids is discouraged due to side-effects.
Recommendations for ophthalmologic screening in children with JIA are available
from the American Academy of Pediatrics (Table).
Table. Frequency of Ophthalmologic Examination in Patients With JIA
Type ANA Age at Duration of Risk Eye Examination
Onset, y Disease, y Category Frequency, mo
Oligoarthritis or + ≤6 ≤4 High 3
polyarthritis + ≤6 >4 Moderate 6
+ ≤6 >7 Low 12
+ >6 ≤4 Moderate 6
+ >6 >4 Low 12
− ≤6 ≤4 Moderate 6
− ≤6 >4 Low 12
− >6 NA Low 12
Systemic disease (fever, NA NA NA Low 12
rash)

Educational Objective: Recognize the role of systemic therapy for treatment of


uveitis and juvenile idiopathic arthritis.

Case 15
A 27-year-old woman with a past medical history significant for systemic lupus
erythematosus (SLE) presents to a rheumatologist to establish
care. Manifestations of her SLE have included mucosal ulceration, malar rash,
hemolytic anemia, and discoid skin lesions. She has been well controlled on
hydroxychloroquine monotherapy for 18 months and mentions to you that she is
interested in pregnancy.
The physical examination reveals a well-nourished female in no acute distress;
cardiopulmonary and abdominal examinations are benign, but the skin
examination reveals atrophy and hyperpigmentation of her scalp without
erythema or ulceration. Her ophthalmologic evaluation several months ago did
not reveal evidence of hydroxychloroquine toxicity. Results of a recent urinalysis
are unremarkable, without evidence of pathologic casts, red cells, or proteinuria.
Results of laboratory studies are shown in below (Table).

Table. Results of Laboratory Tests


Laboratory Test Value Reference Range
Anti–double-stranded DNA 0 IU/mL 0–7 IU/mL
antibodies
Anticardiolipin antibodies
IgG Negative Greater than 20 GPL units is
positive
IgM Negative Greater than 40 MPL units is
positive
Anti–beta-2-glycoprotein
antibodies
IgG Negative Less than 21 SGU
IgM Negative Less than 21 SMU
Complement, serum
C3 123 mg/dL 100–233 mg/dL
C4 25 mg/dL 14–48 mg/dL
Creatinine, serum 0.9 mg/dL 0.7–1.5 mg/dL
C-reactive protein 0.4 mg/dL 0.8 mg/dL or less
Erythrocyte sedimentation rate 15 mm/hr Female: 0–20 mm/hr
(Westergren)
Platelet count 230,000/µL 150,000–300,000/µL
Russell viper venom time, dilute 37 seconds 33–44 seconds

Click here for references

Which of the following is the best management now?

A. Continue hydroxychloroquine without additional treatment

B. Continue hydroxychloroquine and add prednisone 10 mg daily

C. Discontinue hydroxychloroquine without replacement

D. Discontinue hydroxychloroquine and replace with prednisone 5 mg daily

E. Discontinue hydroxychloroquine and replace with azathioprine 50 mg


twice daily

Rationale

Hydroxychloroquine remains a mainstay in the management of patients with


SLE, and its benefits in terms of preventing morbidity and mortality associated
with SLE have been well established. Hydroxychloroquine use has been
associated with decreased placental transmission of anti-Ro antibodies and risk
of neonatal lupus and decreased incidence of disease activity flares in pregnant
patients with SLE. Hydroxychloroquine is recommended for all pregnant women
with SLE (unless otherwise contraindicated), as use of hydroxychloroquine in
pregnancy is associated with better clinical outcomes and is not associated with
an increase in congenital malformations or adverse events. Hydroxychloroquine
is also considered safe during lactation; levels of hydroxychloroquine in breast
milk are low.
Alternative therapies for immunosuppression (eg, azathioprine, prednisone) are
used in SLE during pregnancy when required but have a less favorable risk
profile than hydroxychloroquine. Data on azathioprine during pregnancy is drawn
largely from the literature on use of azathioprine and 6-mercaptopurine (6MP) in
women with inflammatory bowel disease and patients who have undergone
transplantation. Studies have shown the use of 6MP to be associated with
increased rates of congenital malformations. Data on azathioprine suggest it is
safer than 6MP, but the safety of azathioprine in pregnancy is not as well
established as it is for hydroxychloroquine. Use of glucocorticoids in the first
trimester is associated with increased risk of cleft lip and cleft palate. Use of
glucocorticoids during pregnancy can also be associated with problems during
pregnancy.
Adding prednisone would not be warranted for this patient with clinically
quiescent disease.
Educational Objective: Recognize the importance of continuing
hydroxychloroquine during pregnancy to prevent systemic lupus erythematosus
flares.

Case 16
A 75-year-old man presents for initial evaluation of myalgia and elevated serum
creatine kinase (CK) level in association with statin therapy for hyperlipidemia.
His statin was stopped approximately 3 months ago by his primary care provider
when increasing proximal muscle discomfort and persistently abnormal serum
CK level raised concern for possible medication toxicity. He denies associated
difficulty in breathing, rash or skin discoloration, and recent illness. Recently
performed colonoscopy and computed tomography (CT) scans of the chest,
abdomen, and pelvis are unremarkable.
The physical examination is remarkable for 3/5 muscle strength in proximal and
distal muscle groups in his upper and lower extremities bilaterally. The
cardiopulmonary evaluation and assessment for lymphadenopathy are
unremarkable, and a skin examination reveals no erythema, hyperpigmentation,
hyperkeratosis, ulceration, or vasculitic lesions.
Results of electromyography are consistent with an irritable myopathy, and
magnetic resonance imaging of his thighs revealed evidence of bilateral edema,
muscle atrophy, and fatty replacement. A quadriceps muscle biopsy shows
myofiber degeneration, necrosis, and regeneration, with minimal inflammatory
changes. Laboratory studies are shown below (Table).
Table. Results of Laboratory Tests
Laboratory Test Result Reference Range

Aldolase, serum 2.0 IU/mL 0.8–3.0 IU/mL

Antinuclear antibody 1:80 Greater than 1:40 is abnormal

Creatine kinase, serum 10,000 IU/L Male: 55–170 U/L

Creatinine, serum 0.8 mg/dL 0.7–1.5 mg/dL

Urine myoglobin Negative Negative

Click here for references

Which of the following antibodies is most likely to be positive in this patient?

A. Antiribonucloprotein (anti-RNP)

B. Anti-Jo1

C. Anti-3-hydroxy-3-methylglutaryl-coenzyme A reductase (anti-HMGCR)

D. Anti-Mi-2

E. Antinuclear matrix protein 2 (anti-NXP-2)

Rationale

With escalating use of HMGCR inhibition for management of atherosclerotic


cardiovascular disease and dyslipidemias, adverse effects associated with
therapy are increasingly being encountered. Complications of therapy can range
from muscle aches and pains to acute rhabdomyolysis and renal failure.
Although statin myopathy is generally self-limited and improves with removal of
the offending agent, statin use can also result in an immune-mediated
necrotizing myopathic process that requires systemic immunosuppression to
address. Features of this myopathy include a strong association with anti-
HMGCR antibodies and prominent necrosis on muscle biopsy without significant
levels of inflammation. Although this immune-mediated myopathy can occur
within months of beginning statin use, years of statin exposure before onset of
symptoms is more likely.
Anti-RNP antibodies are usually seen in patients with inflammatory myopathies
associated with systemic lupus and other connective tissue diseases, also known
as mixed connective tissue disease, but this patient has no other features of
these conditions. Anti-Jo1 antibodies are the most common of the antisynthetase
antibodies seen in inflammatory myopathies characterized by interstitial lung
disease, arthritis, dermatologic findings (mechanics hands), and pathologic
changes at the periphery of muscle fascicles and perimyseal connective tissue,
features not seen in this patient. Anti-Mi-2 antibodies are seen in patients with
dermatomyositis with predominant dermatologic and pulmonary involvement,
often with less severe muscular involvement (amyotrophic
dermatomyositis). Anti-NXP-2 antibodies are usually seen in patients with
cancer-associated dermatomyositis.
Educational Objective: To understand that patients with statin-associated
myopathy may have persistent muscle weakness associated with antibodies to
3-hydroxy-3-methylglutaryl-coenzyme A reductase (HMGCR).

Case 17
A 28-year-old man, who is a physical therapist, presents with a new diagnosis of
ankylosing spondylitis. Tumor necrosis factor inhibitors (TNFi) are recommended
for treatment. He is not currently taking any immunosuppressants. He was born
in the Philippines and reports receiving the bacillus Calmette–Guérin (BCG)
vaccine as a child. His pulmonary review of systems is negative. His lungs are
clear on examination, and a chest radiograph is negative. Testing for tuberculosis
(TB) using interferon gamma release assay (IGRA) indicates a strongly positive
result (5.8 IU/mL in the tuberculin antigen tube minus 0.7 IU/mL in the nil control
tube, for a tuberculin response of 5.1 IU/mL). The patient reports that he
underwent TB skin testing at his work site 1 day before IGRA testing.
Click here for references

The patient’s positive IGRA result is which of the following?

A. False positive because of recent tuberculin skin test


B. False positive because of past BCG exposure

C. Indeterminate-await results of recent tuberculin skin test

D. True positive-unaffected by past BCG exposure and recent tuberculin skin


test

Rationale

Unlike tuberculin skin testing (TST), serum IGRAs are not affected by previous
BCG vaccination and hence are of value in diagnosing TB in BCG-vaccinated
patients.
Serum IGRAs are in vitro enzyme-linked immunosorbent (Quantiferon-TB QFT
and Quantiferon-TB Gold QFT-G test) or immunospot (T-SPOT) assays requiring
a single peripheral blood draw. Whole blood or peripheral blood mononuclear
cells from the patient are incubated with tuberculin antigens. In patients
previously exposed to Mycobacterium tuberculosis, interferon gamma is
produced by T cells recognizing tuberculin antigens. These levels are compared
with those in a control “nil” tube and a positive mitogen control. IGRAs have high
sensitivity 76%/90% (pooled QFT/T-SPOT tests respectively) and specificity
98%/93% (pooled QFT/T-SPOT tests respectively) for detection ofM.
tuberculosis. TST has a sensitivity of 77% (pooled estimate) and a specificity of
97% in non–BCG-vaccinated populations and only 59% in BCG-vaccinated
populations. Therefore, in BCG-vaccinated patients, the risk of false positive is
substantially higher with TST (which can be positive with nontuberculous
mycobacterial exposure) than with IGRA.
Data are conflicting on whether IGRA improves sensitivity and specificity
compared with TST in non–BCG-vaccinated patients with rheumatic disease
before and during TNFi treatment.
Serial IGRA testing does not boost subsequent IGRA test results; therefore,
IGRAs can be used repeatedly in the same patient (eg, for serial screening after
possible exposure to TB or for yearly testing). TST before IGRAs should not
affect IGRA results if performed within the preceding 3 days; however, an interval
of 7 or more days between TST and IGRA can result in a boosting effect.
Like TST, IGRAs cannot differentiate between latent and active TB infection. This
distinction remains a clinical diagnosis based on history, physical examination,
imaging, and sputum testing if needed. Also similar to TST, indeterminate results
can occur in the setting of immunosuppression or anergy (which has been
demonstrated in patients with rheumatoid arthritis, end-stage renal disease on
hemodialysis, HIV infection, and systemic lupus erythematosus). Therefore,
patients with indeterminate IGRA results should undergo repeat testing and be
evaluated for immunosuppression and active tuberculosis. Weakly positive IGRA
responses (close to cutoff values) in low-risk patients should be evaluated on a
case-by-case basis and may represent test variance.
No established guidelines are available for the use of IGRAs in
immunosuppressed patients. Some authors suggest the use of both TST and
IGRA in such patients, with findings of both TST less than 5 mm and IGRA
negative identifying patients who may safely proceed to TNFi treatment.
In this case, the patient’s strongly positive IGRA is unaffected by his previous
BCG exposure, and recent skin testing was less than 3 days ago. Therefore, he
is unequivocally positive for TB, and evaluation and treatment should proceed
accordingly.
Educational Objective: Interpret results of testing for latent tuberculosis in
patients who have received the bacillus Calmette–Guérin (BCG) vaccine

Case 18
The patient is a 44-year-old man who has had severe polyarticular gout since he
was a teenager and subsequently developed multiple tophi. Treatment with
maximum doses of allopurinol and later febuxostat could not prevent
hyperuricemia, persistent polyarticular attacks, and worsening tophi. He
continues on colchicine 0.6 mg daily for prevention and requires long steroid
tapers for acute attacks. On examination, tophi are present on the 1st
metatarsophalangeal joints, olecranon bursa, and multiple joints of the hands,
bilaterally. Laboratory tests reveal a normal complete blood count and
comprehensive metabolic panel. His serum uric acid level is elevated at 10.0
mg/dL (reference range: 3.0–7.0 mg/dL).
He begins treatment with pegloticase 8 mg intravenously every 2 weeks. He is
compliant with monitoring, and his serum uric acid level after each of the first two
infusions (the day before the subsequent infusion) is less than 1.5 mg/dL.
However, one day before his third infusion, his serum uric acid level is elevated
to 9.4 mg/dL.
Click here for references

Given this information, which of the following changes is the first that you would
make to his treatment regimen?


A. Decrease pegloticase dose, add febuxostat

B. Discontinue pegloticase

C. Increase pegloticase dose, add allopurinol

D. Increase pegloticase frequency

Rationale

In this patient, the preinfusion hyperuricemia indicates the formation of high titer
antibodies to pegloticase . Pegloticase must be discontinued in this setting since
these patients have a higher risk of infusion reactions with subsequent
pegloticase infusions. The high titer antibodies are also associated with loss of
efficacy of the pegloticase.
Pegloticase should not be used in conjunction with other uric acid lowering
therapies as these may mask the recognition of the increase in serum uric acid
level associated with the development of antibodies to pegloticase. Adding
febuzostat or adding allopurinol would therefore not be advisable in this case.
Increasing the dose of pegloticase has not been shown to decrease serum uric
acid levels once antibodies have formed, and would not be expected to reduce
the risk of infusion reaction.
Pegloticase infusions are not given at closer intervals than every 2 weeks due to
its half-life of 14 days. Increasing the frequency of pegloticase infusions would
not be expected to elimiate the high risk of infusion reaction related to the
presence of high titer antibodies to pegloticase .
Educational Objective: Recognize results of screening tests indicating that
antibodies have formed to pegloticase and that pegloticase can no longer be
administered.

Case 19
A 28-year-old black man with recently diagnosed systemic lupus erythematosus
(SLE) with lupus nephritis presents for discussion of initiation of treatment. He
was diagnosed with SLE 2 months ago, with manifestations including discoid
lesions, leukopenia, and elevated serum creatinine level of 2.5 mg/dL (reference
range: 0.7–1.5 mg/dL). Tests for antinuclear, anti–double-stranded DNA, anti-
Sm, and anti-Ro antibodies are positive. A renal biopsy was performed showing
focal proliferative, sclerosing, and membranous lupus nephritis, class III A/C and
V. Immunofluorescence shows 3+ to 4+ staining for immunoglobulin (Ig) G, IgA,
IgM, and complement C3. C1q staining is negative.

Click here for references


In addition to high dose steroids, which of the following is the most appropriate
initial therapy?

A. Intravenous belimumab 10 mg/kg every 2 weeks for 4 weeks then monthly


doses thereafter

B. Intravenous cyclophosphamide 0.5 to 1.0 g/m2 once monthly for 6 months

C. Oral azathioprine 2.5 mg/kg once daily for 12 weeks

D. Oral mycophenolate mofetil twice daily, titrated to a target dose of 3 g daily

Rationale

Longitudinal race and ethnicity studies in patients with SLE have shown that
lupus nephritis occurs more frequently and with increased severity and greater
incidence of renal failure among Hispanic and black populations, with multiple
contributing factors to this discrepancy, including genetic and socioeconomic
factors. The Aspreva Lupus Management Study (ALMS) examined the influence
of race and ethnicity on response to lupus nephritis treatment, and the findings
suggest that ethnicity may influence treatment response in lupus nephritis. Black
and Hispanic groups in the study showed a greater number of responders to
mycophenolate mofetil compared with intravenous cyclophosphamide.
Additionally, black patients receiving intravenous cyclophosphamide were more
likely to withdraw prematurely from the study due to adverse effects. The study
findings suggest that racial and ethnic variations may affect response to
established lupus nephritis treatments.
Although studies are ongoing, belimumab has not yet been shown to be
efficacious in the treatment of patients with active lupus nephritis. Azathioprine is
used as maintenance treatment for lupus nephritis but has been shown to be less
efficacious than mycophenolate mofetil or cyclophosphamide as induction
therapy for newly diagnosed lupus nephritis.
Educational Objective: Appreciate possible ethnic differences in response to
treatments for glomerulonephritis in systemic lupus erythematosus

Case 20
A 57-year-old man presented several months ago describing years of
progressive neck stiffness and pain, along with less severe middle and lower
back pain, which had worsened to the point where he had difficulty bending over
to pick things up off the floor. His maternal uncle has ankylosing spondylitis (AS).
On examination, he was noted to have marked reduction in flexion, extension,
and lateral flexion of his neck and flexion of his lower back. No enthesitis was
noted. A test for human leukocyte antigen B27 was positive, and routine
anteroposterior pelvic radiographs suggested sacroiliac fusion. Radiographs of
the lumbar spine and a dedicated view of the sacroiliac joints are shown below.

Figure. Radiographs of the Spine and Dedicated Sacroiliac Views


Click here for references

Which of the following is a potential radiologic feature of his disease process?

A. Erosions or obliteration of the sacroiliac joints

B. Sacroiliac capsular bridging and axial osteophytes

C. Squaring of the corners of the vertebral bodies


D. Syndesmophytes along the lumbar spine

Rationale

This patient has diffuse idiopathic skeletal hyperostosis (DISH), which like AS
can present with increased pain and decreased mobility of the spine. But AS is
usually diagnosed much earlier in the third or fourth decade and can be
differentiated from DISH radiographically. Although patients with AS develop
erosions and eventually obliteration of the sacroiliac (SI) joints, in those with
DISH the SI joints can develop degenerative changes.
SI capsular bridging has been described in patients with DISH, which on the
pelvic anteroposterior radiograph may give the false appearance of obliteration of
the SI joint space (though not seen in the image above). In these cases,
dedicated SI radiographs or computed tomography shows intact SI joint spaces
and at times presence of anterior capsular bridging due to capsular ossification
(but the patients will not respond to tumor necrosis factor inhibitors like
etanercept). Both groups show osteophyte development, but as expected,
syndesmophytes are more frequent in patients with AS, whereas those with
DISH have more degenerative bone spurs.
Erosions or fusion of the SI joints would be seen in AS and not in DISH.
Squaring of the corners of the vertebral bodies would be seen in AS, not
DISH The anterior borders of vertebrae may appear straight or “squared” due to
periosteal proliferation of new bone filling in the normal concavity or erosion at
the anterosuperior and anteroinferior vertebral margins.
Syndesmophytes along the lumbar spine would be seen in AS, not DISH. The
hallmark of spinal disease in AS is the development of these characteristic bony
spurs. These start as thin, vertically-oriented projections of bone that develop
due to ossification within the outer fibres of the annulus fibrosus of the
intervertebral disc. Syndesmophytes are radiographically visible on the anterior
and lateral aspects of the spine starting from the corner of the vertebra.
Educational Objective: Recognize the clinical and radiographic differences
between spondyloarthropathies and diffuse idiopathic skeletal hyperostosis.

Case 21
A 50-year-old woman with no past medical history presents with a 12-week
history of fatigue, arthralgias, myalgia, and progressive shortness of breath. The
symptoms had an insidious onset but have been occurring daily. She has lost 15
lb (6.8 kg) in the last 12 weeks. The shortness of breath is progressive and is
now limiting her activities of daily living. She has a mild dry cough and has
coughed up blood-tinged sputum 2 to 3 times. She has no associated skin rash,
skin tightness, chest pain, or other symptoms. She is a previous 30 pack-year
smoker and quit 6 months ago.
On examination, she is afebrile with a temperature of 98.6°F (37°C). Her heart
rate is 108 bpm, blood pressure is 132/78 mm Hg, weight is 134 lb (74 kg), and
body mass index is 23.6 kg/m2. The systemic examination is unremarkable. The
heart examination reveals normal heart sounds without any murmur, rubs, or
gallops. The pulmonary examination reveals diffuse crackles, most prominent in
the lower lung zones. There are no rashes. On musculoskeletal examination,
mild tenderness is noted in multiple metacarpophalangeal joints, wrists, and
elbows, with preserved range of motion. The neurologic examination is afocal.
Results of laboratory tests are listed below (Table). A chest radiograph reveals
diffuse interstitial infiltrates.

Table. Results of Laboratory Tests


Laboratory Test Result Reference Range

Antineutrophil cytoplasmic antibody, cytoplasmic (C-ANCA) Negative Negative

Antineutrophil cytoplasmic antibody, perinuclear (P-ANCA) / 1:320


Negative
antimyeloperoxidase antibody IU/L

C-reactive protein 5.2 mg/dL 0.8 mg/dL or less

Creatinine, serum 2.8 mg/dL 0.7–1.5 mg/dL

Female: 0–20
Erythrocyte sedimentation rate (Westergren) 76 mm/hr
mm/hr

Female: 37%–
Hematocrit, blood 31 %
47%

Female: 12–16
Hemoglobin, blood 10.7 g/dL
g/dL

Protein-to-creatinine ratio, urine 2 gm Less than 0.2

Urinalysis 5 RBC/hpf 4 RBC/hpf

Click here for references

Which of the following is the most likely cause of her pulmonary symptoms?

A. Atypical pneumonia


B. Chronic obstructive pulmonary disease

C. Pulmonary fibrosis

D. Pulmonary hypertension

Rationale

The presentation in this case is of interstitial fibrosis in the setting of pulmonary


renal syndrome suggestive of microscopic polyangiitis (MPA). This is a less
common but frequent presentation of lung involvement of MPA. The presentation
and work-up is consistent with an inflammatory autoimmune pulmonary renal
syndrome most suggestive of MPA. She has progressive shortness of breath
with crackles on pulmonary examination and an abnormal chest radiograph
suggestive of interstitial fibrosis. Pulmonary fibrosis can be seen in MPA and may
be the presenting symptom. In two different cohorts, one from Europe and one
from China, pulmonary fibrosis was observed in about 30% of patients with MPA;
half of these cases were present at initial MPA diagnosis. In both cohorts,
patients with MPA who also presented with pulmonary fibrosis seemed to have
positive P-ANCA/myeloperoxidase antibodies and had worse prognosis than
those without the manifestation. The most common computed tomography (CT)
pattern was that of usual interstitial pneumonia. In the cohort from China, these
patients were more likely to be older and female and have extrapulmonic
involvement. This patient should undergo further evaluation of her pulmonary
symptoms with high-resolution CT scanning and pulmonary function testing and
also be evaluated for renal involvement, which seems to be active
glomerulonephritis. She should also be started on aggressive therapy with
corticosteroids, and additional therapy with cyclophosphamide or rituximab
should be considered.
This patient has no obvious signs of an infection or history of fever and is
afebrile. The protracted duration of the symptoms makes infectious causes less
likely. Moreover, her findings are suggestive of MPA, not infection. Nonetheless,
the infectious causes need to be worked up.
She has no known history of chronic obstructive pulmonary disease (COPD),
and physical examination, laboratory, and radiography findings are not
consistent with COPD.
She has progressive shortness of breath but normal heart sounds with lack of
loud P2. Pulmonary hypertension is rare in MPA, and the physical findings and
radiographs are suggestive of an interstitial process. Undergoing an
echocardiogram would be appropriate, but the suspicion for pulmonary
hypertension is low in this clinical setting.
Educational Objective: To recognize pulmonary fibrosis as a significant lung
manifestation associated with microscopic polyangiitis.

Case 22
A 40-year-old woman presents with gradual onset of weakness in her arms and
legs over the past 3 months. She also complains of anorexia, fatigue, shortness
of breath, and arthralgias in both hands. She denies any recent signs or
symptoms of infection and other illnesses. She has a history of hypertension and
is taking hydrochlorothiazide.
Her physical examination is notable for difficulty raising the arm above the head
and minimal pain when rising from a seated position. A scaly, violaceous rash is
present over the extensor surface of fingers. Results of laboratory tests are listed
below (Table).

Table. Results of Laboratory Tests


Laboratory Test Result Reference Range

Aldolase, serum 10 IU/mL 0.8–3.0 IU/mL

Anti-Jo-1 antibody Positive Negative

Antinuclear antibodies 1:640 Greater than 1:40 is abnormal

Creatine kinase (total), serum 1016 U/L Female: 30–135 U/L

Click here for references

Which of the following findings on electromyography is most likely to be present?

A. Increased duration, large amplitude, polyphasic motor unit action


potentials with spontaneous fibrillations, positive sharp waves, and
fasciculations

B. Marked variation in amplitude of motor unit action potentials with


consecutive discharges

C. Short duration, small amplitude, polyphasic motor unit action potentials


with fibrillations, positive sharp waves, insertional irritability, and complex
repetitive discharges

D. Spontaneous independent repetitive potentials that gradually wax and


wane in amplitude and frequency over a few seconds

Rationale

Electromyography (EMG) is a helpful diagnostic tool in the evaluation of


weakness. EMG provides evidence for myopathies and neuromuscular
conditions that can lead to weakness. EMG measures electrical activity arising
within a muscle, specifically a motor unit action potential (MUAP). An MUAP is
the summation of electrical activity generated by all the individual muscle fibers
innervated by a single motor nerve fiber and is characterized by 3 parameters:
duration, amplitude, and number of phases. Duration is affected by size of
muscle fibers, number of muscle fibers, spatial distance of the motor unit’s
terminal axons in the endplate zone, and conduction velocity in the terminal axon
and muscle fibers. Amplitude is related to the diameter of the muscle fiber and
density of muscle fibers as this affects the ability of the EMG electrode to
generate a spike in the waveform. The number of phases is measured by the
number of times the waveforms cross the baseline. A normal MUAP has less
than 4 phases. An MUAP is called polyphasic if more than 4 phases are present.
In myopathic disorders, loss of muscle fibers occurs from within the motor unit,
but the total number of motor units is unchanged. Therefore, myopathic MUAP is
characterized by short duration, small amplitude, and polyphasic motor unit
action potentials, which reflect the smaller motor unit. This patient has
inflammatory dermatomyositis, which is an inflammatory myopathy. Presence of
active inflammation and/or necrosis of the muscle fiber also lead to a denervation
pattern because the muscle fibers are not in contact with terminal axons. The
muscle fibers can therefore fire spontaneous discharges measured as
fibrillations, positive short waves, and insertional irritability. In metabolic
myopathy, spontaneous activity is rare. Complex repetitive discharges are
another spontaneous activity of grouped fibrillations and/or positive short waves.
These complex repetitive discharges start and stop abruptly and can last for
several minutes. Complex repetitive discharges are believed to be formed by
muscle fibers activating one another to fire. Therefore, the EMG triad for an
inflammatory myopathy is 1) short duration, small amplitude, polyphasic motor
unit action potential, 2) spontaneous fibrillations, positive sharp waves, and
insertional irritability, and 3) complex repetitive discharges.
The EMG findings of increased duration, large amplitude, polyphasic motor unit
action potentials with spontaneous fibrillations, positive sharp waves, and
fasciculations are most consistent for amyotrophic lateral sclerosis (ALS). ALS is
a progressive neurodegenerative disease affecting upper motor neurons and
lower motor neurons. In the clinical case above, the patient does not
demonstrate any clinical findings of upper motor neuron loss such as spasticity
and hyperreflexia. In ALS, distal weakness typically presents initially in
comparison to inflammatory myopathies characterized by proximal weakness.
EMG in ALS demonstrates active and chronic loss of lower motor neurons
respectively leading to ongoing denervation and compensatory reinnervation by
collateral axon branches, which results in decreased number of motor units but
increased number of muscle fibers within a motor unit. The increased muscle
fibers within a motor unit results in increased duration, large amplitude, and
polyphasic motor unit action potentials. The active loss of lower motor neurons
results in an acute denervation pattern demonstrated by spontaneous fibrillations
and positive sharp waves. In contrast to insertional irritability noted in
inflammatory myopathies, ALS demonstrates spontaneous and involuntary
discharge of a single motor unit resulting in fasciculations.
The EMG findings of marked variation in amplitude of motor unit action potentials
with consecutive discharges are most consistent for myasthenia
gravis. Myasthenia gravis is a disorder of neuromuscular transmission at the
postsynaptic neuromuscular juncture. Clinical presentation is typically fluctuating
muscle weakness and fatigability. EMG examinations in patients with myasthenia
gravis typically demonstrate variable motor unit action potentials because of
intermittent failure of transmission at the neuromuscular juncture.
The EMG findings of spontaneous independent repetitive potentials that
gradually wax and wane in amplitude and frequency over a few seconds are
most consistent for myotonic dystrophies. Myotonic dystrophies are autosomal
dominant neurologic disorder characterized by skeletal weakness and myotonia,
which is delayed relaxation following normal muscle contraction. In myotonic
dystrophies, EMG will detect myotonic discharges that are fired from single
muscle fibers.
Educational Objective: To recognize the importance of electromyography in
differentiating a primary myopathic process from other causes of muscle
weakness

Case 23
A 50-year-old woman presents for ongoing evaluation and treatment of
seropositive rheumatoid arthritis (RA). Six months ago, she was started on
leflunomide at 20 mg daily. This medication has resulted in symptomatic
improvement of her pain and stiffness. She remains on a low dose of prednisone
5 mg daily. Her past medical history includes hypothyroidism, for which she takes
thyroid hormone replacement . She has noted an unintentional 35-lb (16-kg)
weight loss in the past 6 months. She denies diarrhea, abdominal pain, vomiting,
or other gastrointestinal symptoms. She has no lymphadenopathy or night
sweats. She feels that her appetite is unchanged. Her internist was concerned
about the weight loss and performed laboratory studies as noted below (Table).
She also underwent colonoscopy, mammography, and a computed tomography
scan of the chest, abdomen, and pelvis. These studies were normal.

Table. Results of Laboratory Tests

Laboratory Test Result Reference Range

Albumin, serum 3.7 g/dL 3.5–5.5 g/dL

Alkaline phosphatase, serum 98 U/L 30–120 U/L

Aminotransferase, serum alanine 35 U/L 10–40 U/L


(ALT, SGPT)

Aminotransferase, serum aspartate 35 U/L 10–40 U/L


(AST, SGOT)

Calcium, serum 9.3 mg/dL 8.6–10.2 mg/dL

Creatinine, serum 1.0 mg/dL 0.7–1.5 mg/dL

Erythrocyte sedimentation rate 25 mm/hr Female: 0–20 mm/hr


(Westergren)

Glucose, plasma 110 mg/dL 70–99 mg/dL


Normal (fasting)

Hemoglobin, blood 12.5 g/dL Female: 12–16 g/dL

Lactate dehydrogenase 95 U/L 60-100 U/L

Leukocyte count 6500/µL 4000-11,000/µL

Platelet count 210,000/µL 150,000-300,000/µL


Thyroid-stimulating hormone (TSH), 0.5 µU/mL 0.5–4.0 µU/mL
serum

Her examination reveals a well-developed, well-nourished female in no acute


distress. Her blood pressure is 124/78 mm Hg, heart rate is 76 bpm, weight is
152 lb (69 kg) but was 187 lb (85 kg) 6 months ago, and height is 66 in (168 cm).
She has no lymphadenopathy. The musculoskeletal examination reveals
minimal synovitis and tenderness at both 2nd metacarpophalangeal joints.

Click here for references

Which of the following is the likely cause of this patient's 35-lb (16-kg) weight loss?

A. Lymphoma

B. Excess thyroid hormone replacement

C. Leflunomide-associated weight loss

D. Steroid-induced diabetes mellitus

Rationale

The finding of unintentional and significant weight loss is always of clinical


concern. In this patient, an extensive evaluation was performed to determine the
etiology of the 35-lb (16-kg) weight loss. No evidence of an occult malignancy,
hyperthyroidism, or any other readily explainable cause for the weight loss was
found.
A number of reports have implicated leflunomide therapy as a cause of
significant weight loss. This adverse effect may occur even though the patient
has responded clinically to leflunomide and may necessitate cessation of the
drug. The mechanism of action by which this occurs is unclear; however, it is
thought that it may be related to interference with oxidative phosphorylation and
ATP generation in mitochondria.
The other answer choices would be unlikely. The patient has no systemic
symptoms of lymphoma and no detectable lymphadenopathy on examination or
imaging. Furthermore, she has normal cell lines and a normal lactate
dehydrogenase level. Her erythrocyte sedimentation rate is only slightly raised.
These all make a suspicion of lymphoma less likely. Hyperthyroidism could
certainly be a potential cause of significant weight loss; however, the TSH level
was normal. Although the patient’s fasting glucose level is slightly elevated, it
would be unlikely to lead to significant weight loss in the absence of polyuria or
polydipsia.
Educational Objective: Recognize that leflunomide can cause significant weight
loss in a patient with rheumatoid arthritis, which may necessitate discontinuation
of therapy.

Case 24
A 38-year-old man with psoriatic arthritis is interested in starting apremilast
therapy. He has been taking oral methotrexate 20 mg weekly for 6 months
without significant relief of his joint pain, although his psoriasis has improved. On
initial physical examination, his blood pressure was 136/82 mm Hg, heart rate
was 72 bpm, height was 68 in (172.7 cm), weight was 138 lb (62.6 kg), and body
mass index was 21 kg/m2. He was in no acute distress. He had psoriatic plaques
on the extensor surfaces of both elbows. He had moderate tenderness and
swelling of his right 2nd, 3rd, and 4th proximal interphalangeal (PIP) joints and
his left knee. His right Achilles tendon was tender and swollen, and his left 3rd
and 4th toe showed dactylitis. Results of his laboratory tests, including a purified
protein derivative test, were within normal limits. Apremilast treatment was
initiated and methotrexate was discontinued.
He returns to his rheumatologist after taking aprelimast 30 mg twice daily for the
past 4 months. He thinks his joint pain and psoriasis have improved significantly.
He feels well and denies any symptoms of upper respiratory infections, fevers,
diarrhea, or depression. He has recently traveled to India but denies any other
recent travel history. On physical examination his blood pressure is 132/84 mm
Hg, heart rate is 68 bpm, weight is 122 lb (55.3 kg), and body mass index is 18.5
kg/m2. He has new temporal wasting. He has no active psoriatic plaques. He also
has mild tenderness and swelling of his right 2nd PIP joint. His left 3rd toe shows
dactylitis. His current medications include apremilast 30 mg twice daily,
methotrexate 20 mg weekly, and folic acid 1 mg daily.
Click here for references

Which of the following is the most appropriate next step in management?


A. Begin treatment for Mycobacterium tuberculosis

B. Discontinue treatment with apremilast

C. Perform colonoscopy

D. Send a stool specimen for fecal leukocyte analysis

Rationale

Apremilast is known to cause weight loss. This has also been reported with other
phosphodiesterase 4 inhibitors and may be a class effect. In the initial trials for
apremilast (PALACE-1, -2, and -3), 57% of patients experienced weight loss,
10% reported weight loss of 5-10% (compared with 3.3% of patients receiving
placebo) and 1.8% of patients reported weight loss as an adverse event. Weight
loss with apremilast is not attributable to nausea or diarrhea. Weight loss with
apremilast occurs without any overt symptoms, such as in the patient
described. It is recommended that patients receiving apremilast should have
weight monitored regularly, and if unexplained or clinically significant weight loss
occurs, discontinuation of apremilast should be considered.
The use of apremilast has not resulted in a significant increase in reactivation
tuberculosis. In fact, in the majority of phase 1, 2, and 3 trials for apremilast,
patients were not screened for latent tuberculosis before initiation of the
medication. Unexplained weight loss can always be indicative of malignancy, but
as the patient feels well, has no fevers, chills, or night sweats and no organ-
specific manifestations to suggest a malignancy, this is less likely to be correct.
The patient has traveled to India, but as he has no gastrointestinal symptoms,
fecal leukocyte analysis is likely to be of low yield.
Educational Objective: Recognize weight loss as a side effect of a novel
treatment for psoriatic arthritis, apremilast.

CASE 25

Which of the following receptors is activated by endogenous molecules released by


necrotic cells, triggering inflammation?

A. CTLA-4 receptor

B. Fas receptor

C. NOD-like receptor

D. PD-1 receptor

Rationale

Stimulation of the innate immune system by pattern recognition receptors (PRR)


is the first line of defense against microbial pathogens, promoting pathways that
increase microbial killing and T and B cell activation. The PRR family consists of
Toll-like receptors, RIG-I-like receptors, AIM2-like receptors, C-type lectin
receptors, and the nucleotide-binding domain, leucine-rich repeat (LRR)-
containing (or NOD-like) receptors. During infection, PRRs recognize
pathogen-associated molecular patterns (PAMPs), components common to
microbes but not mammalian cells (eg, lipopolysaccharides). Endogenous host
molecules may also activate PRRs when they are detected by the immune
system outside of their usual physiologic context. These molecules are referred
to as damage-associated molecular patterns (DAMPs). Under normal physiologic
conditions, DAMPs perform noninflammatory functions in living cells. However,
DAMPs released from their normal physiologic actions may be recognized by
pattern-recognition receptors, initiating and propagating an inflammatory
response. DAMPs can be released from cells when loss of plasma membrane
integrity occurs, such as with cell necrosis or necroptosis (regulated cell death
without apoptotic markers). Examples of potential DAMPs released from necrotic
cells include high-mobility group protein B1 (HMGB-1), double-stranded DNA,
S100, and serum amyloid A. Uric acid crystals released into the joint also act as
a DAMP, stimulating a gout flare by triggering interleukin 1 (IL-1) release.
The other receptors listed, including the Fas receptor, programmed cell death-1
(PD-1) receptor, and cytotoxic T-lymphocyte-associated protein 4 (CTLA-4)
receptor, are not part of the PRR family involved in recognizing DAMPs.
The CTLA-4 receptor interacts with CD80/CD86 to prevent T cell
activation. CTLA-4 antagonizes the activity of CD28, which also binds to
CD80/CD86 to promote T cell activation. Abatacept is a fusion protein of CTLA-4
and immunoglobulin G that blocks T cell activation by preventing CD80/CD86
from interacting with CD28. The Fas receptor recognizes Fas ligand (FasL), a
member of the tumor necrosis factor superfamily. Fas activation typically results
in intracellular signaling pathways leading to apoptosis (programmed cell death).
Recent studies suggest that the Fas receptor can activate nonapoptotic
pathways promoting cellular proliferation and a proinflammatory response
leading to disease states such as cancers and rheumatoid arthritis.
The PD-1 receptor recognizes two different ligands, the PD ligand-1 (PD-L1) and
PD ligand-2 (PD-L2). Activation of the PD-1 pathway promotes apoptosis in
activated T cells. The PD-1 pathway plays a role in infection, autoimmunity, and
tumor immunity by affecting survival and function of activated T cells. Clinical
trials are currently underway blocking the PD-L1/PD-1 pathway to promote
antitumor immunity.
Educational Objective: Understand how damage-associated molecular patterns
(DAMPs) contribute to inflammation.

Case 26
A 60-year-old man presents with 1 week of sharp, burning pain in the right groin
and thigh. For the past 2 days he has also had weakness in the right leg.
Symptoms were not exacerbated by cough or valsalva maneuver. He has no
history of trauma, recent illness, or associated fever. His past medical history is
notable for diet controlled type 2 diabetes mellitus with a recent hemoglobin A1c
level of 6.7% (reference range: 4.0%–6.1%) after an intentional 20-lb (9-kg)
weight loss achieved with exercise and dietary changes. His social history is
negative for recreational drug use, and his family history is negative for similar
conditions.
The physical examination reveals normal vital signs and prominent weakness of
the iliopsoas, quadriceps, and gastrocnemius muscles of the right leg. Allodynia
and decreased pin-prick sensation are present over the anteromedial part of the
right calf. The right knee deep tendon reflex is absent. The cranial nerves and
upper extremity nervous system examination is normal. The remainder of the
examination, including the sinuses, lymph nodes, lung, cardiovascular system,
abdomen, and joints, is also normal.
The complete blood count with differential, serum creatinine level, liver enzymes,
urinalysis, and erythrocyte sedimentation rate are normal, as are Lyme disease
serology, serum creatine kinase level, serum protein electrophoresis, antinuclear
antibodies, and rheumatoid factor.
Magnetic resonance imaging (MRI) of the lower spine and pelvis is normal.
Electromyography (EMG)/nerve conduction velocity (NCV) 3 weeks after
symptom onset reveals abnormalities in the distribution of 2 nerve roots and 2
peripheral nerves and paraspinal denervation. No absent F waves, conduction
blocks, or abnormal temporal dispersion are noted in any motor nerves.
Click here for references

Which of the following is the most likely cause of the patient’s symptoms?

A. Chronic inflammatory demyelinating polyradiculoneuropathy

B. Complex regional pain syndrome

C. Lumbosacral radiculoplexus neuropathy

D. Lumbar radiculopathy

Rationale
The lumbrosacral plexus is comprised of an upper lumbar plexus and lower lumbosacral plexus. The
ventral rami of the T12-L4 nerve roots form the upper portion. The ventral rami of the L4-S4 nerve roots
contribute to the lower portion. The lumbrosacral plexus is situated within the psoas major muscle and
provides motor and sensory function to the ipsilateral lower limb and pelvic girdle (Table and Figure).
Table. Lumbosacral Plexus Nerves and Functions

Figure: Diagramatic Illustration of the Lumbosacral Plexus


Many different disease processes can cause lumbosacral plexopathy. Localized processes typically impact
individual portions of the plexus. Tumors, infection/psoas abscess, high impact trauma (pelvic fractures),
radiation, and hematomas are examples of lesions capable of causing localized lumbosacral plexopathy.
Inflammatory and infiltrative causes tend to also involve nerve roots and the nerve itself in addition to the
plexus and are termed lumbosacral radiculoplexus neuropathies. Diabetes mellitus is the most common
inflammatory cause of lumbosacral radiculoplexus neuropathies. Unlike distal diabetic sensory neuropathy,
diabetic lumbosacral plexopathy is caused by inflammation and miscrovasculitis. It typically occurs in
patients with well-controlled type 2 diabetes mellitus and often after weight loss.
Lumbosacral plexopathy and lumbosacral radiculoplexus neuropathies typically cause the acute onset of
leg pain followed by weakness. EMG reveals a patchy pattern of denervation in the distribution of part or
all of the lumbosacral plexus. Fibrillation potentials are present in the paraspinals in radiculoplexus
neuropathies but absent in pure lumbosacral plexopathies. Spinal fluid studies are usually normal except for
nonspecific increased protein concentration. Little evidence is available to guide therapy of inflammatory
lumbosacral plexopathies and diabeticradiculoplexus neuropathies, but immunosuppressive therapies are
often used. Recovery is variable and may be incomplete.
Patients with chronic inflammatory demyelinating polyradiculoneuropathy (CIDP) would be expected to
have symmetrical weakness for at least 6 weeks. The absence of features consistent with demyelination
also argues against CIDP being the correct diagnosis.
Although complex regional pain syndrome (CPRS) can cause a burning pain and allodynia, CPRS would
not cause weakness or the electrophysiologic findings in this case. Additionally, a history of trauma or an
inciting event is absent as are cardinal features of CPRS such as swelling and color change.

The negative MRI of the spine and diffuse abnormalities on EMG/NCV


are inconsistent with a lumbar radiculopathy.

Educational Objective: Recognize lumbosacral plexopathy and lumbosacral radiculoplexus


neuropathy as causes of pain and weakness involving the lower extremities

Case 27
An 82-year-old woman with a history of hypertension and recent deep venous
thrombosis presents with right shoulder pain, limited range of motion, and a large
joint effusion after a brief hospital admission for dehydration. She is a retired
nurse who exercises regularly with daily 2-mile walks and upper-extremity light
weight lifting. Plain radiographs of the shoulder show no fracture or dislocation
but reveal prominent narrowing of the right glenohumeral joint, sclerosis, and
osteophytes of the humeral head. Given the large effusion, she was referred to
rheumatology for shoulder arthrocentesis.
She currently takes hydrochlorothiazide 25 mg daily and warfarin 5 mg daily.
Laboratory examination performed earlier in the morning shows a normal blood
count, preserved renal function, and a therapeutic international normalized ratio
(INR) at 2.8.
Click here for references

Which of the following is the best course of action at this time?

A. Proceed with arthrocentesis immediately

B. Admit for fresh frozen plasma before arthrocentesis

C. Replace warfarin with clopidogrel for 72 hours before arthrocentesis


D. Replace warfarin with low molecular weight heparin for 72 hours,
withholding heparin on day of arthrocentesis

E. Withhold warfarin for 24 hours and proceed with elective arthrocentesis

Rationale

Several large and small studies, prospective and retrospective, indicate that joint
and soft tissue aspirations/injections are safe in patients prescribed warfarin with
a therapeutic INR. A large retrospective review of 640 arthrocentesis and joint
injection procedures performed in more than 500 patients on anticoagulation
therapy assessed the incidence of early and late clinically significant bleeding in
or around a joint, infection, and procedure-related pain. No statistically significant
difference in early and late complications was found between patients with an
INR 2.0 or higher and those with INR less than 2.0.
The safety of joint and soft tissue aspirations and injections was also evaluated in
a few small prospective studies either through a standardized interview
(ascertained by patient-reported increases in swelling, bruising or warmth at the
procedure site) or physical examination a few days after the procedure was
performed. The incidence of bleeding complications was very low and not
statistically significantly different between patients on warfarin at a therapeutic
level and those without anticoagulation.
Given the documented safety of joint aspiration on patients treated with
anticoagulants with a therapeutic INR, there is no reason to refuse to aspirate a
joint when medically indicated. Similarly, giving fresh frozen plasma or reducing
the level of anticoagulation before joint aspiration does not seem to be necessary
in these patients.
Educational Objective: Review risks of joint injections in patients on
anticoagulation therapy.

Case 28
A 75-year-old man with past medical history of diabetes, gout, and
hyperlipidemia presents for evaluation of frequent falls in the last few months. He
was very active and was an avid gardener until a few years ago when he started
having problems standing up after squatting and kneeling. He later developed
problems pulling weeds and using some of his gardening tools. In the last year,
he has even been having problems picking up small objects, squeezing a
toothpaste tube, and other daily activities.
His current medications include metformin, colchicine, and atorvastatin. He has
not had fever, night sweats, or other constitutional symptoms. He has also been
having some dysphagia, but his esophagogastroduodenoscopy was normal. Vital
signs are in the normal range. The patient walks with a steppage gait and has
problems transferring to the examination table. He has some atrophy of the
facial muscles, but the extraocular muscles are intact. He also has significant
muscle atrophy and weakness in the biceps and quadriceps bilaterally that is
worse on the left than on the right. Motor strength is 3+/5 in the proximal upper
and lower extremities muscle groups. He has weakness with dorsiflexion of the
feet bilaterally and weakness of finger flexion. Reflexes are less than 1+
throughout. Babinski’s reflex is negative. Laboratory studies are unremarkable
with normal creatine phosphokinase and aldolase levels, inflammatory indices,
and thyroid function tests.
An electromyography/nerve conduction velocity test is done, which reveals
fibrillation potentials with positive sharp waves, polyphasia, and small-duration
small-amplitude motor unit potentials (MUP) in the distal muscles, and
polyphasia with small-duration small-amplitude MUP in the proximal musculature.
Nerve conduction studies show absent H responses and absent sensory
responses in the lower extremities, normal sensory responses in the upper
extremities, and strongly decreased compound motor action potential amplitudes
and slightly reduced motor conduction velocities in the arms and legs.
Muscle biopsy demonstrates a slight decrease in density of myelinated fibers
with a preferential loss of large myelinated fibers. Clusters of small regenerated
axons and small epineural perivascular lymphocytic infiltrate of CD4+ and CD8+
T cells are observed, but no vasculitic features are noted.
Click here for references

What is the most likely cause of this patient’s muscle weakness and gait
abnormalities?

A. Autoimmune necrotizing myopathy (statin-induced)

B. Colchicine toxicity

C. Facioscapulohumeral dystrophy


D. Sporadic inclusion body myositis

Rationale

Sporadic inclusion body myositis (s-IBM) is the most common myopathy in the
elderly. The disease course is slowly progressive leading to delayed diagnosis in
many patients. It most severely affects the forearm flexor and quadriceps femoris
muscles as well as distal muscle groups. The distribution can be asymmetrical at
times. The patient can also present with facial muscle involvement, although
ocular muscles are usually spared. Some authors have described more
prominent weakness in the flexor digitorum profundus than the flexor digitorum
superficialis, which results in more weakness in distal interphalangeal joint flexion
than in proximal interphalangeal joint flexion. Early in the course of the disease
the symptoms can be attributed to arthritis. s-IBM can also be misdiagnosed as
motor neuron disease. Creatine kinase levels are variable and may be normal or
only mildly elevated. Muscle biopsy in s-IBM shows a triad of inflammatory
changes, with invasion by CD8+ lymphocytes of muscle fibers expressing major
histocompatibility complex-I; cytoplasmic and intranuclear inclusions containing
amyloid beta and several other Alzheimer-type proteins; and segmental loss of
cytochrome C oxidase activity in muscle fibers.
Autoimmune necrotizing myopathy has usually been described after exposure to
statins, and usually persists following discontinuation of statin therapy. It can also
be seen in association with other connective tissue disorders such as
scleroderma or mixed connective tissue disease. It has also been described as a
paraneoplastic syndrome and is idiopathic in some cases as well. Patients
present with proximal upper and lower extremity muscle weakness without
involvement of the distal and intrinsic muscles. Electromyography findings are
typical of an inflammatory myopathy, and muscle enzymes are elevated.
However, muscle biopsies show necrotic muscle fibers without significant
inflammatory cell infiltrate around nonnecrotic fibers. No evidence of muscle
atrophy is present. Some of these patients have antibodies to signal recognition
peptide or to another antibody, anti-200/100. This antibody targets
hydroxymethylglutaryl coenzyme, which is a reductase protein.
Long-term daily colchicine use can be associated with neuromyopathy. Patients
present with proximal muscle weakness, often more prominent in the lower than
upper extremities. The patients may also have mild sensory symptoms and
diminished deep tendon reflexes. Unlike in this case, creatine phosphokinase
levels are almost always elevated (10- to 20-fold above normal).
Electromyography and nerve conduction studies show myopathic changes and
axonal neuropathy. Myotonic discharges may be noted in some patients and
have been correlated with clinical findings of myotonia. Muscle biopsy frozen
section samples show vacuolar changes. Paraffin fixation is to be avoided as it
results in vacuole damage.
Facioscapulohumeral dystrophy (FSHD) is the third most common type of
muscular dystrophy, which presents with slowly progressive muscle weakness
that is usually asymmetrical and involves the facial, scapular, upper arm, lower
leg, and hip girdle muscles. The age of symptom onset varies from infancy to
middle age, but 90% of patients have clinical findings by 20 years of age. Facial
muscles are involved early in the disease and can involve periorbital muscles.
Scapular winging is a common early feature. Biceps and triceps are affected, but
forearm and more distal upper extremity muscles are spared. Abdominal muscles
involvement results in a protuberant abdomen. Other manifestations of FSHD
may include pain, retinal vasculopathy, hearing loss, cardiac arrhythmia,
cognitive impairment, and epilepsy. Serum creatine kinase is often but not
always elevated (2-3 times normal range)in patients with symptomatic FSHD.
Electromyography typically displays myopathic features with low amplitude, short
duration, and polyphasic potentials. The muscle biopsy shows mainly nonspecific
myopathic changes, with large hypertrophic fibers (these are absent in
polymyositis) and a few angulated atrophic fibers. Mild inflammatory infiltrates
can be present in up to 40% of patients with FSHD. The diagnosis of FSHD is
confirmed by genetic testing. Interestingly, the extensor digitorum brevis muscle
is helpful in the diagnosis of FSHD, because it is usually hypertrophic in FSHD
but is usually atrophic in peripheral motor neuropathies.

Educational Objective: To recognize the importance of atypical patterns of


muscle weakness, such as weakness of finger flexion and knee extension, in
differentiating inclusion body myositis from other types of inflammatory myopathy

Case 29
A 66-year-old man presents to the emergency room with a 4-day history of
worsening cough and nonradiating chest pain, which worsens with deep breaths.
He denies recent travel, immobilization, orthopnea, and nocturnal dyspnea. He is
a current smoker (1 pack per day for the last 30 years).
He has a known history of granulomatosis with polyangiitis diagnosed 1 year
ago, when he was found to have acute renal failure, crescentic
glomerulonephritis, and cavitary lung disease. He was treated with monthly
intravenous cyclophosphamide for 6 months and a tapering dose of steroids.
Following successful induction therapy, he has been doing well on azathioprine.
His current medications include azathioprine 150 mg daily, prednisone 5 mg
daily, hydrochlorothiazide 25 mg daily, lisinopril 10 mg daily, calcium/vitamin D
600 mg/400 IU twice daily, and trimethoprim–sulfamethoxazole 160 mg/800 mg 3
times weekly (Monday, Wednesday, and Friday).
On examination, his temperature is 100.0°F (37.8°C), heart rate is 120 bpm,
blood pressure is 153/90 mm Hg, respiratory rate is 28 breaths per minute, and
oxygen saturation is 89% on room air. He is in mild distress due to dyspnea. No
nasal crusting is noted. His neck is supple, with no jugular venous distention.
Crackles are heard at both lung bases. His heart is tachycardic with regular
rhythm and no rubs or gallops. No clubbing or cyanosis is noted. Homan’s sign is
negative. No calf asymmetry is apparent, but slight pitting edema is noted, which
is slightly greater in the right lower extremity. Laboratory results on today’s visit
are shown below (Table).

Table. Results of Laboratory Tests

Laboratory Test Result Reference Range

Antineutrophil cytoplasmic Positive 1:160 Less than 1:20


antibody, cytoplasmic (C-ANCA)

C-reactive protein 2.0 mg/dL 0.8 mg/dL or less

Creatinine, serum 2.4 mg/dL (stable from 6 months 0.7–1.5 mg/dL


ago)

Erythrocyte sedimentation rate 60 mm/hr Male: 0–15 mm/hr


(Westergren)

Hemoglobin, blood 10.9 g/dL Male: 14–18 g/dL

Lactate dehydrogenase, serum 200 U/L 80–225 U/L

Leukocyte count 8000/μL (70% segmented 4000–11,000/μL


neutrophils, 20% lymphocytes)

Urinalysis No blood, trace protein Negative

A chest radiograph is obtained, which is unchanged from his previous image,


showing slightly increased interstitial markings at the bases. Electrocardiography
is performed, which aside from showing tall R waves in the lateral leads, shows
no evidence of ischemic changes. He is started on empiric antibiotics.

Click here for references

Which of the following is the next most appropriate step in diagnosis?

A. Bronchoscopy with Grocott's methenamine silver stain


B. Contrasted computed tomography of chest

C. Left heart catheterization

D. Ventilation/perfusion scan

Rationale

An increased rate of thromboembolic disease among the ANCA-associated


vasculitides is well described. This increased risk is likely multifactorial and
related to the effects of systemic inflammation leading to a procoagulant state,
stasis related to chronic illness/immobilization, and endothelial damage. The risk
is highest within 2 years after initial diagnosis and remains high despite control of
inflammation. In this patient, the acute nature of symptoms, manifesting as
pleuritic chest pain and dyspnea, and his stable pulmonary examination findings
suggest an alternate etiology from granulomatosis with polyangiitis. Even with a
negative Homan’s sign (the sensitivity of which is rather poor) and no calf
asymmetry, venous thromboembolism is a consideration and should be further
evaluated. Note that elevated acute phase reactants can occur in the setting of
infection or an inflammatory rheumatic process and when there is an acute
infarction/thromboembolic event.
An infectious process is also of concern in this immunosuppressed patient. In
addition to conventional bacterial organisms, less common pathogens such
asPneumocystis jiroveci pneumonia (PJP) should be considered. Although the
decision to start empiric antibiotics for coverage of community acquired
pneumonia is reasonable in this patient, the absence of leukocytosis/bandemia
and an infiltrate on the chest radiograph is reassuring. Also, the normal lactate
dehydrogenase level, absolute lymphocyte count greater than 1000/µL, and the
fact that the patient is on trimethoprim–sulfamethoxazole makes PJP less likely.
Safer methods can be used to assess for venous thromboembolism in this
patient, and pursuing a contrasted computed tomography scan of the chest
places him at risk of worsening renal dysfunction. Although the risk of
cardiovascular disease is also increased in patients with ANCA vasculitis, the
nature of this patient’s chest pain, along with the reassuring electrocardiogram,
suggest against an acute coronary event.
Educational Objective: To recognize that there is an increased risk of venous
thromboembolic events in granulomatosis with polyangiitis.
Case 30
A 10-year-old boy presents with hip and knee pain after exercise. Clinically
normal at birth, he exhibited a mild delay in motor development but otherwise has
been healthy. His parents are healthy without any medical conditions. A maternal
grandfather had a total hip arthroplasty at 40 years of age. Physical examination
reveals short stature, mild muscular hypotonia, and laxity of the knees and hips
without synovitis or effusion. Laboratory tests reveal a normal serum creatine
kinase level.
Results of a radiographic skeletal survey show small, rounded phalangeal
epiphyses of the hands bilaterally (Figure 1). The proximal femoral epiphyses are
round and small for chronologic age bilaterally, with evidence of
undermineralization and irregularity of the acetabular margin (Figure 2).
Vertebrae are normal.

Figure 1. Radiograph of Hands

Figure 2. Radiograph of Pelvis


Click here for references

What is the most likely diagnosis?

A. Legg-Calve-Perthes disease

B. Multiple epiphysial dysplasia

C. Pseudoachondroplasia

D. Vitamin D-dependent rickets

Rationale

The multiple epiphysial dysplasias (MED) are a heterogenous group of disorders


marked by a genetic defect that results in abnormal epiphysial
ossification. Mutations in COMP, DTDST (SLC26A2), and MATN3produce the
most severe expressions of disease; however, multiple other gene mutations
have been described that produce milder phenotypes. Most forms exhibit
autosomal dominant inheritance patterns, although mutations in EDM4 exhibit an
autosomal recessive pattern.
Common presenting features include hip and knee pain with physical activity.
Although manifestations are not apparent at birth, mild to moderate deviation
from the normal growth curve occurs throughout development, resulting in short
stature. Some forms of MED also manifest with muscular hypotonia. Patients
with MED often have a family history of early osteoarthritis or joint replacement in
first- or second-degree relatives.
The other answer choices are incorrect. Radiographic changes of the femoral
heads in patients with MED may resemble those seen in idiopathic necrosis of
the femoral head (Legg-Calve-Perthes Disease), but in Legg-Calve-Perthes
disease the abnormality is typically unilateral and not associated with short
stature or other generalized skeletal anomaly. Pseudoachondroplasia may have
similar genetic mutations but tends to show a more severe phenotype including
vertebral involvement, specifically rounding of the vertebral bodies. Severe
vitamin D-dependent Rickets results in defective bone mineralization before
epiphyseal plate closure. Classic radiographic findings include pseudofractures,
especially pseudofractures of the femoral neck and long bones, and some
vertebral abnormalities, such as “cupping and fraying.”
Educational Objective: Recognize the bone changes in school-age children
with multiple epiphyseal dysplasia.

Case 31
A 61-year-old woman with a 15-year history of erosive and seropositive
rheumatoid arthritis (RA) is being treated with hydroxychloroquine and
sulfasalazine. She was hospitalized 2 months ago for her second bout of
bacterial pneumonia in 6 months. She reports still not feeling well and has had
persistent fatigue, night sweats, and a 15-lb (6.8-kg) weight loss.
On physical examination, her temperature is 99.9°F (37.7°C), blood pressure is
120/70 mm Hg, respiratory rate is 18 breaths per minute, and heart rate is 75
bpm. Her examination is normal, except for the musculoskeletal examination,
which reveals chronic synovitis of the metacarpophalangeal and proximal
interphalangeal joints, wrists, and knees with multiple rheumatoid nodules over
the elbows and Achilles tendons bilaterally.
A chest radiograph shows resolution of her pneumonia, and a computed
tomography scan of the abdomen reveals no lymphadenopathy, a normal liver,
and a slightly enlarged spleen. Results of laboratory studies are listed below
(Table).

Table. Results of Laboratory Tests


Laboratory Test Result Reference Range
Anticyclic citrullinated 835 units Less than 20 units
peptide, antibodies to
Antinuclear antibodies 1:320 Greater than 1:40 is abnormal
Hemoglobin, blood 10.1 g/dL Female: 12-16 g/dL
Leukocyte Count 3900/µL 4000-11,000/µL
Eosinophils 4% 0-3%
Lymphocytes 60% 30-45%
Monocytes 16% 0-6%
Neutrophils 20% 50-70%
Platelet count 250,000/µL 150,000-300,000/µL
Rheumatoid factor 350 IU/mL Less than 24 IU/mL
(nephelometry)

A peripheral blood smear reveals numerous large granular lymphocytes. Flow


cytometry of peripheral blood reveals 40% CD3+ CD8+ T cells, expressing an
alpha beta T cell receptor but no natural killer cell markers.
Click here for references

Which agent would be your next treatment of choice for her neutropenia?

A. Etanercept

B. Methotrexate

C. Prednisone

D. Rituximab

Rationale

This patient with seropositive RA has developed recurrent bacterial infections in


the setting of neutropenia. Flow cytometry studies show a clonal population of
cytotoxic T cells, consistent with a large granular lymphocyte (LGL) syndrome,
associated with her RA. The spectrum of LGL syndromes includes subsets of
cytotoxic T cells (T-LGL) and natural killer cells (NK-LGL), representing 85% and
15% of the cases respectively. Each subset may follow an indolent course of
lymphocytosis (commonly associated with autoimmune diseases) or manifest as
leukemia, with an aggressive and fatal course. The pathogenesis of LGL
syndromes is poorly understood, but it is thought to be from chronic antigenic
stimulation of the immune system, with an increase in survival pathway signaling
and a decrease in apoptosis pathways. Neutropenia results from granulopoiesis
inhibition via LGL cell chemokine and cytokine secretion, and abnormalities in the
Fas pathway are hypothesized to play a key role.
Up to 40% of patients with LGL syndrome will have an autoimmune disease such
as RA, whereas only 1% of patients with RA have the associated LGL
syndrome. LGL and Felty’s syndrome may exist on a continuum of
disease. LGL disorders can also be seen in association with other hematologic
malignancies (eg, myelodysplastic syndrome), allogeneic stem cell
transplantation, and viral infection. In older individuals, LGL disorders can result
from immune system senescence.
In addition to neutropenia, patients with RA and LGL syndrome may develop
constitutional symptoms (20%-40%), anemia (50%), and less commonly,
thrombocytopenia. Agents that have shown some therapeutic success include
methotrexate, cyclosporine A, and cyclophosphamide. Each agent has been
shown to induce remission in more than 50% of patients, but relapse is common
after the agent is discontinued. Induction of remission has been seen in up to
55% of patients treated with methotrexate 10 mg/m2, half of whom also had
molecular disappearance of the clonal T cell population. In patients with RA,
continuation of low dose weekly methotrexate may be sufficient to control
neutropenia.
Etanercept would not be a correct choice. In the few studies of tumor necrosis
factor inhibitors in RA-associated LGL syndrome, no effect on the neutropenia
was observed. In fact, these agents have been known to cause leukopenia.
Prednisone has shown disappointing results for the treatment of neutropenia
related to RA-associated LGL syndromes. In fact, a response rate was seen in
only 9% of patients studied, with no patients achieving remission. The best role
for prednisone is as an adjunct with other immunosuppressive therapies. LGL
syndromes are primarily cell mediated disorders; however, some antibodies have
been implicated in pathogenesis. In theory, rituximab may have some efficacy,
but it has not been studied in this subset of patients with RA.
Educational Objective: Identify large granular lymphocyte syndrome as a cause
of neutropenia in rheumatoid arthritis, and identify methotrexate as a treatment.

Case 32
A 6-month-old boy presents to the emergency room with 5 days of high fever to
104° F (40° C), irritability, and rash. His mother states that he has not been
sleeping or feeding well but is making adequate wet diapers. The fever has
responded slightly to ibuprofen and acetaminophen but rises again 2 hours after
medication. He has no cough, congestion, or rhinorrhea. Yesterday night, he
began to have a rash in the diaper area that is now spreading to the chest, arms,
legs, hands, and feet. He stays home with the mother or grandmother during the
day and has had no contact with anyone known to be sick.
On examination, his temperature is 102.6°F (39.2°C), heart rate is tachycardic at
160 bpm, respiratory rate is within normal limits at 40 breaths per minute, blood
pressure is normal for age at 86/46 mm Hg, and pulse oxygen is 99% on room
air. The patient is irritable and crying on examination. His head is normocephalic
and atraumatic. The tympanic membranes are clear, sclera are clear, extraocular
muscles are intact, pupils are round and reactive to light, lips are dry and
cracked, and tongue and oropharynx are mildly erythematous. A small, pea-sized
(0.5 cm) cervical lymph node is noted on the neck. The chest is clear to
auscultation, but the heart is tachycardic with 2/6 systolic ejection murmur. The
abdomen is soft and nondistended, with positive bowel sounds. An erythematous
macular rash is noted in the diaper area and on the chest, arms, and legs. The
hands and feet appear red and slightly swollen. No desquamation is seen. The
musculoskeletal examination is difficult due to swelling of the hands and feet, but
no specific limitation of range of motion is seen elsewhere. Results of laboratory
tests are listed below (Table).
Table. Results of Laboratory Tests

Laboratory Test Result Reference Range

Albumin, serum 3.0 g/dL 3.5-5.5 g/dL

Aminotransferase, serum 87 U/L 10-40 U/L


alanine (ALT, SGPT)

Aminotransferase, serum 102 U/L 10-40 U/L


aspartate (AST, SGOT)

Bicarbonate, serum 29 mE/L 23-28 mE/L

Calcium, serum 8.7 mg/dL 8.6-10.2 mg/dL

Chloride, serum 102 mE/L 98-106 mE/L

C-reactive protein 5.4 mg/dL 0.8 mg/dL or less

Creatinine, serum 0.3 mg/dL 0.3-1.10 mg/dL


Erythrocyte sedimentation rate 82 mm/hr Male: 0-15 mm/hr
(Westergren)

Ferritin, serum 350 ng/mL Male: 20-235 ng/mL

Hemoglobin, blood 11.2 g/dL 11.5-15.5 g/dL

Leukocyte count 17,000/µL 4000-11,000/µL

Platelet count 175,000/µL 150,000-300,000/µL

Potassium, serum 3.8 mE/L 3.5-5.0 mE/L

Sodium, serum 135 mEq/L 136-145 mEq/L

Total bilirubin, serum 0.6 mg/dL 0.3-1.0 mg/dL

Total protein, serum 6.4 g/dL 5.5-9.0 g/dL

Urea nitrogen, blood 13 mg/dL 8-20 mg/dL

Click here for references

Which of the following is the most appropriate treatment for this patient?

A. Antibiotics

B. Corticosteroids

C. Gammaglobulin and aspirin

D. Nonsteroidal anti-inflammatory drugs and intravenous fluids


Rationale

The infant in this vignette has incomplete Kawasaki disease (KD). KD is one of
the most common vasculitides of childhood and typically presents in children less
than 5 years of age. Coronary artery aneurysms resulting from this vasculitic
condition may lead to significant morbidity and mortality in either complete or
incomplete KD. Criteria for Kawasaki disease include fever for at least 5 days
and 4 of the following characteristics:

 Bilateral conjunctival injection without purulence


 Oral mucous membrane changes including red and cracked lips,
erythematous pharynx, or strawberry tongue
 Redness or swelling of hands and feet with subsequent desquamation
 Polymorphous rash
 Cervical lymphadenopathy with at least one lymph node > 1.5 cm in diameter

This child has had a fever for 5 days but only 3 criteria for KD – oropharyngeal
changes, swelling of hands and feet, and rash. However, infants are at higher
risk than older children of both incomplete KD and coronary aneurysm
development. Prompt treatment with intravenous gammaglobulin (IVIg) and high
dose aspirin has been shown to reduce the development of aneurysms in this
disease. With this child meeting 3 criteria in addition to fever, supplemental
criteria are used to determine his risk of incomplete KD. The American Heart
Association and American Academy of Pediatrics have published an algorithm
for evaluation of suspected incomplete KD, which recommends evaluation of C-
reactive protein (CRP), erythrocyte sedimentation rate (ESR), complete blood
count, urine, alanine aminotransferase (ALT), and albumin. Laboratory findings
suggestive of KD include the following:

 CRP > 3.0 mg/dL or ESR > 40 mm/hr


 White blood cell (WBC) > 15,000/uL
 Normocytic normochromic anemia
 Platelet count > 450,000/uL after 7 days of illness
 Pyuria with > 10 WBC/hpf
 Serum ALT > 50 U/L
 Serum albumin < 3 g/dL

It should be noted that thrombocytosis and desquamation are considered later


findings in KD and may not be seen until later in the disease. Greater than or
equal to 3 supplemental laboratory criteria in the setting of increased acute
inflammatory markers are considered sufficient to empirically treat with IVIg for
incomplete KD. Risk factors for coronary artery aneurysm in KD include age
younger than 1 year or older than 9 years, male gender, longer duration of fever,
and failure to respond to initial IVIg therapy. This patient presents with increased
inflammatory markers and 3 supplemental criteria, and in the setting of a male
infant with increased risk for coronary aneurysm, treatment should be given
regardless of echocardiography findings at this point.
Antibiotics would be used for a bacterial infection. Scarlet fever is a known
sequela of streptococcal pharyngitis that can cause fever and rash; however,
scarlet fever is a syndrome of a diffuse red, “sandpaper” rash, which starts on the
head and neck and moves to the trunk and extremities. It is classically known for
desquamation of the extremities later in the course. Palms and soles are typically
spared. Children with scarlet fever may have a strawberry tongue as well. The
child in this description does not have the fine papular rash typical for scarlet
fever.
Corticosteroids could be considered for an ill child with systemic juvenile
idiopathic arthritis (sJIA); however, children with sJIA present with quotidian fever
(fever that spikes once or twice daily) and arthritis. A salmon-colored transient
migratory rash and lymphadenopathy may commonly be seen. The child in this
description does not have quotidian fever, nor does he have arthritis on
examination. He does have an elevated ferritin level, but ferritin is an acute
phase reactant, which is elevated in many inflammatory conditions. A markedly
elevated ferritin level (> 1,000 ug/L) may be more suggestive of sJIA. KD and
sJIA may have features in common with marked elevation of inflammatory
markers, but the quality of fever and rash are typically different.
Symptomatic treatment could be considered for self-limited viral infections such
as roseola (also known as 6th disease), due to human herpesvirus 6. With
roseola, children typically develop 3 to 5 days of high fever, which then resolves,
and a blanching maculopapular rash that subsequently appears on the neck and
trunk spreading to face and extremities. Irritability and lymphadenopathy may be
seen while the child is febrile. Roseola is typically a benign, self-limited illness
and treated symptomatically with nonsteroidal anti-inflammatory drugs and
supportive treatment. The reason why the infant does not have this is because
the rash is present while the child is still febrile.
Educational Objective: Understand risk factors for coronary artery aneurysms
in Kawasaki disease. Understand that very young children are at highest risk of
atypical disease and coronary artery aneurysm.

Case 33
A 53-year-old woman presents to the clinic with a 10-year history of acne
rosacea without nasolabial fold involvement that has been responsive to
minocycline. In addition to minocycline, she also takes terbinafine, ranitidine, and
hydralazine. No previous lupus features have been noted, but she developed a
new onset rash described as gyrate erythema (Figure). Her past medical history
is notable for hypertension, gastroesphageal reflux, fungal nail infections, and
seasonal rhinitis. Basic laboratory studies comprising chemistries, complete
blood count, and urinalysis are all normal. Antibody testing reveals positive
antinuclear antibody (ANA), only at 1:80, with a homogeneous pattern, positive
anti-Ro (SSA) antibodies, negative antihistone antibodies, and negative anti–
double-stranded DNA antibodies. Complement levels are normal. Skin biopsy
reveals interface dermatitis with cellular infiltrate but without eosinophil infiltration,
between the dermis and epidermis.

Figure. New Onset Rash at Presentation

Click here for references

Which of the following is most likely to be the cause of this patient’s rash?

A. Ranitidine

B. Minocycline

C. Hydralazine

D. Terbinafine

Rationale

Both the anti-Ro (SSA) antibody and the biopsy showing interface dermatitis can
be seen in a number of illnesses on the lupus spectrum and in other connective
tissue diseases. The gross appearance of the rash and the anti-Ro positivity
further define the problem as subacute cutaneous lupus erythematosus (SCLE).
In this case, lack of systemic lupus erythematosus (SLE) features make it among
the 50% of SCLE cases that are independent, rather than SLE with SCLE as a
manifestation. No test is available that will determine definitively if the SCLE is
drug induced versus native (idiopathic) SCLE. However, because the patient is
taking the most highly suspect drug for drug-induced SCLE—the terbinafine—
this drug should be removed with the expectation that the gyrate, serpiginous
rash will resolve within several weeks (mean of 7 weeks, median of 4 weeks).
Stopping medication may work as fast or faster than adding hydroxychloroquine,
rendering the latter unnecessary. It would not be expected to act within the
timeframe.
SLE and SCLE can occur concurrently when they are not drug induced.
However, the drug-induced variants differ pathophysiologically. For example, the
drugs that trigger drug-induced SLE seem to be almost entirely independent from
those that trigger drug-induced SCLE. Anti-tumor necrosis factor (TNF) drugs are
the only class associated with both drug-induced SLE and drug-induced SCLE.
In addition, as many as one third of SCLE cases seen in the general population
may be drug induced, whereas the drug-induced variant of SLE is much less
frequent compared with native lupus.
Odds ratios for the occurrence of drug-induced SCLE are 38.5 (95% confidence
interval [CI] 6.6–∞) for terbinafine, 8.0 (95% CI 1.6–37.2) for anti-TNF drugs, 3.4
(95% CI 1.9–5.8) for antiepileptics, and 2.9 (95% CI 2.0–4.0) for proton pump
inhibitors. Hydralazine and procainamide are perhaps the agents most firmly
associated with drug-induced SLE, whereas procainamide and hydralazine do
not predispose patients to drug-induced SCLE. Tetracyclines have been weakly
associated with drug-induced SLE but not with drug-induced SCLE.
Even though antinuclear, anti-Ro, and antihistone antibodies could occur in SLE,
SCLE, or their drug-induced variants, anti-Ro antibodies are more frequent in
SCLE (80% of cases), whereas antihistone antibodies occur at the highest
frequency, 90%, in drug-induced SLE cases. Confusingly, the antihistone
antibodies are nonspecific. They also occur in two frequently encountered
situations that are decidedly not drug-induced SLE. These two situations are the
33% of drug-induced SCLE cases that are antihistone positive and the 60% of
native SLE cases that are antihistone positive. Therefore, the results of the
antihistone test alone will not be discriminating in choosing between native SLE,
drug-induced SLE, and drug-induced SCLE. The serologies therefore might
contribute to confirming a diagnosis, but they are not as important as the
appearance of the patient on physical examination and the biopsy result.
Of note, the duration between drug initiation and the onset of drug-induced SCLE
varies widely but averages 6 weeks, resulting in a palindromic time course—
about 6 weeks to incubate and about 6 weeks for the rash to resolve upon
discontinuation of the drug. Patients with anti-Ro antibodies due to drug-induced
SCLE remain anti-Ro positive 67% of the time even after the drug is withdrawn
and the rash resolves.
Educational Objective: To test for recognition that the drugs leading to drug-
induced systemic lupus differ from those leading to drug-induced subacute
cutaneous lupus

Case 34
A 45-year-old woman who is a current smoker was referred to the rheumatology
clinic for weakness and pain, especially in her hips and knees, which has been
present for 2 years. She also notes a growing lump on her right wrist. The
physical examination is significant for diffuse tenderness to palpation, 4/5
strength in the hip and shoulder girdle, tenderness on palpation of the long
bones, and a 2-cm nontender soft tissue mass on the dorsal right wrist. No
synovitis is noted. Results of laboratory tests are listed below (Table).

Table. Results of Laboratory Tests

Laboratory Test Result Reference Range

1,25-Dihydroxyvitamin D (1,25 10 ng/mL 15-60 ng/mL


dihydroxycholecalciferol)

25-Hydroxyvitamin D (25- 32 ng/mL 30-60 ng/mL


hydroxycholecalciferol)

Albumin, serum 4.5 g/dL 3.5-5.5 g/dL

Alkaline phosphatase, serum 200 U/L 30-120 U/L

Calcium, serum 9.2 mg/dL 8.6-10.2 mg/dL

Creatinine, serum 1.0 mg/dL 0.7-1.5 mg/dL

Erythrocyte sedimentation rate 30 mm/hr Female: 0-20 mm/hr


(Westergren)

Parathyroid hormone, serum intact 65 pg/mL 10-65 pg/mL

Phosphorus, serum 1.8 mg/dL 3.0-4.5 mg/dL

Magnetic resonance imaging of the right knee showed a partial medial meniscal
tear and a fracture of the medial tibial plateau. The left wrist mass was resected.
Histology is pending, but the patient’s systemic symptoms improved greatly.

Click here for references

What is the most likely cause for this patient’s systemic symptoms?

A. Hypertrophic osteoarthropathy

B. Primary Hyperparathyroidism

C. Osteomalacia

D. Osteoporosis

Rationale

Osteomalacia is a metabolic bone disorder caused by decreased mineralization


of osteoid during bone remodeling. Symptoms have an insidious onset. Bone
pain, muscle weakness, and bone tenderness are the most commonly reported
symptoms. Gait abnormalities such as waddling gait, bone fractures, and muscle
spasms may also occur. Laboratory abnormalities depend on the cause, but an
elevated alkaline phosphatase level is a common finding. Most cases of
osteomalacia are caused by vitamin D deficiency. Hypophosphatemic
osteomalacia may be caused by inherited disorders (X-linked, autosomal
dominant, or autosomal recessive hypophosphatemic rickets; hereditary rickets
with hypercalciuria; Dent disease; and idiopathic hypercalciuria) or tumor-induced
osteomalacia.
In this case, the patient has tumor-induced osteomalaia. Tumor-induced
osteomalacia, also known as oncogenic osteomalacia, is an unusual
paraneoplastic syndrome. Many of the tumors that cause this disorder are
mesenchymal tumors, usually arising in soft tissue or bone.. Other conditions
associated with tumor-induced osteomalacia are disseminated carcinomas,
neurofibromatosis type 1, and polyostotic fibrous dysplasia. Expression of
fibroblast growth factor-23 (FGF23) by tumor cells can lead to elevation of serum
levels of FGF23. FGF23 binds renal proximal tubule cells and causes increased
phosphate excretion. It also decreases hydroxylation of 25-hydroxyvitamin D to
active 1,25-dihydoroxyvitamin D. The low serum phosphorus
(hypophosphatemia) and inappropriately low plasma calcitriol (1,25-OH vitamin
D) seen in this case are characteristic features. Urine phosphorus would be
expected to be high. Tumor-induced osteomalacia is treated with phosphate and
1,25-dihydroxyvitamin D supplementation and resection of the tumor. The tumors
are usually small and may be difficult to detect. Scintigraphy with octreotide
labeled with indium-111 will detect the tumors that express somatostatin
receptors. Tumor localization may require positron emission
tomography/computerized tomography or magnetic resonance imaging.
Hypertrophic osteoarthropathy may cause pain in the distal extremities that is
exacerbated by dependent position. Diffuse arthralgias and joint effusions may
also occur. Clubbing is a common physical finding. Periosteal thickening on
radiography and increased tracer uptake on bone scan are seen in hypertrophic
osteoarthropathy, but fractures are not typical. Alkaline phosphatase level may
be elevated due to periosteal bone formation. Serum phosphate and vitamin D
levels are not affected by this disorder.
Primary hyperparathyroidism increases serum and urine calcium levels and
increases conversion of 25-OH vitamin D to 1,25 OH vitamin D, in contrast to
what is seen in this case. Bone cysts and brown tumors may be seen in long
bones in primary hyperparathyroidism, but improvement in her symptoms would
not be expected with resection of the wrist mass.
Osteoporosis is asymptomatic unless there is a fracture. Elevations in alkaline
phosphatase can be seen in the setting of fractures and fracture healing,
however this patient’s symptoms low phosphorus, and improvement with wrist
nodule resection cannot be explained by a fractures.

Case 34
A 45-year-old woman who is a current smoker was referred to the rheumatology
clinic for weakness and pain, especially in her hips and knees, which has been
present for 2 years. She also notes a growing lump on her right wrist. The
physical examination is significant for diffuse tenderness to palpation, 4/5
strength in the hip and shoulder girdle, tenderness on palpation of the long
bones, and a 2-cm nontender soft tissue mass on the dorsal right wrist. No
synovitis is noted. Results of laboratory tests are listed below (Table).

Table. Results of Laboratory Tests

Laboratory Test Result Reference Range

1,25-Dihydroxyvitamin D (1,25 10 ng/mL 15-60 ng/mL


dihydroxycholecalciferol)

25-Hydroxyvitamin D (25- 32 ng/mL 30-60 ng/mL


hydroxycholecalciferol)

Albumin, serum 4.5 g/dL 3.5-5.5 g/dL

Alkaline phosphatase, serum 200 U/L 30-120 U/L

Calcium, serum 9.2 mg/dL 8.6-10.2 mg/dL

Creatinine, serum 1.0 mg/dL 0.7-1.5 mg/dL

Erythrocyte sedimentation rate 30 mm/hr Female: 0-20 mm/hr


(Westergren)

Parathyroid hormone, serum intact 65 pg/mL 10-65 pg/mL

Phosphorus, serum 1.8 mg/dL 3.0-4.5 mg/dL

Magnetic resonance imaging of the right knee showed a partial medial meniscal
tear and a fracture of the medial tibial plateau. The left wrist mass was resected.
Histology is pending, but the patient’s systemic symptoms improved greatly.

Click here for references

What is the most likely cause for this patient’s systemic symptoms?

A. Hypertrophic osteoarthropathy


B. Primary Hyperparathyroidism

C. Osteomalacia

D. Osteoporosis

Rationale

Osteomalacia is a metabolic bone disorder caused by decreased mineralization


of osteoid during bone remodeling. Symptoms have an insidious onset. Bone
pain, muscle weakness, and bone tenderness are the most commonly reported
symptoms. Gait abnormalities such as waddling gait, bone fractures, and muscle
spasms may also occur. Laboratory abnormalities depend on the cause, but an
elevated alkaline phosphatase level is a common finding. Most cases of
osteomalacia are caused by vitamin D deficiency. Hypophosphatemic
osteomalacia may be caused by inherited disorders (X-linked, autosomal
dominant, or autosomal recessive hypophosphatemic rickets; hereditary rickets
with hypercalciuria; Dent disease; and idiopathic hypercalciuria) or tumor-induced
osteomalacia.
In this case, the patient has tumor-induced osteomalaia. Tumor-induced
osteomalacia, also known as oncogenic osteomalacia, is an unusual
paraneoplastic syndrome. Many of the tumors that cause this disorder are
mesenchymal tumors, usually arising in soft tissue or bone.. Other conditions
associated with tumor-induced osteomalacia are disseminated carcinomas,
neurofibromatosis type 1, and polyostotic fibrous dysplasia. Expression of
fibroblast growth factor-23 (FGF23) by tumor cells can lead to elevation of serum
levels of FGF23. FGF23 binds renal proximal tubule cells and causes increased
phosphate excretion. It also decreases hydroxylation of 25-hydroxyvitamin D to
active 1,25-dihydoroxyvitamin D. The low serum phosphorus
(hypophosphatemia) and inappropriately low plasma calcitriol (1,25-OH vitamin
D) seen in this case are characteristic features. Urine phosphorus would be
expected to be high. Tumor-induced osteomalacia is treated with phosphate and
1,25-dihydroxyvitamin D supplementation and resection of the tumor. The tumors
are usually small and may be difficult to detect. Scintigraphy with octreotide
labeled with indium-111 will detect the tumors that express somatostatin
receptors. Tumor localization may require positron emission
tomography/computerized tomography or magnetic resonance imaging.
Hypertrophic osteoarthropathy may cause pain in the distal extremities that is
exacerbated by dependent position. Diffuse arthralgias and joint effusions may
also occur. Clubbing is a common physical finding. Periosteal thickening on
radiography and increased tracer uptake on bone scan are seen in hypertrophic
osteoarthropathy, but fractures are not typical. Alkaline phosphatase level may
be elevated due to periosteal bone formation. Serum phosphate and vitamin D
levels are not affected by this disorder.
Primary hyperparathyroidism increases serum and urine calcium levels and
increases conversion of 25-OH vitamin D to 1,25 OH vitamin D, in contrast to
what is seen in this case. Bone cysts and brown tumors may be seen in long
bones in primary hyperparathyroidism, but improvement in her symptoms would
not be expected with resection of the wrist mass.
Osteoporosis is asymptomatic unless there is a fracture. Elevations in alkaline
phosphatase can be seen in the setting of fractures and fracture healing,
however this patient’s symptoms low phosphorus, and improvement with wrist
nodule resection cannot be explained by a fractures.

Case 35
A 49-year-old man who was previously healthy is admitted to the hospital with a
3-week history of fever, malaise, arthralgias, myalgias, and rash of the lower
extremities. He developed shortness of breath and cough with 1 episode of
hemoptysis 2 days before admission. He has a history of cocaine use and
smoking (1 pack per day for 30 years). He reports 2 episodes of intravenous drug
use in the past 3 months.
On physical examination, temperature is 101°F (38°C), blood pressure is 160/90
mm Hg, respiration rate is 24 breaths per minute, heart rate is 95 bpm, and
oxygen saturation is 96% on 2L/min. He appears ill. Multiple small purpuric
lesions are noted on the lower extremities. The chest examination reveals
crackles in the left lower lung field. The heart has a rapid rate and regular rhythm
with a Grade 2/6 systolic murmur, best heard at the base. The abdomen is
slightly tender in the right upper quadrant, with normal bowel sounds and no
hepatosplenomegaly. The extremities have no edema. The musculoskeletal
examination reveals tender metacarpophalangeal and proximal interphalangeal
joints and tender and slightly swollen knees.
A chest radiograph reveals scattered nodules in both upper lobes and infiltrate in
the left lower lobe. Results of laboratory tests are listed below (Table).

Table. Results of Laboratory Tests


Laboratory Test Result Reference Range
Antineutrophil cytoplasmic Positive ANCA Negative
antibody, cytoplasmic (C-
ANCA) Positive PR3 (proteinase 3)

Blood cultures 1 out of 4 bottles positive for Negative


Gram-positive cocci at 24 hours
Creatinine 1.7 mg/dL 0.7–1.5 mg/dL
Complements, serum
C3 80 mg/dL 100–233 mg/dL
C4 10 mg/dL 14–48 mg/dL
Hemoglobin, blood 9.7 g/dL Male: 14–18 g/dL
Leukocyte count 13,000/μL 4000–11,000/μL
Platelet count 120,000/μL 150,000–300,000/μL
Rheumatoid factor 185 IU/mL Less than 24 IU/mL
(nephelometry)
Urinalysis Negative blood Negative
1+ protein

Further blood cultures were ordered and Rheumatology was consulted


to evaluate for a primary ANCA-associated vasculitis.

Click here for references

Which diagnosis should you consider next?

A. Bacterial endocarditis

B. Goodpasture's syndrome

C. Granulomatous polyangiitis

D. Systemic lupus erythematosus

Rationale
ANCA have been associated with medications, irritable bowel syndrome,
connective tissue diseases, infections, and primary ANCA-associated vasculitis
(AAV). The reported infections associated with ANCA include HIV, chronic
hepatitis C, tuberculosis, leprosy, malaria, parvovirus, invasive amebiasis, and
endocarditis. This patient has multiple symptoms and signs that could be seen in
systemic vasculitis; however, mimics of AAV must be considered. The finding of
a cardiac murmur and absence of definitive blood culture results mandate
caution and consideration of bacterial endocarditis.
Other antibodies, especially rheumatoid factor, antinuclear antibody, and
anticardiolipin antibodies can be seen in subacute bacterial endocarditis (SBE)
as well. Treatment with antibiotics usually eliminates ANCA positivity in SBE. The
exact mechanisms whereby ANCA are induced in SBE are unknown, although
several have been postulated.
Granulomatous polyangiitis can present with all the features listed in this case;
however, the presence of a cardiac murmur and positive blood cultures make
bacterial endocarditis more likely. SLE and Goodpasture’s are less likely to
present with positive ANCA and PR3.
Educational Objective: Recognize that bacterial endocarditis can induce
production of antineutrophil cytoplasmic antibodies (ANCA).

Case 36
A 4-year-old boy with swelling and pain in his ankles and knees is referred by his
pediatrician for a rheumatologic evaluation. For the past 2 months, the child’s
parents have noted that in the morning, he walks stiffly and asks to be carried
frequently. For the past week, he has been awakening in the middle of the night
screaming inconsolably that his legs hurt but falls asleep eventually after
receiving ibuprofen.
On physical examination, the patient has normal vital signs for a 4-year-old child.
His knees are mildly swollen with normal range of motion but increased warmth
and tenderness on palpation. His musculoskeletal examination is otherwise
normal. The patient’s heart sounds are normal, lungs are clear, and abdomen is
not tender. Results of laboratory studies are listed below (Table).

Table. Results of Laboratory Tests


Laboratory Test Result Reference Range (Pediatric)
Erythrocyte sedimentation rate 75 mm/hr Male: 0-15 mm/hr
(Westergren)
Hemoglobin, blood 11 g/dL 11.5–14.5 g/dL
Leukocyte count 5500/µL 5000–14,500/µL
Platelet count 117,000/µL 150,000–450,000/µL

Click here for references

Which of the the patient’s following findings should prompt an evaluation for
malignancy in this patient?

A. Two arthritic joints

B. Morning stiffness

C. Severe nocturnal pain

D. Normal leukocyte count

Rationale

In general, nocturnal musculoskeletal pain in a child is a cause for further


investigation. Nocturnal pain is atypical for children with oligoarticular juvenile
idiopathic arthritis (JIA). Children with oligoarticular JIA may not complain of pain
at all. If they have pain, it is in the morning or at the end of an active day and
does not cause nighttime awakening. Children with benign nocturnal pains of
childhood, also known as growing pains, will have pain in the lower extremities
that is usually deep within the thigh, shin, or calf and is nonarticular. Pain occurs
in the evenings or nighttime, may awaken the child from sleep, and is resolved by
the morning. Physical examination and radiographs are normal. Benign nocturnal
pains of childhood are managed with massage and, when necessary, analgesics.
Common malignant causes of musculoskeletal pain in children are primary bone
tumors, metastatic bone tumors (most often neuroblastoma), and acute
lymphocytic leukemia causing malignant infiltration of bone marrow.
Children with oligoarticular JIA by definition have 4 or fewer arthritic joints.
Children with leukemia can have swollen, tender joints and often have pain on
palpation of bones due to marrow infiltrates. Most children with leukemia who
present with musculoskeletal symptoms have fewer than 4 painful joints.
Children with oligoarticular JIA commonly have morning stiffness and/or limping
that will resolve as the day goes on. Children may not specifically complain of
pain in the morning.
Children with oligoarticular JIA usually have normal blood cell counts and
erythrocyte sedimentation rate. Cell counts reflecting mild inflammation such as
mild leukocytosis and mild thrombocytosis are occasionally seen, but leukopenia,
thrombocytopenia and/or anemia, especially hemoglobin below 11 g/dL, are not
typical and should prompt further work-up for malignancy. The combination of
two abnormally low cell parameters is a significant cause for concern, especially
when accompanied by an elevated sedimentation rate.
Educational Objective: Recognize that thrombocytopenia, anemia, elevated
erythrocyte sedimentation rate and/or nighttime pain are features suggestive of
malignancy rather than juvenile idiopathic arthritis.

Case 37
A 71-year-old woman with longstanding osteoarthritis and a left knee
replacement presents with worsening right groin pain and many months of pain
and tenderness of the lower extremities from the knees to the ankles. She has
also had a lingering nonproductive cough. Her past medical history is otherwise
significant for osteoporosis, and she has been on bisphosphonates for the last 2
years. On physical examination, she has tripe palms, right groin pain with
passive hip flexion and internal rotation, a right knee effusion with crepitus and
joint line tenderness, mild nonpitting edema proximal to the ankles, and diffuse
lower extremity tenderness. A radiograph of the ankle was obtained (Figure).

Figure. Radiograph of Ankle


Click here for references

What is most likely to be found on further work-up?

A. An inflammatory knee effusion with leukocyte count of 5500/µL


B. Low T3 and free T4 thyroid hormone levels

C. Parenchymal lung nodules on a radiograph of the chest

D. Atypical femoral fracture on a radiograph of the hip

Rationale

This patient does have multifocal osteoarthritis, including the knees and the right
hip, but the combination of a chronic cough and leg tenderness – with
radiographic proximal ankle periostitis as seen in the figure (along the medial
tibia) – suggests that she has hypertrophic osteoarthropathy (HOA). Many
patients will also exhibit fingernail clubbing. Although HOA can be primary
without an underlying process, also known as pachydermoperiostosis, the
secondary form is often associated with malignancies and pulmonary diseases
including lung cancer. When periostitis and/or clubbing are present in an
otherwise healthy patient, a search for an underlying illness is imperative.
In both osteoarthritis and HOA, the two likely causes of this patient’s knee
effusion, the synovial fluid is not inflammatory, and the leukocyte count is usually
less than 500/µL. In HOA, the effusion is thought to represent a sympathetic
reaction to the nearby periostosis. Thus, a cell count of 5500/µL would be
unlikely in this patient.
Thyroid acropachy is another known cause of HOA, but it would present with
many of the symptoms of hyperthyroidism that are not described in this patient’s
history as well as high values for T3 and free T4.
Atypical femoral fractures (AFFs)are stress or insufficiency fractures frequently
associated with a periosteal stress reaction at the fracture site. As these
fractures occur along the diaphysis of the femur, patients with AFFs often present
with unilateral or bilateral dull or aching pain in the groin or thigh. Absolute risk of
AFFs in patients taking bisphosphonates is low, particularly with relatively short
term use as in this case. Furthermore, AFF would not explain this patient’s lower
leg pain, swelling and periostitis at the proximal ankle.
Educational Objective: Recognize clinical and radiographic evidence of
hypertrophic (pulmonary) osteoarthropathy that can develop in the setting of
chronic lower extremity osteoarthritis.
Case 38
A 68-year-old woman presents with a 3-month history of bilateral shoulder and
neck pain. She has 30 minutes of morning stiffness. Plain radiographs of the
neck performed 2 months ago revealed degenerative joint disease at cervical
vertebrae C3 to C7. Nonsteroidal anti-inflammatory drugs have offered minimal
relief. She has associated fatigue but no hip pain, headaches, jaw claudication,
scalp tenderness, fevers, night sweats, or weight loss.
On physical examination she has tenderness at the biceps tendon and the
subacromial bursa bilaterally. Active range of motion is limited to 120 degrees by
pain, which improves with passive range of motion. Laboratory results are listed
below (Table). Ultrasound imaging of the right shoulder demonstrates
glenohumeral synovitis, subacromial bursitis, and biceps tenosynovitis.
Table. Results of Laboratory Tests
Laboratory Test Result Reference Range
Anticyclic citrullinated peptide, 15 units Less than 20 units
antibodies to
C-reactive protein 1.7 mg/dL 0.8 mg/dL or less
Erythrocyte sedimentation rate 62 mm/hr Female: 0–20 mm/hr
(Westergren)
Rheumatoid factor 16 IU/ml Less than 24 IU/mL
(nephelometry)

Click here for references

Which of the following is the most appropriate treatment?

A. Ibuprofen

B. Prednisone

C. Prednisone and methotrexate


D. Subacromial glucocorticoid injections

Rationale

Subacromial bursitis, glenohumeral synovitis, and biceps tenosynovitis are


commonly seen on ultrasound or magnetic resonance imaging of the shoulder in
patients with polymyalgia rheumatica (PMR). Such findings are typically present
bilaterally and can be used to differentiate between PMR and noninflammatory
shoulder conditions. It should be noted that approximately one quarter of patients
with PMR have no hip girdle symptoms at diagnosis, as in this case. When hip
girdle symptoms are present, advanced imaging can frequently demonstrate
trochanteric bursitis. In this case, the ultrasound findings, elevated inflammatory
markers, and absence of distal symptoms support the diagnosis of PMR.
Therefore, the patient should be treated with low-dose prednisone.
Ibuprofen or other nonsteroidal anti-inflammatory drugs can be useful in the
treatment of osteoarthritis or other noninflammatory shoulder conditions. In this
case, morning stiffness, inflammatory findings in articular and periarticular tissues
and bilateral symptoms suggest against a noninflammatory process.
Prednisone and methotrexate would be an appropriate treatment if the diagnosis
of rheumatoid arthritis (RA) was suspected. Subacromial bursitis, glenohumeral
synovitis, and biceps tenosynovitis can be seen in RA and cannot be used to
differentiate between PMR and RA with certainty. However, the absence of distal
symptoms and examination findings, in combination with negative rheumatoid
factor and anticyclic citrullinated peptide antibodies suggest against RA as the
diagnosis.
Subacromial glucocorticoid injection can be effective in short-term treatment of
subacromial bursitis or rotator cuff tendinitis but would not be expected to provide
long term efficacy for PMR.
Educational Objective: To understand that patients with polymyalgia
rheumatica usually have a low grade synovitis and/or periarticular inflammation in
proximal joints that may be seen on ultrasound, magnetic resonance imaging, or
other imaging modalities

Case 39
A 48-year-old woman with a history of asthma and allergic rhinitis presents with
left foot and right wrist numbness and weakness, which began 2 weeks ago. For
the past 5 years she has had intermittent dyspnea, cough, and wheezing, which
improves with bronchodilators and is currently controlled with inhaled
corticosteroids. She had 1 episode of dizziness with an elevated heart rate 1
week ago but has no other associated symptoms.
On physical examination, vital signs are normal, and the examination of the heart
and lungs is unremarkable. Notable findings include diminished sensation to
pinprick in the left foot and right hand and weakness of the left foot dorsiflexion.
Results of laboratory studies are shown below (Table). A biopsy of the left sural
nerve demonstrates eosinophilic infiltrates in the walls of small blood vessels.
Table. Results of Laboratory Tests
Laboratory Test Result Reference Range
Antineutrophil cytoplasmic negative Negative
antibody (ANCA)
C-reactive protein 9.7 mg/dL 0.8 mg/dL or less
Creatinine, serum 1.0 mg/dL 0.7–1.5 mg/dL
Erythrocyte sedimentation rate 82 mm/hr Female: 0–20 mm/hr
(Westergren)
Leukocyte count 9100/μL 4000–11,000/μL
Eosinophils 35% 0-3%
Urinalysis Normal Negative

Click here for references

Which of the following disease manifestations is the most likely to increase the risk
of mortality in this patient?

A. Bowel perforation

B. Glomerulonephritis

C. Myocarditis


D. Pulmonary hemorrhage

Rationale

This patient is suffering from eosinophilic granulomatosis with polyangiitis


(EGPA), formerly Churg-Strauss syndrome. Cardiac involvement is common in
EGPA, occurring in 15% to 60% of patients. Several small studies suggest that
cardiac involvement accounts for nearly 50% of deaths from EGPA. Risk factors
for cardiac disease include high eosinophil counts at diagnosis and negative
ANCA testing. Myocarditis leading to cardiomyopathy and arrhythmias,
pericarditis, and valvular lesions (which are frequently asymptomatic) are the
most common cardiac manifestations of EGPA. Heart block, coronary vasculitis,
and intraventricular thrombosis are rare but serious manifestations.
Gastrointestinal vasculitis can occur in EGPA leading to bowel perforation but is
a rare manifestation. This patient has no symptoms/signs of gastrointestinal
involvement, which is seen in about 20% of patients with EGPA. Eosinophilic
esophagitis, gastroenteritis, and colitis are more common gastrointestinal
manifestations of EGPA.
Renal involvement, manifesting as hematuria and/or proteinuria, occurs in
approximately one-quarter of patients with EGPA. Of these, less than half are
found to have glomerulonephritis (other manifestations include eosinophilic
interstitial nephritis, mesangial glomerulonephritis, and focal sclerosis). The
majority of patients with biopsy proven glomerulonephritis are ANCA positive,
which is not the case in this patient. Additionally her renal studies are normal.
Although asthma, nonhemorrhagic pulmonary infiltrates, and fibrosis are
common in EGPA, pulmonary hemorrhage is a rare but serious manifestation of
EGPA seen in less than 5% of patients. She has no evidence of pulmonary
symptoms, and her examination is completely normal.
Educational Objective: Recognize myocarditis/cardiomyopathy as a significant
clinical manifestation in eosinophilic granulomatosis with polyangiitis

Case 40
A 20-year-old autistic man presents for nonspecific myalgias and arthralgias. He
has developed bilateral pain in his ankles, knees, and calves, with bruising and a
rash noted on the calves. No weakness or neurologic complaints are present.
On examination, he is afebrile with a blood pressure of 110/70 mm Hg and heart
rate of 80 bpm. The heart, lung, and abdominal examination are within normal
limits with no abnormalities appreciated. The joints are not swollen or inflamed
but are painful with active range of motion. The skin has multiple petechial
lesions with perifollicular hemorrhages (Figure).
Figure. Petechial Lesions With Perifollicular Hemorrhage

Laboratory testing reveals a normal complete blood count with differential,low


albumin, and low serum cholesterol.

Click here for references

Which of the following laboratory studies would you order to confirm the diagnosis?

A. Vitamin A

B. Vitamin B12

C. Vitamin C

D. Vitamin D

Rationale

The following patient is suffering from scurvy as a result of low vitamin C levels
from dietary imbalances. Vitamin C has important biological significance in
humans. It is a cofactor in many enzymatic reactions and in collagen synthesis.
Deficiencies in vitamin C can cause alterations in collagen structure and
increased bone resorption. Patients may present with musculoskeletal
manifestations of arthralgia, myalgia, and hemarthrosis. Other
nonmusculoskeletal manifestations include petechial lesions, corkscrew hairs,
purpura, and bleeding. Laboratory studies will show a low vitamin C level with
improvement in symptoms following vitamin C supplementation.
Vitamin A deficiency would manifest as visual complaints ranging from night
blindness and complete blindness to stages of xerophthalmia. Vitamin A
deficiency is rare in industrialized countries around the world but is still the third
most common nutritional deficiency worldwide.
Vitamin B12 deficiency can also be caused by nutritional deficiencies as can
vitamin C deficiency, but the complete blood count with differential would show
severe anemia and macrocytosis. In addition, patients can present with
neurologic manifestations ranging from weakness and paresthesias to dementia.
No neurologic complaints or hematologic abnormalities were noted in this
patient.
Vitamin D deficiency can be seen with musculoskeletal complaints such as bone
pain and tenderness; however, it would not explain the cutaneous manifestations
seen in this patient. Vitamin D deficiency can also cause secondary
hyperparathyroidism and osteomalacia.
Educational Objective: Recognize signs and symptoms of scurvy

Case 41
A 65-year-old woman presents to her ophthalmologist complaining of decreased
vision. She has a past medical history of rheumatoid arthritis, diabetes mellitus,
chronic renal insufficiency, and breast cancer. Her height is 65 in (165.1 cm) and
weight is 121.3 lb (55 kg). Her medications are hydroxychloroquine 400 mg daily,
methotrexate 10 mg weekly, sulfasalazine 1000 mg twice daily, prednisone 2.5
mg daily, metformin 500 mg daily, and tamoxifen 20 mg daily.
On visual field testing, she is noted to have partial ring scotomas mainly involving
the parafoveal region. On spectral-domain optical coherence imaging, she is
noted to have parafoveal thinning of the outer retina and loss of photoreceptor
outer segment marker lines. Based on these findings, the opthalmologist
suspects retinal toxicity resulting from hydroxychloroquine use and contacts her
rheumatologist, who immediately discontinues her treatment with the medication.
Click here for references

Which other medication may have contributed to her retinal toxicity?

A. Metformin

B. Methotrexate

C. Sulfasalazine

D. Tamoxifen

Rationale

This patient has several critical reasons for retinal toxicity from
hydroxychloroquine (HCQ). Tamoxifen is a retinal toxin of its own
accord. Although rare at current doses, toxicity was seen early in the drug’s use,
and at higher doses than used today. In a recent study evaluating HCQ retinal
toxicity in patients with lupus, those who had concurrent tamoxifen therapy for
breast cancer were at greatly increased risk of developing retinal toxicity.
Furthermore, retinal toxicity correlated with greater cumulative tamoxifen intake.
Retinal toxicity has not been reported with the use of metformin, methotrexate
(MTX) or sulfasalazine (SSZ). The combinations of MTX, HCQ, and SSZ are
used commonly as “triple therapy” in rheumatoid arthritis. This patient has other
critical risk factors for the development of HCQ retinal toxicity that deserve
mention. First, the kidneys are the main mechanism for clearance for
HCQ. Decreased renal function leads to a higher serum concentration, and
kidney disease has been reported to markedly increase the risk of retinal toxicity.
A drop in kidney function by 50% leads to an approximate doubling of the risk of
retinopathy. Although no guidelines for dosing adjustment currently exist, it is
prudent to adjust the dose of HCQ in patients with renal insufficiency and
vigilantly monitor for retinal toxicity.
Secondly, HCQ should be dosed at less than 6.5 mg/kg of ideal body weight. A
recent study looking at HCQ use found that patients with a mean daily use
exceeding 5.0 mg/kg of actual body weight had approximately a 10% risk of
retinal toxicity within 10 years of treatment and an almost 40% risk after 20 years.
Patients using an intermediate amount of 4.0 mg/kg to 5.0 mg/kg had a risk of
less than 2% within the first 10 years of use but almost 20% risk after 20 years.
Thus, the authors recommended using less than 5.0 mg/kg of actual body weight
as an appropriate dosing alternative to 6.5 mg/kg ideal body weight and stressed
the importance of vigilant screening with a longer duration of treatment. Based on
either dosing regimen, this patient’s daily dose was too high, putting her at risk of
retinal toxicity. Newer screening methods for retinal toxicity may be more
sensitive, and the implications of these findings are yet to be determined.

Case 42
A 35-year-old woman presents with diffuse pain involving the neck, shoulders,
elbows, low back, and knees lasting most of the day. She reports fatigue and
frequent depression. She does not sleep well and wakes frequently because of
pain. She has tried ibuprofen 600 mg 3 times daily and naproxen 500 mg twice
daily in the past without benefit. She has used nortriptyline 25 mg nightly for 6
months but now reports worsening diffuse pain with depression and increased
anxiety at her job, manifesting as increased frustration with customers. Her past
medical history includes migraine headaches. She is married and works in the
retail setting selling women’s clothing. She drinks no alcohol now but had
significant alcohol abuse in college causing hospitalization. She has been sober
since that time.
Her review of systems is negative for skin rashes, fever, weight loss, dyspnea,
and chest pain. She is afebrile, and vital signs are normal. She has widespread
positive tender points bilaterally at the suboccipital, mid-trapezius, and
paralumbar muscles, medial elbows, and medial inferior knees. All joints have
normal range of motion. No joint swelling is noted. Laboratory tests reveal normal
thyroid stimulating hormone, complete blood count, liver function tests, and
electrolytes.
Click here for references

What is the next best pharmacologic treatment for this patient?

A. Duloxetine 30 mg daily

B. Hydrocodone 5 mg/acetaminophen 325 mg twice daily

C. Nabumetone 750 mg twice daily with food

D. Tramadol 50 mg 4 times daily

Rationale

This patient has definite widespread pain of fibromyalgia (FM) without another
diagnosis of rheumatoid arthritis, osteoarthritis, or a metabolic disorder. FM is
considered a central pain augmentation syndrome with associated altered pain
processing; therefore, medications aimed at reducing this pain and improving
sleep such as tricyclic antidepressant medications (eg, nortriptyline) are
therapeutic. If tricyclic antidepressants are not tolerated and/or not helpful,
anticonvulsants such as gabapentin and pregabalin can help sleep and pain and
are nonhabituating. She has anxiety and depression with a new sleep
disturbance. Therefore, a serotonin and norepinephrine reuptake inhibitor such
as duloxetine, which can improve depression and pain, can be extremely helpful
as well.
Narcotics do not have a role in the treatment of FM because they do not address
the origins of the pain, are habituating and possibly addictive, and do not help
depression, which can often coexist in FM, and in fact can exacerbate
depression. Also, in this particular patient with a history of alcohol abuse,
narcotics can be especially problematic and addictive.
Nonsteroidal anti-inflammatory drugs (NSAIDs) can help primary arthritis pain of
osteoarthritis and rheumatoid arthritis, among other conditions, because there is
an inflammatory component, but they do not help the soft tissue pain of FM,
which is primarily noninflammatory. Although NSAIDs can help patients with FM
and degenerative joint disease, NSAIDs do not address the nociceptive pain of
FM. Since this patient does not have arthritis pain, a medication like nabumetone
will not help, as ibuprofen and naproxen did not help.
Modest evidence supports the efficacy of tramadol as an analgesic in the
treatment of FM. Unlike duloxetine, it would not be expected to potentially
improve this patient’s mood disorder. The concominant use of tramadol and
tricylic antidepressants is associated with an increased risk of serotonin
syndrome.
Educational Objective: Review pharmacologic therapies for fibromyalgia and
understand importance of tailoring treatment to appropriate symptom domain.

Case 43
A 7-year-old girl with no prior medical problems presented to your office 3
months ago with a 2-month history of polyarticular joint pain and swelling. At that
visit, she had swelling, warmth, tenderness, and loss of range of motion of her
wrists, 2nd and 3rd metacarpophalangeal joints, elbows, knees, and ankles. A
latex test for rheumatoid factor is positive. Erosions of carpal bones were present
on radiographs of the wrists. You started the patient on a daily nonsteroidal anti-
inflammatory drug, methotrexate, and a prednisone taper. At today’s visit, the
patient remains on prednisone 10 mg daily. Although her joint examination is
somewhat improved, she still has active arthritis in her wrists, elbows, knees, and
ankles.

Click here for references

What medication changes would be consistent with the 2011 American College of
Rheumatology treatment recommendations?

A. Add a tumor necrosis factor inhibitor and implement a steroid tapering


plan

B. Add a tumor necrosis factor inhibitor and maintain prednisone at 10 mg


daily

C. Add hydroxychloroquine and sulfasalazine and implement a steroid


tapering plan

D. Change the methotrexate to leflunomide and implement a steroid tapering


plan

Rationale
This patient has very active polyarticular arthritis with erosions on wrist
radiographs, which is a poor prognostic sign. Two of the important guiding
principles in JIA treatment recommendations from the American College of
Rheumatology and the Childhood Arthritis and Rheumatology Research Alliance
are: 1) in patients with ongoing moderately- to highly-active arthritis after 3
months on methotrexate, escalating therapy by adding a tumor necrosis factor
(TNF) inhibitor is recommended, and 2) long-term maintenance with oral steroids
is discouraged. Therefore, adding a TNF inhibitor and implementing a steroid
tapering plan is the correct answer. Etanercept and adalimumab are currently the
only TNF inhibitors that are approved by the US Food and Drug Administration
for the treatment of JIA.
Adding a TNF inhibitor but waiting until follow-up before implementing a steroid
tapering plan is incorrect. When arthritis is significantly active, treatment
recommendations suggest adding a TNF inhibitor at the 3-month follow-up visit.
However, treatment recommendations also encourage steroid tapering. Oral
steroids are used as sparingly as possible in children with polyarticular JIA.
Steroid side effects of particular concern for children are growth suppression and
skeletal fragility.
Adding hydroxychloroquine and sulfasalazine and implementing a steroid
tapering plan is incorrect. Unlike adult rheumatoid arthritis treatment plans, JIA
treatment plans do not recommend treatment with double or triple combination
disease-modifying antirheumatic drugs (DMARDs). Triple combination DMARDs
have not been extensively studied in comparative effectiveness trials for JIA.
Changing the methotrexate to leflunomide and implementing a steroid tapering
plan is also incorrect. When arthritis is significantly active, treatment
recommendations suggest adding a TNF inhibitor at the 3-month follow-up
visit. Treatment recommendations also encourage steroid tapering, so this part
of the answer is correct.
Educational Objective: Understand the consensus treatment plans for
polyarticular juvenile idiopathic arthritis, and recognize that long term treatment
with daily corticosteroids is not recommended.

Case 44
A 55-year-old man presents with severe Raynaud’s with recent early ulceration of
his second fingertips bilaterally. He also admits to worsening dyspnea on
exertion over the past 2 years, now after 1 block of walking on flat ground, and a
new nonproductive cough. He currently takes amlodipine 10 mg daily and
omeprazole 20 mg twice daily.
On examination, his blood pressure is 125/80 mm Hg, respiratory rate is 24
breaths per minute, and heart rate is 80 bpm. He is afebrile. The jugular venous
pressure is normal. He has tightening of the skin of the forearms and hands
bilaterally and upper chest wall. Dry gangrene is noted on the second fingertips
bilaterally. The chest examination reveals bilateral crackles a third of the way up
both lung fields. The abdominal examination reveals positive bowel sounds, no
hepatosplenomegaly, and no masses. The lower extremities have no edema.
Pulmonary function tests show decreased forced expiratory volume in 1 second,
and forced vital capacity is 55% of predicted consistent with restrictive disease.
Diffusing capacity of the lung for carbon monoxide is 50% of predicted. High-
resolution computed tomography reveals findings of early nonspecific interstitial
pneumonia. Results of laboratory tests are listed below (Table).
Table. Results of Laboratory Tests
Laboratory Test Result Reference Range
Anti-RNP antibodies Negative Negative
Anticentromere antibodies Negative Negative
Antinuclear antibodies 1:2560 Greater than 1:40 is abnormal
Anti-Scl-70 (topoisomerase 1) Positive Negative
antibodies
Hemoglobin, blood 11.4 g/dL Male: 14–18 g/dL
Leukocyte count 8200/μL 4000–11,000/μL

Over the next 6 weeks, further work-up reveals a normal echocardiogram


(normal ejection fraction and valves, no pericardial effusion), and a right heart
catheterization demonstrated mild elevation of pulmonary pressure.

Click here for references

Which of the following therapies is most likely to be of benefit for this patient’s
pulmonary disease?

A. Azathioprine

B. Bosentan

C. Cyclophosphamide

D. Methotrexate

Rationale

This patient has progressive scleroderma lung disease without pulmonary


hypertension. It is important in working up and treatment of scleroderma lung
disease that congestive heart failure, fluid overload, pulmonary emboli, recurrent
aspiration, and drug toxicity be ruled out before treating with immunosuppressive
therapy.
Data from several studies have shown modest benefits from intravenous and oral
cyclophosphamide (CYC) to slow the progression of scleroderma lung disease
and stabilize the decline in pulmonary function studies. In addition, some
observational studies suggest that mycophenolate mofetil may be helpful in
slowing scleroderma lung disease as well, but this therapy has not been studied
as extensively for this condition.
The other treatment choices are not correct. Azathioprine has been used as a
second line agent to CYC but does not have the efficacy of CYC. In one small,
unblinded, randomized controlled trial, azathioprine was not as efficacious as
CYC in preventing progression of interstitial lung disease (ILD). Bosentan is an
endothelin receptor antagonist that is beneficial in mild-to-moderate severity
pulmonary arterial hypertension but does not have a role in ILD of scleroderma.
Although low dose prednisone is used with CYC in the treatment of ILD, it is not
efficacious by itself. High dose prednisone is typically not used in ILD and
scleroderma partly because of the concern for potential precipitation of
scleroderma renal crisis. Methotrexate has shown no benefit in the treatment of
scleroderma lung disease and has also been associated with a small but
possible chance of lung inflammation and fibrosis.

Case 45
A 67-year-old man presents with a history of 3 episodes of painful lower
extremity swelling over the last 6 months. The first episode affected the right
knee with swelling that lasted for 1 to 2 days, the second involved the left knee
for a few days, and the third, which occurred last week, caused left ankle
swelling. He has had only minimal pain this week. In each case, oral nonsteroidal
anti-inflammatory drugs provided relief.
He has a history of hypertension treated with metoprolol. He denies any rashes,
ocular symptoms, diarrheal illnesses, dysuria, and any history of sexually
transmitted diseases. He does not smoke or drink. He has a family history of
psoriasis but no inflammatory arthritis or crystal disease.
The physical examination reveals an overweight man in no acute distress. His
blood pressure is 138/84 mm Hg, heart rate is 68 bpm, weight is 204 lb (93 kg),
and height is 67 in (170 cm). The musculoskeletal examination reveals a minimal
amount of erythema and tenderness over the left knee but no swelling. The left
ankle is normal. The remainder of the physical examination is normal. Results of
laboratory studies are listed below (Table).
Table. Results of Laboratory Tests
Laboratory Test Result Reference Range
Erythrocyte sedimentation rate 27 mm/hr Male: 0–15 mm/hr
(Westergren)
Rheumatoid factor 16 IU/ml Less than 24 IU/mL
(nephelometry)
Uric acid, serum 7.5 mg/dL 3.0–7.0 mg/dL

Radiographs of the knees and ankles are normal. A musculoskeletal ultrasound


of the left knee was performed, and the image demonstrates a hyperechoic
enhancement of the superficial aspect of the hyaline cartilage (Figure 1).
Figure 1. Musculoskeletal Ultrasound of Left Knee
Click here for references

Which of the following is the most likely diagnosis in this patient?

A. Gout

B. Pseudogout

C. Psoriatic arthritis

D. Reactive arthritis

Rationale

The diagnosis of gout is often made by evaluating findings from the medical
history, physical examination, serum uric acid test, radiography, and most
importantly joint aspiration for the presence of monosodium urate crystals. Some
patients, however, have a less obvious clinical presentation as noted in this case
(with only mild hyperuricemia). Musculoskeletal ultrasound is a technique that
can assist with the diagnosis of gout. This modality has the potential to detect a
pathognomonic finding such as the double contour sign, as seen in the image of
the knee (Figure 1), representing the deposition of microscopic urate (microtophi)
on most any articular cartilage surface. Ultrasound images can also identify
larger heterogeneous tophi, differentiating them from rheumatoid nodules.
Although a diagnosis of pseudogout could be clinically reasonable in this 67-
year-old patient, the typical finding of calcium-rich calcium pyrophosphate
dihydrate (CPPD) deposits in the middle of the femoral articular cartilage (and
not on the surface) are not seen in this case. The double contour sign would
also not be expected with CPPD.
Figure 2. Labeled Musculoskeletal Ultrasound of Left Knee
Psoriatic arthritis and reactive arthritis can present with an asymmetrical
oligoarticular arthritis, but in addition to lacking other clinical features of these
conditions, the double contour sign would not be expected on ultrasound in
psoriatic arthritis or reactive arthritis.
Of note, dual energy computed tomography (DECT) imaging can also be a useful
diagnostic tool for gout, although this is associated with radiation exposure and
greater cost than ultrasound.

Case 46
A 55-year-old woman presents with a 2-year history of slowly worsening left thigh
pain. Pain is elicited by walking and descending stairs. She drives to work for 20
minutes every day, and it is very painful for her to get out of her car. She denies
nocturnal pain or fever. She has diabetes mellitus and hypertension. On physical
examination, her body mass index is found to be 37 kg/m 2. The left hip has a
significantly reduced external and internal rotation with pain. Radiographic
examination of the left hip shows a nonuniform superolateral joint space
narrowing with new bone formation and subchondral bone sclerosis in the left
hip, a normal-shaped femoral head, and no enthesopathy. Knee radiographs are
normal. She would like to try nonpharmacologic measures rather than new
medications to treat her pain and functional limitation.
Click here for references

Which of the following nonpharmacologic therapies for this patient is best


supported by the literature?

A. Medially wedged insoles

B. Physical therapy

C. Tai chi

D. Weight loss

Rationale

The American College of Rheumatology (ACR) 2012 recommendations for the


use of nonpharmacologic therapies for hip osteoarthritis (OA) include
cardiovascular and/or resistance land-based exercise, aquatic exercise, and
weight loss for patients who are overweight. In addition, a recent systematic
review, consistent with other literature, concluded that bariatric surgery may
reduce joint pain in obese patients with hip and knee OA.
A varus insole is effective in decreasing the medial thrust and reducing pain in
knees with lateral compartment OA, but this has not been shown to be effective
for hip OA.
Limited evidence is available to support the use of physical therapy for hip OA. A
recent randomized, placebo-controlled trial involved 102 individuals, 50 years of
age or older, with hip osteoarthritis fulfilling the ACR criteria classification of pain
and radiographic changes. Physical therapy (in the form of manual therapy,
home exercise, and gait aid if appropriate) did not result in greater improvement
in pain or function compared with sham treatment.
Tai chi can effectively improve balance and aerobic capacity status in patients
with severe hip osteoarthritis but will not effectively alleviate the pain and
improve hip motion in these patients.
Case 47
A 37-year-old black woman presents for the treatment of worsening skin rash.
You have been seeing her for the last several years after she presented with a
malar rash, arthritis, and pleuritis, with a positive antinuclear antibody test and
elevated titers of anti–double-stranded DNA, anti-Ro, and anti-Sm antibodies.
She was initially treated with prednisone 40 mg daily, which was then tapered to
5 mg daily, and hydroxychloroquine 400 mg daily. A good clinical response was
achieved, and the patient has been serologically and clinically inactive for 2
years. Previous medical history was positive for sickle cell anemia, with a
baseline hemogloblin level of 10 g/dL (reference range [female]: 12–16 g/dL),
and hypersensitivity to multiple foods and medications.
Several months ago, she presented to your office with new onset of fatigue and a
worsening skin rash, which began after she self-discontinued steroids. Her blood
pressure today is 130/72 mm Hg, heart rate is 72 bpm, and weight is 137 lb (62
kg). A papulosquamous rash involving sun-exposed areas of the skin was
present on the face, upper chest, forearms, and lower legs. In addition, several
noticeable lesions were present on her scalp that showed peripheral
hyperpigmentation and central depigmentation, with scattered alopecia. The
current history and physical examination do not suggest active disease in any
other organ system except for the skin. Anti–double-stranded DNA antibodies
and C3 and C4 complement levels are normal.
Click here for references

What is the most appropriate treatment at this time?

A. Add chloroquine 200 mg daily

B. Add dapsone 150 mg twice daily

C. Add quinacrine 100 mg daily

D. Initiate intravenous belimumab 10 mg/kg at 0, 2, and 4 weeks and every 4


weeks thereafter

Rationale

This patient with systemic lupus erythematosus now presents with active disease
limited to the skin in the form of severe cutaneous lupus. She initially had a good
response to low dose steroids and hydroxychloroquine but experienced a
significant flare after she discontinued the prednisone.
Antimalarial agents are an excellent choice for treatment of cutaneous lupus
manifestations, with hydroxychloroquine usually being the first choice from this
class (which also includes quinacrine and chloroquine) among rheumatologists.
Once a patient fails hydroxychloroquine, addition of quinacrine is beneficial in the
majority of patients. Furthermore, addition of quinacrine to hydroxychloroquine
does not increase the risk of retinal toxicity. Actually, quinacrine is an
antiprotozoal agent rather than an antimalarial agent, but in common parlance
this regimen is referred to as a “double-antimalarial” treament. It is important to
note that quinacrine is expensive and has to be obtained through a compounding
pharmacy.
The other answer choices are incorrect. Although chloroquine can be combined
with quinacrine—the other theoretically possible “double-antimalarial” regimen—
chloroquine and hydroxychloroquine is an unacceptable combination due to
higher risk of retinopathy.
The patient may respond to initiation of dapsone. However, this drug is
associated with a decrease in hemoglobin and would not be the first choice in a
patient with existing anemia. Furthermore, the usual dose is 25 mg to 150 mg
daily.
Belimumab blocks the B lymphocyte stimulator/B cell-activating factor
(BLyS/BAFF) pathway and can be helpful in cutaneous disease in lupus patients.
However, benefits of belimumab are primarily seen in patients with low
complement and high anti–double-stranded DNA antibodies, which this patient
lacks. Response rates to belimumab may also be lower in black patients, as in
this case.
Educational Objective: Recognize potential role for double antimalarial therapy
with quinacrine and hydroxychloroquine for persistently active cutaneous lupus.

Case 48
A 61-year-old woman with anticyclic citrullinated peptide positive rheumatoid
arthritis on methotrexate and etanercept presents with progressive dyspnea over
several months. She has no history of tobacco use.
On examination, her blood pressure is 128/72 mm Hg, her heart rate is 86 bpm,
and her oxygen saturation is 94% on room air. Her examination is remarkable for
inspiratory fine crackles at the lung bases and mild edema at her 2nd and 3rd
metacarpophalangeal joints bilaterally. Recent laboratory testing showed normal
complete blood count, creatinine level, and liver function tests.
A chest radiograph showed increased peripheral reticular infiltrates at the lung
bases. Pulmonary function testing revealed a decreased diffusing capacity of the
lung for carbon monoxide (DLCO) at 46% of the predicted value.
Click here for references

What other findings on pulmonary function testing are most characteristic with this
patient’s presentation?

A. Decreased FVC, decreased FEV1/FVC ratio, decreased TLC

B. Decreased FVC, decreased FEV1/FVC ratio, increased TLC

C. Decreased FVC, normal FEV1/FVC ratio, decreased TLC

D. Normal FVC, normal FEV1/FVC ratio, normal TLC

Rationale

In this patient with seropositive rheumatoid arthritis, reduced oxygenation on


room air, crackles on inspiration, and basilar reticular opacities on chest
radiography are most consistent with interstitial lung disease as the cause of her
progressive dyspnea. Pulmonary function tests are useful in diagnosis, grading
disease severity, and evaluating therapeutic response. The DLCO in this patient
is abnormal. Measurement of DCLO uses carbon monoxide to assess the ability
of the lungs to transfer gas into the circulation. It can be reduced by a decrease
in alveolar capillary membrane permeability as seen in interstitial lung disease or
pulmonary hypertension. However, it can also be reduced by a decrease in the
alveolar-capillary membrane surface area (emphysema or chronic pulmonary
embolism) or by an inability of the blood to accept oxygen (anemia). Therefore,
other pulmonary function tests are needed to diagnose an interstitial lung disease
pattern.
Pulmonary function testing also includes spirometry, which measures the rate of
air flow from the lungs. FVC and the FEV1/FVC ratio are useful measurements to
separate a restrictive process such as interstitial lung disease from an obstructive
process such as chronic obstructive pulmonary disease (COPD) or asthma. FVC
is the total lung volume that can be expired after a maximal inspiration. FEV1 is
the volume of air expired in the first second after maximal expiration. In a
restrictive disease, loss of lung volume can lead to a decrease in both FVC and
FEV1, but the loss is proportional, leading to a normal FEV1/FVC ratio. In
contrast, in an obstructive process, these values are abnormal due to flow
restriction. Although FEV1 and FVC are also decreased, FEV1 is
disproportionately affected, leading to a decreased FEV1/FVC ratio.
Pulmonary function testing can also measure lung volumes, including residual
volume (RV, volume left after maximal expiration), functional residual capacity
(FRC, volume left after normal [tidal] expiration), and total lung capacity (TLC,
volume of air in the lungs at maximal inhalation). Restrictive processes, such as
interstitial lung disease, are defined by loss of lung volume. Loss of RV is the
most sensitive indicator, but the severity of a restrictive process is graded by the
degree of loss in TLC. Nonrestrictive processes generally result in normal lung
volumes or even increased lung volumes, if air-trapping is present from an
obstructive process. Therefore, the most characteristic pattern for this patient
with connective tissue interstitial lung disease, in addition to a decreased DLCO,
is a decreased FVC, normal FEV1/FVC ratio, and a decreased TLC.
Decreased FVC, decreased FEV1/FVC ratio, and decreased TLC is a pattern
most consistent with a mixed obstructive restrictive process. The decreased
FEV1/FVC ratio indicates an obstructive process. The decreased TLC indicates a
restrictive process. Examples of mixed processes would include a patient with
emphysema that develops a neuromuscular disorder or a patient with sarcoidosis
with both endobronchial and interstitial disease involvement.
Decreased FVC, decreased FEV1/FVC ratio, and increased TLC is a pattern
most consistent with emphysema. A decreased FEV1/FVC ratio indicates an
obstructive process such as COPD or asthma. An increased TLC indicates air-
trapping. Loss of DLCO is typically seen in emphysema but not in other forms of
COPD and is secondary to the loss of alveolar capillary membrane surface area.
Normal FVC, normal FEV1/FVC ratio, and normal TLC is a pattern most
consistent with a loss of gas transfer from a process that does not involve the
lung parenchyma, such as pulmonary hypertension or anemia. Early interstitial
lung disease can also present with an isolated decrease in DCLO. However, in
this patient, this pattern would be less typical, as chest radiography findings
indicate more advanced disease. Table 1 provides a summary of pumonary
function test findings reviewed in this question.
Table 1. Summary Pulmonary Function Test Findings
Pattern DLCO FVC FEV1/FVC TLC Disease Examples
ratio

Restrictive Low Decreased Normal Decreased Interstitial lung disease

Obstructive with Low Decreased Decreased Increased Emphysema


air trapping

Mixed restrictive Low Decreased Decreased Decreased Sarcoidosis with


obstructive endobronchial and
interstitial involvement

Loss of gas Low Normal Normal Normal Pulmonary hypertension,


transfer early interstitial lung disease

Educational Objective: Understand interpretation of pulmonary function tests in


interstitial lung disease

Case 49
A 44-year-old woman presents to the rheumatology clinic complaining of pain
and swellng in both knees and ankles for 3 to 4 weeks. Physical examination
showed normal knees with swelling and tenderness in both ankles and dorsal
feet and painful erythematous nodules on the pretibial aspects of the lower
extremities. Results of laboratory tests are listed below (Table).

Table. Results of Laboratory Tests


Laboratory Test Result Reference Range
Aminotransferase, serum alanine 60 U/L 10–40 U/L
(ALT, SGPT)
Aminotransferase, serum 37 U/L 10–40 U/L
aspartate (AST, SGOT)
Anticyclic citrullinated peptide, Negative Less than 20 units
antibodies to
Antinuclear antibodies 1:160 (speckled Greater than 1:40 is
pattern) abnormal
Complements, serum
C3 144 mg/dL 100–233 mg/dL
C4 34 mg/dL 14–48 mg/dL
C-reactive protein 4.6 mg/dL 0.8 mg/dL or less
Erythrocyte sedimentation rate 39 mm/hr Female: 0–20 mm/hr
(Westergren)
Hemoglobin, blood 11.4 g/dL Female: 12–16 g/dL
Leukocyte count 8600/μL 4000–11,000/μL
Platelet count 290,000/μL 150,000–300,000/μL
Rheumatoid factor Negative

Findings of knee, ankle, hip, and hand radiographs show periarticular


osteopenia. Chest radiography findings show focal mediastinal widening in both
the hilum and the right upper trachea, and a chest computed tomography scan
indicates multiple enlarged lymph nodes of the mediastinum and both hila. A skin
biopsy of one of the nodules over the lower legs shows an inflammatory infiltrate
of predominantly lymphocytes and macrophages of the septa between the fat
lobules. The inflammatory infiltrate spreads into fat lobules.

Click here for references

Which of the following is the most likely diagnosis?

A. Lupus profundus (lupus panniculitis)

B. Löfgren syndrome

C. Lupus tumidus

D. Weber-Christian disease

Rationale

Erythema nodosum (EN) is the most common type of panniculitis. It is


characterized histologically by an inflammatory infiltrate of the fibrous septa
between fat lobules (septal panniculitis). It is recognized as a reactive disorder to
a variety of stimuli. Streptococcal infection is the most common precipitant of EN
in children, whereas acute sarcoidosis (Löfgren syndrome), drugs, and
inflammatory bowel disease are the most common causes in adults. EN typically
presents in young women with tender warm nodules and plaques on the shins
that change to a color that is similar to deep bruises and resolve without
ulceration, atrophy, or scarring. Commonly associated symptoms include fever,
fatigue, and malaise. Löfgren syndrome is a form of acute sarcoidosis that is
characterized by the presence of hilar adenopathy, EN, and arthritis and/or
periarthritis, usually involving lower extremities.
Lupus panniculitis (also known as lupus profundus) is much less common than
EN and is characterized by erythematous nodules in other locations, such as the
upper arms, shoulders, face, scalp, and buttocks. The deep lesions are often
associated with overlying lesions of discoid lupus erythematosus. Histopathologic
examination reveals a lobular panniculitis.
Lupus tumidus is characterized by urticarial plaques and nodules that are
pink/violaceous and are usually photo-distributed. Pathologically, a dense,
superficial and deep, perivascular, lymphocytic infiltrate is seen, without evidence
of panniculitis.
Weber-Christian disease is a term used to describe an idiopathic lobular
panniculitis associated with fever, polyarthralgias, and painful subcutaneous
nodules. These nodules may ulcerate or scar and may be associated with fat
necrosis. This condition was once thought to be a distinct entity. However, this
term has largely been abandoned by dermatologists with the recognition of other
more specific types of panniculitis.
Educational Objective: To recognize features of acute sarcoidosis

Case 50
A 50-year-old man presents with a 1-year history of bilateral shoulder pain. He is
paraplegic for the past 2 years and uses a manual wheelchair for mobility. His
pain is exacerbated when transferring in and out of the wheelchair, combing his
hair, or lying on his side at night. He has a mild sensation of weakness with
lifting that has been exacerbated after completing a 10-km race 1 week ago.

Click here for references

Which physical examination maneuver is most likely to be abnormal/positive?

A. Acromioclavicular joint compression

B. Bicipital groove palpation


C. Jobe's/"empty can" test

D. O'Brien/resisted arm extension during adduction and internal rotation

Rationale
Rotator cuff tears have been found by magnetic resonance imaging (MRI) to be 4
times more likely in paraplegic, wheelchair-bound patients than in age-matched
controls, with up to 61% suffering supraspinatus tears. Rotator cuff pathology is
also the most common shoulder pathology identified by phyisical examination in
manual wheelchair users. The Jobe’s/"empty can” test puts the supraspinatus
tendon under strain and is used to diagnose tears in the supraspinatus tendon.
The Jobe’s test is performed with the arm in 90 degrees of forward flexion and
full internal rotation with the thumb pointing down as if emptying a beverage can.
The examiner applies downward pressure on the superior aspect of the distal
forearm, and the patient resists. The empty can test is consired positive if there is
significant pain or weakness. When weakness is used to define a positive test, it
is 76% to 95% specific and 57% to 74% sensitive.
Although rotator cuff pathology is the most common shoulder problem in manual
wheelchair users, MRI evidence of acromioclavicular (AC) and glenohumeral
joint arthritis is also more likely to be present in paraplegic patients than in age-
matched controls. The AC joint compression test is performed with the patient
lying supine with the involved arm related at the side. The examiner stands on
the involved side and places one hand on the patient’s clavicle and the other
hand on the spine of the scapula. The examiner gently squeezes the hands
together. Pain or movement of the clavicle define a postive test indicating either
AC joint arthritis or sprain of the AC ligament or coracoclavicular ligament.
O’Brien’s labrum test is meant to evaluate for glenoid labrum tear. It is performed
by adducting the arm 10 degrees and resisting arm extension while the arm is in
a position of internal rotation compared with that in external rotation. Greater pain
with internal rotation than external rotation is a positive test. The sensitivity and
specificity of the test is 63% and 73%, respectively. It is less commonly positive
than the Jobe’s/“empty can” test in wheelchair-bound patients due to paraplegia.
Bicipital groove tenderness on physical examination can indicate bicipital
tendinitis, tendinopathy, and impingement. Bicipital pathology is less commonly
found by ultrasonography and MRI in paraplegic wheelchair users than rotator
cuff disease.

Case 51
A 53-year-old woman presents to establish care. She has a history of
granulomatosis with polyangiitis and completed a course of rituximab therapy 4
months before this visit. Current medications include prednisone 10 mg once
daily, double-strength trimethoprim-sulfamethoxazole 160 mg/800 mg 3 times
weekly, calcium 600 mg/ vitamin D 400 IU twice daily, and alendronate 70 mg
once weekly. She reports being compliant with her medications and has been
getting routine blood work monthly. Results are listed below (Table).

Table. Results of Laboratory Tests

Date Laboratory Test Result Reference Range


3 months ago Absolute neutrophil count 4500/µL 2000-8250/µL
1 month ago Absolute neutrophil count 500/µL 2000-8250/µL
Today Absolute neutrophil count 3000/µL 2000-8250/µL

Click here for references

Which of the patient’s medications is most likely to have caused the laboratory
results in the table provided?

A. Alendronate

B. Prednisone

C. Rituximab

D. Trimethoprim-sulfamethoxazole

Rationale

Late onset neutropenia has been observed in up to 5% of patients receiving


rituximab for autoimmune disorders. Late onset neutropenia occurs at least 4
weeks after completion of a rituximab cycle. Case series have noted the onset to
occur 6 to 52 weeks after rituximab therapy and to coincide with a period of B cell
depletion. Serum immunoglobulin M levels have also been noted to decrease
during the same time period.
Late onset neutropenia may be associated with an increased risk of infection or
observed as an isolated lab finding. The neutropenia responds to treatment with
granulocyte colony-stimulating factor or can spontaneously resolve. It can take
between 3 days and 52 weeks for the neutrophil count to recover without
intervention. Bone marrow biopsies reveal white cell line maturation arrest with
an increased number of lymphocytes without the presence of blasts. In a case
series of 209 patients treated with rituximab for autoimmune diseases, 11
patients developed late onset neutropenia. Among the 11 patients, 5 received a
second course of rituximab. One patient developed a second episode of late
onset neutropenia. In a second series of patients, 8 patients developed late onset
neutropenia (6% of patients receiving rituximab). Of the 6 patients rechallenged
with rituximab after resolution of neutropenia, none developed a second episode
of late onset neutropenia. In patients who have experienced prior episodes of
late onset neutropenia, a sustained hypogammaglobulinemia either induced or
aggravated by rituximab may increase the risk of infections.
The other treatment choices are incorrect. Transient leukopenia and fever have
been reported with intravenous use of bisphosphonates, but neutropenia has not
been reported with the use of oral alendronate. Prednisone is associated with
immunosuppression and increased risk of infection but not neutropenia
specifically. Trimethoprim-sulfamethoxazole is associated with agranulocytosis
and neutropenia. However, the neutropenia will not improve without
discontinuation of the medication.
Educational Objective: Recognize risk of neutropenia with ritixumab therapy.
Romosozumab is a humanized monoclonal antisclerostin antibody, which has been
shown to improve the bone mineral density in postmenopausal women in phase 2
studies. Romosozumab exerts its effects on bone remodeling by preventing
sclerostin from doing which of the following?

A. Binding to LRP5/6 and blocking Wnt signaling pathway resulting in bone


formation

B. Binding to miR-34a resulting in suppression of osteoclastogenesis

C. Inducing expression of Bruton's tyrosine kinase, thus inhibiting osteoclast


maturation

D. Reducing the expression of osteoclastogenic transcription genes c-fos and


Nfatc1

Rationale

Romosozumab is a monoclonal antibody against sclerostin and is administered


subcutaneously at intervals of 1 month or 3 months over a period of 12 months. It
has been shown to transiently increase the markers of bone formation and
moderate sustained decrease in bone resorption and increase in bone mineral
density at both hip and spine. The rise in bone mineral density was greater with
romosozumab than with placebo, alendronate, or teriparatide. The mechanism of
action of sclerostin is to decrease bone formation through inhibition of the Wnt
signaling pathway. However, its precise molecular mechanism of action and
factors controlling its secretion are not completely understood. Animal studies
have shown that mechanical loading and high parathyroid hormone levels
downregulate the expression of the gene encoding for sclerostin (SOST) in
osteocytes and decrease the production of sclerostin, which would result in
stimulation of bone formation. Romosozumab prevents sclerostin from binding to
LRP5/6 and blocking Wnt signaling pathway resulting in bone formation. LRP5 is
a transmembrane low-density lipoprotein receptor that acts as a co-receptor with
“Frizzled” protein family, which is a family of G protein-coupled receptor proteins
that serve as the receptors in the Wnt signaling pathway and play a key role in
skeletal homeostasis.
MicroRNAs have now been recognized as cell function regulators. miR-34a has
been recognized as a novel suppressor of osteoclastogenesis and bone
resorption. miR-34a is downregulated during osteoclast differentiation.
Osteoclastic miR-34a-overexpressing transgenic mice exhibit lower bone
resorption and higher bone mass. Transforming growth factor-beta-induced
factor 2 (Tgif2), which promotes osteoclastogenesis, has been noted to be an
essential direct miR-34a target.
Bruton’s tyrosine kinase (Btk) is expressed in B lymphocytes and myeloid lineage
and osteoclast lineage cells and links receptor activator of nuclear factor kappa-B
(RANK) with the immunoglobulin-like receptors contributing to osteoclast
differentiation. The orally administered Btk inhibitor, ibrutinib, binds covalently to
a cysteine residue in the active site of Btk and inhibits its activity irreversibly.
Ibrutinib also downregulates the expression of nuclear factor of activated T cells
(Nfatc1), the key transcription factor for osteoclastogenesis, and disrupts the
formation of the actin ring in mature osteoclasts. It has been shown that ibrutinib
administration decreased bone loss that developed in a RANK ligand (RANKL)-
induced osteoporosis mouse model.
Delphinidin, one of the major anthocyanidins in berries, is believed to have
significant benefits in prevention of osteoporosis through the suppression of
osteoclast formation. Delphinidin suppressed the activity of nuclear factor kappa-
B, c-fos, and Nfatc1, which are important transcriptional factors for
osteoclastogenesis.
Educational Objective: Recognize the mechanism of action of romosozumab,
an emerging therapy for osteoporosis

Case 53
A 65-year-old man with a 2-year history of seronegative inflammatory arthritis
with migratory arthritis of the large joints presents for clinic follow-up with a
weight loss of 15 Ib (6.8 kg) and chronic diarrhea for the past 3 months. He has
been treated for the past 2 years with prednisone 5 mg daily and methotrexate
20 mg orally once weekly. He undergoes an endoscopy and colonoscopy for the
chronic diarrhea and is found to have extensive periodic acid schiff (PAS)-
positive material in the lamina propia on small bowel biopsy with villous
atrophy. Methotrexate and prednisone are discontinued, and therapy with
parenteral ceftriaxone for 2 weeks then oral trimethoprim-sulfamethoxazole is
begun. Three weeks into treatment with antibiotics the patient develops high
fevers and swelling of both knees and ankles.
On physical examination, temperature is 101.2°F (38.4°C), heart rate is 90 bpm,
and blood pressure is 134/65 mm Hg. Bilateral moderate-sized effusions, with
warmth and tenderness to touch, are noted on the knees. Mild swelling is noted
on both ankles, which have intact range of motion but tenderness upon
ambulation.
Click here for references

Which of the following is the next best step in the management of the patient?

A. Begin prednisone

B. Continue antibiotics and add metronidazole

C. Discontinue trimethorprim-sulfamethoxazole and restart intravenous


ceftriaxone


D. Repeat endoscopy and biopsy

Rationale

Tropheryma whipplei infection is the source of Whipple disease, which typically


presents as a migratory, polyarthritis that may precede symptoms of diarrhea,
abdominal pain, and weight loss by several years. Diagnosis of Whipple disease
is confirmed in small bowel biopsy specimens, which reveal extensive PAS-
positive material in the lamina propria on pathologic examination. Treatment
consists of intravenous antibiotics in the form of ceftriaxone or meropenem for
several weeks followed by trimethoprim-sulfamethoxazole for up to a year.
The immune reconstitution inflammatory syndrome (IRIS) is defined as a flareup
of inflammatory signs and symptoms occurring after starting appropriate
antimicrobial therapy. IRIS has been reported after treatment of HIV, leprosy,
tuberculosis, Whipple disease, and opportunisitic infection in patients who have
undergone transplantation. In untreated Whipple disease, T. whipplei induces an
immunosuppressive effect on CD4+ T cells. With treatment, rapid immune
reconstitution results in a rapid proliferation of CD4+ T cells, which target affected
organs.
Several weeks after beginning therapy (generally 4-6 weeks, but up to 4 months
in cases of tuberculosis), patients may experience an apparent relapse of their
disease, which often manifests as a high fever and a return of joint pain, which is
characteristic of IRIS. Other manifestations of IRIS can include uveitis,
inflammatory orbitopathy, and intestinal perforation.
The treatment of choice in the setting of IRIS is prednisone, which helps relieve
the inflammation. Polymerase chain reaction of the small bowel tissue will be
negative in these patients. Continued antibiotic therapy as planned is
recommended, but switching of antibiotics will not result in improvement. IRIS
was seen in 10% of 142 patients treated for Whipple disease in two academic
centers in Germany. Previous use of immunosuppressive treatment was
correlated with development of IRIS.
Fever, abdominal pain, and the return of diarrhea might raise suspicion
for Clostridium difficile induced by antibiotic therapy, but the patient has fever and
joint pain and lacks other features of C. difficile infection warranting antibiotic
treatment. Discontinuation of trimethorprim-sulfamethoxazole and restarting of
intravenous ceftriaxone would be a reasonable course if the patient was believed
to have relapsed; however, the time course of treatment is not long enough to
consider relapse, and high fever would not be expected. Repeat endoscopy and
small bowel biopsy could remain positive after such a short time but would not
explain the sudden return of joint pain or associated high fever.
Educational Objective: Recognize features of the immune reconstitution
syndrome (IRIS) in Whipple disease.
Case 54
A 52-year-old woman with seropositive erosive rheumatoid arthritis (RA) presents
with increased pain, stiffness, and swelling over the last 3 months.
She has a past medical history of hypertension and melanoma in her right thigh,
treated 6 years ago with excision but no chemotherapy. On physical examination,
her vital signs are normal. She has no rash or nodules on the skin but has a well-
healed scar on the right thigh. No enlargement of the lymph nodes is noted. The
general examination is negative for significant findings. The musculoskeletal
examination shows active synovitis in multiple joints. Laboratory testing reveals a
normal complete blood count and comprehensive metabolic panel. Her
inflammatory markers are elevated. Her hand radiographs show an increase in
metacarpophalangeal joint erosions bilaterally when compared with results of
previous radiographs.
Her disease has remained active despite the use of nonbiologic disease-
modifying antirheumatic drugs in combination, and a decision is made to escalate
her therapy to include a biologic agent. Screening for hepatitis B and C viruses
and tuberculosis are negative.
Click here for references

Which of the following biologic medications is recommended according to the ACR


2012 guidelines for treatment of her RA?

A. Abatacept

B. Adalimumab

C. Etanercept

D. Rituximab

Rationale
This patient has aggressive and erosive disease; therefore, according to the
most recent 2012 update of the American College of Rheumatology
recommendations for the use of disease-modifying antirheumatic drugs in the
treatment of RA (Class C Evidence), a change to a biologic medication is
indicated. Based on these recommendations, no biologic agent is excluded in
patients treated for a solid organ malignancy or nonmelanoma skin cancer more
than 5 years ago. However, the panel recommended using rituximab in patients
with previously treated solid organ or nonmelanoma skin cancer within 5 years or
with any history of melanoma or lymphoproliferative malignancy. Given the
history of melanoma, rituximab infusion is the recommended biologic
treatment. Little is known about the other agents in the setting of solid organ
tumors treated within 5 years because those patients were excluded from trials.
Educational Objective: Select the appropriate biologic thearpy in a patient with
rheumatoid arthritis and a history of malignancy.

Case 55
A 60-year-old man presents with decreased range of motion in the right hip. He
notes a long history of ankylosing spondylitis (AS), diagnosed at 31 years of age.
For the last several years, his symptoms have been well controlled with
adalimumab. Approximately 3 months ago, he had a fall and fractured his left hip.
He did very well initially, but for the last couple of months, he has had decreased
range of motion in the right hip, accompanied by mild pain. He notes no fever
and no pain elsewhere. He resumed his adalimumab 8 weeks postoperatively,
without any complications.
On physical examination, he is afebrile. Decreased range of motion with mild
pain is noted at the right hip. Flexion is limited to 90 degrees (normal: 125
degrees), and abduction and adduction are both limited to 30 degrees (normal:
45 degrees). Hyperextension is limited to 5 degrees (normal: 15 degrees). He
notes mild pain at maximal range of motion. The joint is not warm or visibly
swollen on examination. A radiograph is obtained, and shown below (Figure).
Figure. Radiograph of Right Hip
Click here for references

What is the best treatment that might have prevented this problem?

A. Cefazolin 1 hour before surgery then postoperatively every 8 hours for 24


hours

B. Adalimumab restarted 2 weeks postoperatively

C. Ibuprofen immediately after surgery then 3 times daily for 10 days


D. Irradiation of the joint 24 hours postoperatively then once daily for 10
days

Rationale

The radiograph shows heterotopic ossification, a complication seen in 5% to 30%


of patients after hip replacement surgery and in increased incidence among
patients with AS. Although most patients are asymptomatic, some have
decreased range of motion and pain. Patients with more severe cases may have
swelling, tenderness, and low grade fever, mimicking infection. Although routine
prophylaxis is not recommended, patients at high or moderate risk of heterotopic
ossification after hip arthroplasty should be considered for prophylaxis. High-risk
patients include those with a history of heterotopic ossification, bilateral
hypertrophic osteoarthritis of the hip, or post-traumatic arthritis characterized by
hypertrophic osteophytosis. Moderate-risk patients include those with AS, diffuse
idiopathic skeletal hyperostosis, Paget’s disease, or unilateral hypertrophic
osteoarthritis. Several trials have shown that postoperative prophylaxis with
moderate-dose nonsteroidal anti-inflammatory drugs can decrease the risk of
heterotopic ossification postoperatively.
Antibiotic prophylaxis with cefazolin would be a very reasonable choice, and this
is the appropriate frequency and duration, but it would have no impact on the
development of heterotopic ossification. This complication is not an
infection. Adalimumab, and other tumor necrosis factor inhibitors, have not been
shown to decrease the likelihood of heterotopic ossification. Postoperative
radiation, given between 24 and 72 hours after the operation, can also decrease
the risk of heterotopic ossification. However, only a single dose is necessary.
Treatment for 10 days would not further decrease the risk but would increase the
risk of complications from the radiation.

Case 56
A 70-year-old man returns to his rheumatologist for management of tophaceous
gout in the feet and olecranon bursae. His medications include febuxostat 80 mg
daily, colchicine 0.6 mg daily, and nifedipine 90 mg daily. Although he has not
had any further acute joint attacks, his tophi have not improved sufficiently and
his serum uric acid level remains greater than 6.0 mg/dL (reference range: 3.0–
7.0 mg/dL), despite a lengthy trial of febuxostat. He is allergic to allopurinol and is
not interested in pegloticase infusions.
His medical history includes hypertension, and his family history is significant for
multiple relatives with gout. His physical examination reveals a blood pressure of
160/80 mm Hg. His weight is 136 pounds (61.8 kg) and his height is 65 inches
(165.1 cm). He has a body mass index of 22.6 kg/m 2. The rest of his physical
examination is within normal limits except for bilateral olecranon bursal tophi and
numerous tophaceous desposits in his toes. Laboratory results from 2 weeks ago
are significant for a serum creatinine level of 1.1 mg/dL (0.7–1.5 mg/dL) and a
serum uric acid level of 6.6 mg/dL (reference range: 3.0–7.0 mg/dL).
Click here for references

Addition of which of the following antihypertensive medications may help decrease


the size of his tophi?

A. Atenolol

B. Hydrochlorothiazide

C. Lisinopril

D. Losartan

Rationale

Losartan’s uricosuric effect has been noted for many years. This effect is thought
to be mediated through inhibition of urate reabsorption in the proximal tubule and
may be transient. Interestingly, the other angiotensin receptor blockers do not
seem to have a similar uricosuric effect. A recent retrospective case-control study
has suggested that use of losartan rather than beta-blockers, angiotensin-
converting-enzyme (ACE) inhibitors, and thiazide diuretics for hypertension is
associated with a decreased risk of incident gout. In this patient, replacing
febuxostat with pegloticase might be the next option if he was willing to try it.
A recent retrospective case-control study has suggested that the use of beta
blockers may actually cause an increased risk of gout. Although thiazide diuretics
have been implicated less often than loop diuretics in causing hyperuricemia and
gout attacks, they still do not have a uricosuric effect or ability to reduce serum
uric acid levels. Another retrospective case-control study found that ACE
inhibitors increased the risk of gout.
Case 57
A 20-year-old woman presents with 4 months of joint pain and stiffness. She
reports morning stiffness for more than 1 hour and difficulty using her hands to
write and grip objects. She reports that these symptoms have affected her ability
to function as a college student. Her family history is notable for a maternal aunt
with rheumatoid arthritis (RA) and paternal grandmother with systemic lupus
erythematosus.
On physical examination, her blood pressure is 111/72 mm Hg, heart rate is 72
bpm, temperature is 98.6°F (37°C), and body mass index is 33 kg/m 2. The
examination is notable for bilateral swollen and tender 2nd and 3rd
metacarpophalangeal joints and left elbow with warmth and swelling and a
flexion contracture. No other joint findings are noted. No rash is present. The
Disease Activity Score in 28 joints with erythrocyte sedimentation rate (DAS28-
ESR) is greater than 5.1. Results of laboratory tests are listed below (Table).
Table. Results of Laboratory Tests
Laboratory Test Result Reference Range
Anticyclic citrullinated peptide, 50 units Less than 20 units
antibodies to
Antinuclear antibodies Less than 1:40 Greater than 1:40 is abnormal
C-reactive protein (high 30 mg/L Low risk = less than 1.0 mg/L
sensitivity), serum
Average risk = 1.0-3.0 mg/L
High risk = more than 3.0 mg/L
Erythrocyte sedimentation rate 49 mm/hr Female: 0–20 mm/hr
(Westergren)
Rheumatoid factor 350 IU/ml Less than 24 IU/mL
(nephelometry)

After contraception counselling, the patient is started on methotrexate (MTX)


monotherapy.
Click here for references

What is the approximate likelihood of this patient achieving a DAS28-ESR less than
3.2 on optimal MTX monotherapy at follow-up in 24 weeks?

A. 10%


B. 30%

C. 60%

D. 80%

Rationale

Until recently, studies comparing conventional disease-modifying antirheumatic


drug strategies with biologic therapy in RA had been lacking in both early and
established RA. In the Treatment of Early Rheumatoid Arthritis (TEAR) trial, 755
participants with early, poor-prognosis RA were randomized to receive MTX
monotherapy or combination therapy (MTX plus etanercept or MTX plus
sulfasalazine [SSZ]/hydroxychloroquine [HCQ]). Participants in the MTX
monotherapy group were stepped up to combination therapy (MTX plus
etanercept or MTX plus SSZ/HCQ) at week 24 if the DAS28-ESR was 3.2 or
higher. Importantly, at 24 weeks, approximately 30% (28%) of patients on initial
MTX monotherapy achieved DAS28-ESR less than 3.2 and did not require step
up to combination therapy.
Approximately 40% of the participants assigned to immediate combination
therapies (41% of participants in the immediate MTX plus etanercept group; 43%
of participants in the immediate triple-therapy group) achieved DAS28-ESR less
than 3.2 by week 24.
More than half (56%) of all participants in the TEAR trial achieved DAS28-ESR
remission (DAS28-ESR < 2.6) at some point during therapy. No statistically
significant difference was found between the 4 treatment arms in the proportion
achieving DAS28-ESR remission (56.6% in the immediate MTX plus etanercept
group; 59.1% in the immediate triple-therapy group; 52.9% in the step up from
MTX monotherapy to MTX plus etanercept group; 56.5% in the step up from
MTX monotherapy to MTX plus SSZ/HCQ group). Patients who switched from
initial combination therapy groups (MTX plus etanercept to triple and vice versa)
did equally well.
Current guidelines from the European League Against Rheumatism and revised
guidelines presented at the American College of Rheumatology (ACR) Annual
Meeting in 2014 support the use of MTX monotherapy in early and poor-
prognosis RA, with close monitoring (treat-to-target) strategies implemented for
patients not attaining acceptable disease control. According to the 2012 update
of the 2008 ACR reccommendations for the treatment of RA, features of RA
which portend a poor progosis include: functional limitation, extra-articular
disease (eg, nodules, vasculitis), positive rheumatoid factor or anticyclic
citrullinated peptide antibodies, and bony erosions by radiograph.

Case 58
A 28-year-old woman is admitted for evaluation of 6-week history of gradually
worsening chest pain, exertional dyspnea, episodes of low grade fever, edema,
and malaise. Additionally, she has noted left arm pain with repetitious activities
over the last few months. She denies cough and hemoptysis and has no history
of superficial phlebitis, thrombotic events, thrombocytopenia, recurrent
miscarriages, or any recent travel. She also denies dysphagia, abdominal pain,
hematochezia, oral and genital ulcers, uveitis, meningitis, and other neurologic
conditions. Her past medical history is negative for chronic illnesses.
On physical examination, the patient appears ill and pale. Her temperature is
100.8°F (38.2°C), blood pressure is 100/75 mm Hg in left arm and 130/85 mm Hg
in the right arm, and heart rate is 88 bpm. Peripheral pulses are palpable in the
lower extremities and right arm , but the left radial pulse is absent and she has a
left subclavian artery bruit. No skin rash or nail fold capillary are noted. The heart
and lung examinations are normal. The abdomen is benign and nontender,
without epigastric bruit or pulsation. A chest radiograph is unremarkable. The
echocardiogram shows mild tricuspid regurgitation, and the estimated pulmonary
artery pressure is 55 mm Hg. Pulmonary function tests reveal decreased
diffusing capacity of the lung for carbon monoxide and normal lung volumes.
Results of laboratory studies are listed below (Table). Blood cultures are
pending. A computed tomography (CT) scan of the chest is requested.

Table. Results of Laboratory Tests


Laboratory Test Result Reference Range
Antineutrophil cytoplasmic Negative Negative
antibody
Antinuclear antibodies Less than 1:40 Greater than 1:40 is abnormal
Antiphospholipid antibody panel Negative Negative
C-reactive protein 11.3 mg/dL 0.8 mg/dL or less
Creatine kinase, serum 3% of total Less than 5% of total
MB isoenzymes
Creatine phosphokinase, total Normal 10-120 µg/L
Erythrocyte sedimentation rate 90 mm/hr Female: 0–20 mm/hr
(Westergren)
Troponin I 0 ng/mL[µg/L] 0–0.1 ng/mL [μg/L]
Urinalysis Negative Negative
Click here for references
What is the most likely cause of this patient’s chest pain and dyspnea?

A. Acute respiratory distress syndrome

B. Chronic thromboembolic pulmonary hypertension

C. Pulmonary arteritis

D. Pulmonary capillaritis

Rationale

Takayasu’s arteritis (TA) usually involves the aorta and its branches, but
pulmonary arteritis (PA) and involvement of its branches has been reported. The
prevalence varies from 2% to 30% of patients in different studies. The pathologic
changes in TA were characterized by granulomatous panarteritis with adventitial
thickening and cellular infiltration of the tunica media and myofibroblast
proliferation leading to intimal hyperplasia, fibrosis, and stenosis.
Chest symptoms, such as thoracic pain, shortness of breath, palpitations, and left
ventricular dysfunction may mask underlying PA and lead to delay in diagnosis.
Central cyanosis and leg edema are late symptoms and generally represent
longstanding PA and heart failure due to pulmonary arterial hypertension (PAH).
The rate of PAH in TA patients is approximately 12% to 13% in various reports.
Patients with PAH due to PA had a poor prognosis and higher rates of death.
The diagnosis of TA is mainly based on a variety of imaging techniques. Low or
absent uptake on pulmonary perfusion scan appears to be a helpful and accurate
screening test. Conventional angiography remains the gold standard imaging tool
for diagnosis and evaluation of TA. CT or magnetic resonance angiography
studies are alternative diagnostic tools; they allow visualization of vessel wall
thickening, lumen narrowing, and/or dilation, which are all characteristic of
TA. Positron emission tomography-fluorodeoxyglucose can be used in diagnosis
of aortitis and TA as well as early acute phase of PA but is less useful in later
fibrotic stage.
Acute respiratory distress syndrome (ARDS) is an acute, diffuse, inflammatory
lung injury that leads to increased pulmonary vascular permeability. Clinical
hallmarks of ARDS are hypoxemia and bilateral opacities on chest radiography.
Underlying pathology in ARDS is diffuse alveolar edema with or without focal
hemorrhage, acute inflammation of the alveolar walls, and hyaline
membranes. The protracted course, presence of subclavian bruit, and normal
chest radiograph are not consistent with the classic ARDS presentation, the
hallmark of which is hypoxemia and bilateral radiographic opacities. CT scan of
the chest usually reveals widespread patchy or coalescent airspace opacities
that are usually more apparent in the dependent lung zones.
Chronic thromboembolic pulmonary hypertension (CTEPH) can occur in 1% to
5% of patients after acute pulmonary embolism. Underlying hypercoagulable
state, pulmonary arteriopathy, and impaired fibrinolytic mechanism have been
implicated as the etiology. The presentation is similar to this patient with
progressive dyspnea, peripheral edema, exertional chest pain, and syncope or
near-syncope. A definitive prior diagnosis of pulmonary embolus may be absent
in many patients. A helpful finding from the physical examination, in a subset of
patients, is the presence of high pitched blowing flow murmurs over the lung
fields (due to turbulent flow through recanalized or partially
obstructed pulmonary arteries). Chest radiography may show areas of
prominent pulmonary arteries, hypoperfusion or hyperperfusion of lung fields,
and right ventricular enlargement (in lateral radiograph).
The ventilation-perfusion (V/Q) lung scanning is the initial imaging procedure of
choice in patients with CTEPH. Patients with CTEPH have one or more
segmental or larger mismatched ventilation-perfusion defects, whereas patients
with distal small vessel types (more common cause of pulmonary hypertension)
have subsegmental defects presenting as a normal or mottled perfusion scan.
Patients with CTEPH are more likely to have an abnormal V/Q lung scan than an
abnormal CT pulmonary angiography.
Pulmonary capillaritis or alveolar capillaritis is characterized by neutrophilic
infiltration of the lung interstitium, resulting in alveolar septanecrosis and diffuse
alveolar hemorrhage. The disease course is rapidly progressive cough,
hemoptysis, fever, and dyspnea, but one third of patients may not have any
hemoptysis. Bronchoalveolar lavage shows increasingly hemorrhagic fluid on
sequential bronchoalveolar lavage and hemosiderin laden macrophages in
microscopic examination. The chest radiograph is nonspecific and most
commonly shows new or fleeting patchy or diffuse alveolar opacities. Laboratory
findings are nonspecific. The erythrocyte sedimentation rate is usually high along
with leukocytosis and progressive drop in hemoglobin. A sensitive marker for
pulmonary capillaritis and diffuse alveolar hemorrhage is a sequential increase in
diffusing capacity for carbon monoxide, which is due to presence of hemoglobin
in the alveolar compartment.
Educational Objective: To recognize that pulmonary arteries may be involved
due to Takayasu’s arteritis either in isolation or along with involvement of other
vessels.

Case 59
A 49-year-old woman is admitted to the hospital for a 2 to 3 week history of
swelling in her right lower leg with acute chest pain and dyspnea and is
diagnosed with deep venous thrombosis and pulmonary embolism. She has
been healthy otherwise. She has no past history of thrombosis or pregnancy
loss, no family history of thrombosis, no history of oral contraceptive therapy or
hormonal replacement, and no recent history of trauma or prolonged immobility.
Laboratory studies drawn before initiating therapy demonstrate positive lupus
anticoagulant by dilute Russell viper venom testing and high levels of
immunoglobulin IgG and IgM anticardiolipin and anti–beta-2 glycoprotein
antibodies.

Click here for references

After initial therapy with low molecular weight heparin, which of the following
would be the most appropriate approach to treating this patient?

A. Oral rivaroxaban indefinitely

B. Warfarin therapy with target International Normalization Ratio (INR) 2.0


to 3.0 indefinitely

C. Warfarin therapy with target INR 2.0 to 3.0 for 6 months, then low dose
aspirin indefinitely

D. Warfarin therapy with target INR 3.0 to 4.0 indefinitely

Rationale
This patient has a new onset, unexplained deep venous thrombosis with
pulmonary embolus, associated with positive studies for lupus anticoagulant and
high levels of anticardiolpin and anti–beta-2 glycoprotein antibodies. Even though
confirmatory laboratory studies have not been repeated 12 weeks later, the
patient’s presentation is most consistent with a diagnosis of antiphospholipid
syndrome (APS). The current standard of care for the long-term management of
APS after an initial venous thromboembolic event is long-term warfarin therapy to
maintain the INR between 2.0 and 3.0.
As of the writing of this question, the novel oral anticoagulants, such as direct
thrombin inhibitors or direct factor Xa inhibitors, have not been adequately
evaluated for the treatment of APS. Although these agents have the potential to
be effective, individual case reports of rivaroxaban and dabigatran for patients
with severe or refractory APS have failed to demonstrate efficacy. Randomized
clinical trials evaluating the role of rivaroxaban (a direct factor Xa inhibitor) in the
management of APS are currently underway.
In patients with a first venous event and a known transient precipitating factor
(trauma, pregnancy, estrogen use) or low level antiphospholipid studies, a 3 to 6
month course of anticoagulation therapy can be considered. However,
observational studies in patients with APS have demonstrated an increased risk
of recurrent events in patients stopping anticoagulant therapy, and lifelong
therapy is warranted for most patients with APS.
Previous recommendations, based on observational studies in the 1990s,
suggested regimens that maintained the INR between 3.0 and 4.0 were needed
to prevent recurrent thrombotic events. However, subsequent studies have
shown that high-intensity warfarin therapy is associated with a higher rate of
bleeding complications and has no demonstrable additional benefit compared
with lower-intensity dosing.

Case 60
A 55-year-old woman presents to clinic for fatigue and pruritus. She has had a
diagnosis of primary Sjögren’s syndrome for the past 2 years that manifested as
dry eyes and mouth. The patient states that her symptoms started approximately
6 months ago and that the fatigue is associated with daytime somnolence, which
is interfering with her daily activities. She has also noted a persistent pruritus that
is worse at night. The patient notes a mild hyperpigmentation of her skin over the
past 3 months.
On examination, her temperature is 98.8°F (37.1°C), heart rate is 80 bpm, and
blood pressure is 120/80 mm Hg. Weight is 140 lb (65 kg), and height is 64 in
(163 cm). The skin examination is remarkable for multiple areas of excoriations
from scratching and mild hyperpigmentation. Cardiovascular and pulmonary
examinations are unremarkable. The abdominal examination is noted for
hepatomegaly but negative for splenomegaly. No lower extremity edema is
noted, and the neurologic examination is normal.
The patient had blood drawn last week from a health fair at work, and the results
of those tests show a normal complete blood count with differential and a
comprehensive metabolic panel with elevated alkaline phosphatase and total
cholesterol levels. Liver enzymes and serum bilirubin levels are within normal
limits.
Click here for references

Which of the following antibodies is most likely to be abnormal in this patient?

A. Anticentromere antibodies

B. Antimitochondrial antibodies

C. Antinuclear dot antibodies (anti-sp100)

D. Antinuclear pore antibodies (anti-gp210)

Rationale

This patient has a diagnosis of primary biliary cirrhosis (PBC). Fatigue and
pruritus are the most common presenting symptoms. Other symptoms include
musculoskeletal complaints, xanthomas, skin hyperpigmentation, right upper
quadrant abdominal pain, cognitive impairment, osteoporosis/osteomalacia, and
hepatosplenomegaly. Sicca symptoms occur in approximately 50% of patients
with PBC.
The hallmark of diagnosis is the detection of antimitochondrial antibodies (AMA) ,
which are present in 90% to 95% of patients. It isAMA antibodies are the most
sensitive serologic marker for PBC. Prevalence in the general population is low,
ranging from 0.16% to 1%. The AMA is a strong predictor for the development of
PBC but not helpful in monitoring the clinical course of the disease.
Anticentromere antibodies are seen at a prevalence rate of 16% to 30% in PBC.
The presence of an anticentromere antibody in a patient with PBC is a significant
risk factor for the development of portal hypertension. Antinuclear dot antibodies
(anti-sp100) are seen at a prevalence rate of 30% to 50%. However, this
antibody has been found in a number of other autoimmune disorders also and
may support the hypothesis of bacterial infections causing induction of PBC
autoimmunity. Antinuclear pore antibodies (anti-gp210) are seen at a prevalence
rate of 30% to 50% in PBC. Studies have shown patients with PBC with high
levels of anti-gp210 have a higher risk of progression to end-stage liver failure
than those without. Some of these antinuclear antibody patterns may not be
reported routinely by immunology laboratories.

You might also like